[obm-l] Re: [obm-l] Re: [obm-l] Re: [obm-l] FW: pentágono cíclico

2015-04-25 Por tôpico Rogerio Ponce
Perfeito, Ralph!
E a solucao mostra que dados os comprimentos dos lados, qualquer poligono
pode ser ciclico.
[]'s
Rogerio Ponce

2015-04-25 0:57 GMT-03:00 Ralph Teixeira :

> Hmmm... Mas *faz* sentido -- se voce dah apenas os 4 comprimentos dos
> lados, o quadrilatero nao estah fixo. Acho que eh sempre possivel deformar
> seus angulos ateh ele ficar ciclico...
>
> 2015-04-24 20:41 GMT-03:00 Sergio Lima :
>
>> Oi Ralph,
>>
>> Desculpe a minha ignorância, mas o seu método funcionaria
>> a princípio para qualquer quadrilátero, o que não faz sentido.
>>
>> De todo modo, vindo do Luís, acredito que seja um problema
>> de contrução com régua e compasso.
>>
>> Abraço,
>> Sergio
>>
>> On Friday, April 24, 2015, Ralph Teixeira  wrote:
>>
>>> Construir, tipo, com regua e compasso? Ou, num sentido mais teorico e
>>> geral?
>>>
>>> Pegue um crculo com raio 300, marque pontos ABCDEF tal que as cordas
>>> AB, BC, CD, DE e EF tenham os comprimentos pedidos. Agora diminua o raio do
>>> circulo ateh que A=F... Hmmm... alguem tem algum motivo para essa
>>> construcao NAO dar certo? (Eu queria fazer no Geogebra, mas estou sem
>>> Geogebra neste instante).
>>>
>>> Tem que fazer algum argumento do tipo "nenhum dos lados eh maior do qu
>>> eo diametro do circulo" durante minha construcao, mas acho *ACHO* que isto
>>> eh equivalente a cada lado ser menor que a soma dos outros.
>>>
>>> Abraco, Ralph
>>>
>>> 2015-04-24 11:18 GMT-03:00 Luís :
>>>
 Sauda,c~oes,

 Alguém saberia responder ?

 Abraços,
 Luís

 > Date: Fri, 24 Apr 2015 10:31:14 +0100
 > Subject: pentágono cíclico
 >
 > Estimado, Luís
 >
 > ¿Es posible construir un pentágono inscrito en una circunferencia
 cuyos
 > lados a1,a2,a3,a4,a5 tiene longitudes: 13, 13, 5 + 12*Sqrt[3],
 > 20*Sqrt[3], -5 + 12*Sqrt[3]?
 >
 > Un saludo
 > Angel

 --
 Esta mensagem foi verificada pelo sistema de antivírus e
 acredita-se estar livre de perigo.

>>>
>>>
>>> --
>>> Esta mensagem foi verificada pelo sistema de antivírus e
>>> acredita-se estar livre de perigo.
>>
>>
>> --
>> Esta mensagem foi verificada pelo sistema de antivírus e
>> acredita-se estar livre de perigo.
>
>
>
> --
> Esta mensagem foi verificada pelo sistema de antivírus e
> acredita-se estar livre de perigo.

-- 
Esta mensagem foi verificada pelo sistema de antiv�rus e
 acredita-se estar livre de perigo.



[obm-l] Re: [obm-l] Re: [obm-l] FW: pentágono cíclico

2015-04-24 Por tôpico Ralph Teixeira
Hmmm... Mas *faz* sentido -- se voce dah apenas os 4 comprimentos dos
lados, o quadrilatero nao estah fixo. Acho que eh sempre possivel deformar
seus angulos ateh ele ficar ciclico...

2015-04-24 20:41 GMT-03:00 Sergio Lima :

> Oi Ralph,
>
> Desculpe a minha ignorância, mas o seu método funcionaria
> a princípio para qualquer quadrilátero, o que não faz sentido.
>
> De todo modo, vindo do Luís, acredito que seja um problema
> de contrução com régua e compasso.
>
> Abraço,
> Sergio
>
> On Friday, April 24, 2015, Ralph Teixeira  wrote:
>
>> Construir, tipo, com regua e compasso? Ou, num sentido mais teorico e
>> geral?
>>
>> Pegue um crculo com raio 300, marque pontos ABCDEF tal que as cordas
>> AB, BC, CD, DE e EF tenham os comprimentos pedidos. Agora diminua o raio do
>> circulo ateh que A=F... Hmmm... alguem tem algum motivo para essa
>> construcao NAO dar certo? (Eu queria fazer no Geogebra, mas estou sem
>> Geogebra neste instante).
>>
>> Tem que fazer algum argumento do tipo "nenhum dos lados eh maior do qu eo
>> diametro do circulo" durante minha construcao, mas acho *ACHO* que isto eh
>> equivalente a cada lado ser menor que a soma dos outros.
>>
>> Abraco, Ralph
>>
>> 2015-04-24 11:18 GMT-03:00 Luís :
>>
>>> Sauda,c~oes,
>>>
>>> Alguém saberia responder ?
>>>
>>> Abraços,
>>> Luís
>>>
>>> > Date: Fri, 24 Apr 2015 10:31:14 +0100
>>> > Subject: pentágono cíclico
>>> >
>>> > Estimado, Luís
>>> >
>>> > ¿Es posible construir un pentágono inscrito en una circunferencia
>>> cuyos
>>> > lados a1,a2,a3,a4,a5 tiene longitudes: 13, 13, 5 + 12*Sqrt[3],
>>> > 20*Sqrt[3], -5 + 12*Sqrt[3]?
>>> >
>>> > Un saludo
>>> > Angel
>>>
>>> --
>>> Esta mensagem foi verificada pelo sistema de antivírus e
>>> acredita-se estar livre de perigo.
>>>
>>
>>
>> --
>> Esta mensagem foi verificada pelo sistema de antivírus e
>> acredita-se estar livre de perigo.
>
>
> --
> Esta mensagem foi verificada pelo sistema de antivírus e
> acredita-se estar livre de perigo.

-- 
Esta mensagem foi verificada pelo sistema de antiv�rus e
 acredita-se estar livre de perigo.



[obm-l] Re: [obm-l] FW: pentágono cíclico

2015-04-24 Por tôpico Sergio Lima
Oi Ralph,

Desculpe a minha ignorância, mas o seu método funcionaria
a princípio para qualquer quadrilátero, o que não faz sentido.

De todo modo, vindo do Luís, acredito que seja um problema
de contrução com régua e compasso.

Abraço,
Sergio

On Friday, April 24, 2015, Ralph Teixeira  wrote:

> Construir, tipo, com regua e compasso? Ou, num sentido mais teorico e
> geral?
>
> Pegue um crculo com raio 300, marque pontos ABCDEF tal que as cordas
> AB, BC, CD, DE e EF tenham os comprimentos pedidos. Agora diminua o raio do
> circulo ateh que A=F... Hmmm... alguem tem algum motivo para essa
> construcao NAO dar certo? (Eu queria fazer no Geogebra, mas estou sem
> Geogebra neste instante).
>
> Tem que fazer algum argumento do tipo "nenhum dos lados eh maior do qu eo
> diametro do circulo" durante minha construcao, mas acho *ACHO* que isto eh
> equivalente a cada lado ser menor que a soma dos outros.
>
> Abraco, Ralph
>
> 2015-04-24 11:18 GMT-03:00 Luís  >:
>
>> Sauda,c~oes,
>>
>> Alguém saberia responder ?
>>
>> Abraços,
>> Luís
>>
>> > Date: Fri, 24 Apr 2015 10:31:14 +0100
>> > Subject: pentágono cíclico
>> >
>> > Estimado, Luís
>> >
>> > ¿Es posible construir un pentágono inscrito en una circunferencia cuyos
>> > lados a1,a2,a3,a4,a5 tiene longitudes: 13, 13, 5 + 12*Sqrt[3],
>> > 20*Sqrt[3], -5 + 12*Sqrt[3]?
>> >
>> > Un saludo
>> > Angel
>>
>> --
>> Esta mensagem foi verificada pelo sistema de antivírus e
>> acredita-se estar livre de perigo.
>>
>
>
> --
> Esta mensagem foi verificada pelo sistema de antivírus e
> acredita-se estar livre de perigo.

-- 
Esta mensagem foi verificada pelo sistema de antiv�rus e
 acredita-se estar livre de perigo.



[obm-l] Re: [obm-l] FW: pentágono cíclico

2015-04-24 Por tôpico Ralph Teixeira
Construir, tipo, com regua e compasso? Ou, num sentido mais teorico e geral?

Pegue um crculo com raio 300, marque pontos ABCDEF tal que as cordas
AB, BC, CD, DE e EF tenham os comprimentos pedidos. Agora diminua o raio do
circulo ateh que A=F... Hmmm... alguem tem algum motivo para essa
construcao NAO dar certo? (Eu queria fazer no Geogebra, mas estou sem
Geogebra neste instante).

Tem que fazer algum argumento do tipo "nenhum dos lados eh maior do qu eo
diametro do circulo" durante minha construcao, mas acho *ACHO* que isto eh
equivalente a cada lado ser menor que a soma dos outros.

Abraco, Ralph

2015-04-24 11:18 GMT-03:00 Luís :

> Sauda,c~oes,
>
> Alguém saberia responder ?
>
> Abraços,
> Luís
>
> > Date: Fri, 24 Apr 2015 10:31:14 +0100
> > Subject: pentágono cíclico
> >
> > Estimado, Luís
> >
> > ¿Es posible construir un pentágono inscrito en una circunferencia cuyos
> > lados a1,a2,a3,a4,a5 tiene longitudes: 13, 13, 5 + 12*Sqrt[3],
> > 20*Sqrt[3], -5 + 12*Sqrt[3]?
> >
> > Un saludo
> > Angel
>
> --
> Esta mensagem foi verificada pelo sistema de antivírus e
> acredita-se estar livre de perigo.
>

-- 
Esta mensagem foi verificada pelo sistema de antiv�rus e
 acredita-se estar livre de perigo.



Re: [obm-l] FW: Polinômio

2014-11-17 Por tôpico Artur Costa Steiner
Esqueça meu outro email, está errado, esqueci dos fatoriais no denominador. É 
preciso elaborar mais.

Artur Costa Steiner

> Em 16/11/2014, às 23:15, marcone augusto araújo borges 
>  escreveu:
> 
> 
> 
> From: marconeborge...@hotmail.com
> To: obm-l@mat.puc-rio.br
> Subject: Polinômio
> Date: Sun, 16 Nov 2014 12:34:12 +
> 
> Prove que não existe polinômio, P(x), com coeficientes inteiros satisfazendo 
> as
> igualdades: P(7) = 5 e P(15) = 9
> 
> -- 
> Esta mensagem foi verificada pelo sistema de antivírus e 
> acredita-se estar livre de perigo. 
> = 
> Instrugues para entrar na lista, sair da lista e usar a lista em 
> http://www.mat.puc-rio.br/~obmlistas/obm-l.html 
> =

-- 
Esta mensagem foi verificada pelo sistema de antiv�rus e
 acredita-se estar livre de perigo.



Re: [obm-l] FW: Polinômio

2014-11-17 Por tôpico Artur Costa Steiner
Suponhamos que, para o polinômio P, de coeficientes inteiros e grau n, tenhamos 
P(7) = 5 e P(15) = 9. Pelo Teorema de Taylor,

P(x) = P(7) + (x - 7) P'(7) + ((x  - 7)^2)/2! P''(7)  ((x - 7)^n)/n! P_n(7)

Então,

P(15) = 5 + 8 P'(7) + (8^2)/2! P''(7)  (8^n)/n! P_n(7), P_n a n-gésima 
derivada

Como todas as derivadas de P têm coeficientes inteiros, a soma 8 P'(7) + 
(8^2)/2! P''(7)  (8^n)/n! P_n(7) é múltipla de 8. E como P(15) = 9, 
concluímos que

9 = 5 + 8k para algum inteiro k. 

k = 1/2, contrariamente ao fato de k é inteiro. Logo, este polinômio não existe.


Artur Costa Steiner

> Em 16/11/2014, às 23:15, marcone augusto araújo borges 
>  escreveu:
> 
> 
> 
> From: marconeborge...@hotmail.com
> To: obm-l@mat.puc-rio.br
> Subject: Polinômio
> Date: Sun, 16 Nov 2014 12:34:12 +
> 
> Prove que não existe polinômio, P(x), com coeficientes inteiros satisfazendo 
> as
> igualdades: P(7) = 5 e P(15) = 9
> 
> -- 
> Esta mensagem foi verificada pelo sistema de antivírus e 
> acredita-se estar livre de perigo. 
> = 
> Instrugues para entrar na lista, sair da lista e usar a lista em 
> http://www.mat.puc-rio.br/~obmlistas/obm-l.html 
> =

-- 
Esta mensagem foi verificada pelo sistema de antiv�rus e
 acredita-se estar livre de perigo.



[obm-l] Re: [obm-l] Re: [obm-l] FW: Congruência(não quero a solução)

2014-06-16 Por tôpico Pedro José
Boa tarde!

Desculpe-me, mas não sei fazer de uma forma mais elegante. Porém, no braço
sai usando a conservação da soma, do produto e da potência nas classes de
congruência módulo p, temos.


1^10 ≡ 1 mod 101
2^10 ≡ x mod 101
3^5  ≡  y mod 101 ==> 3^10 ≡  y^2 mod 101
4^10 ≡ x^2 mod 101
5^3 ≡ k mod 101;  5^5  ≡ 25*k mod 101; 5^10 ≡ (25*k mod 101)^2 mod 101
6^10 ≡ x*y mod 101
7^3 ≡ m mod 101; 7^4 ≡ 7* m mod 101 ≡ n mod 101; 7^6 ≡ m^2 mod 101 ≡ p mod
101; 7^10 ≡ m*p mod 101
8^10 ≡  x^3 mod 101 ou 8^10 ≡  x* (4^10 mod 101) mod 101
9^10≡  (3^10 mod101)^2 mod 101
10^10 ≡  x*(5^10 mod 101)

Depois soma tudo e dá S, onde S ≡  74 mod 101. Portanto 101 não divide a
soma dasérie, como é solicitado para provar. Não há algo errado no
enunciado?

Saudações,
PJMS



Em 15 de junho de 2014 13:18, saulo nilson 
escreveu:

> S= 1^10 + 2^10 + ... + 100^10=
> (x+y)^10=x^10+C10,1x^9y+c10.2x^8y^2+c10,3x^7y^3+c10,4x^6y^4++y^10
> x^10+y^10=(x+y)^10-(x+y)f(x,y)
> e x+y=101., logo S e divisivel por 101
>
>
> 2014-06-13 19:57 GMT-03:00 marcone augusto araújo borges <
> marconeborge...@hotmail.com>:
>
>  100^10,quro dizer.
>>
>> --
>> From: marconeborge...@hotmail.com
>> To: obm-l@mat.puc-rio.br
>> Subject: FW: Congruência(não quero a solução)
>> Date: Fri, 13 Jun 2014 22:32:19 +
>>
>>
>> A última parcela na segunda linha é 10^100,e não 10^10
>>
>> --
>> From: marconeborge...@hotmail.com
>> To: obm-l@mat.puc-rio.br
>> Subject: Congruência(não quero a solução)
>> Date: Fri, 13 Jun 2014 22:22:29 +
>>
>> Eu gostaria de alguma pista para a questão:
>> Mostre que 101 divide 1^10 + 2^10 + ... + 10^10
>> Se não me engano 1^100 + 2^100 + ... + 100^100 = = -1 (mod 101)
>> Claro que 101 divide 1+2+...+ 100,mas...
>>
>> --
>> Esta mensagem foi verificada pelo sistema de antivírus e
>> acredita-se estar livre de perigo.
>>
>
>
> --
> Esta mensagem foi verificada pelo sistema de antivírus e
> acredita-se estar livre de perigo.
>

-- 
Esta mensagem foi verificada pelo sistema de antiv�rus e
 acredita-se estar livre de perigo.



[obm-l] Re: [obm-l] FW: Congruência(não quero a solução)

2014-06-15 Por tôpico saulo nilson
S= 1^10 + 2^10 + ... + 100^10=
(x+y)^10=x^10+C10,1x^9y+c10.2x^8y^2+c10,3x^7y^3+c10,4x^6y^4++y^10
x^10+y^10=(x+y)^10-(x+y)f(x,y)
e x+y=101., logo S e divisivel por 101


2014-06-13 19:57 GMT-03:00 marcone augusto araújo borges <
marconeborge...@hotmail.com>:

> 100^10,quro dizer.
>
> --
> From: marconeborge...@hotmail.com
> To: obm-l@mat.puc-rio.br
> Subject: FW: Congruência(não quero a solução)
> Date: Fri, 13 Jun 2014 22:32:19 +
>
>
> A última parcela na segunda linha é 10^100,e não 10^10
>
> --
> From: marconeborge...@hotmail.com
> To: obm-l@mat.puc-rio.br
> Subject: Congruência(não quero a solução)
> Date: Fri, 13 Jun 2014 22:22:29 +
>
> Eu gostaria de alguma pista para a questão:
> Mostre que 101 divide 1^10 + 2^10 + ... + 10^10
> Se não me engano 1^100 + 2^100 + ... + 100^100 = = -1 (mod 101)
> Claro que 101 divide 1+2+...+ 100,mas...
>
> --
> Esta mensagem foi verificada pelo sistema de antivírus e
> acredita-se estar livre de perigo.
>

-- 
Esta mensagem foi verificada pelo sistema de antiv�rus e
 acredita-se estar livre de perigo.



Re: [obm-l] FW: Fibonacci

2013-03-31 Por tôpico Cláudio Gustavo
Devemos usar a igualdade auxiliar: F_m+n+1 = F_m+1F_n+1 + F_mF_n e a igualdade 
na forma mais geral:
F_m+n+k = F_m+1F_n+1F_k+1 + F_mF_nF_k - F_m-1F_n-1F_k-1. Em q o caso pedido 
ocorre qdo m=n=k.
Aplicando indução em k e adotando os casos F_m+n+k e F_m+n+k+1, somando e 
fatorando obteremos: F_m+n+k + F_m+n+k+1 = F_m+1F_n+1(F_k+1 + F_k+2) + 
F_mF_n(F_k + F_k+1) - F_m-1F_n-1(F_k-1 + F_k). Sendo a última igualdade fácil 
de verificar.

Abraços
Claudio Gustavo

Em 30/03/2013, às 10:51, marcone augusto araújo borges 
 escreveu:

> 
> 
> From: marconeborge...@hotmail.com
> To: obm-l@mat.puc-rio.br
> Subject: Fibonacci
> Date: Fri, 29 Mar 2013 14:04:22 +
> 
> Mostre por indução que F_3n = F^3_(n) + F^3_(n+1) - F^3_(n-1)
> 
> -- 
> Esta mensagem foi verificada pelo sistema de antivírus e 
> acredita-se estar livre de perigo.

-- 
Esta mensagem foi verificada pelo sistema de antiv�rus e
 acredita-se estar livre de perigo.



Re: [obm-l] FW: Fibonacci

2013-03-31 Por tôpico terence thirteen
Tem certeza?

F(6) é muito menor que o cubo de F(3)...

-- 
Esta mensagem foi verificada pelo sistema de antiv�rus e
 acredita-se estar livre de perigo.



[obm-l] RE: [obm-l] Re: [obm-l] RE: [obm-l] Re: [obm-l] FW: Tentei e não consegui(geometria)

2012-10-26 Por tôpico marcone augusto araújo borges

Ok.Assim fica uma solução melhor,e é sempre bom uma solução diferente.Valeu 
mesmo.
 Date: Thu, 25 Oct 2012 09:49:14 -0200
Subject: [obm-l] Re: [obm-l] RE: [obm-l] Re: [obm-l] FW: Tentei e não 
consegui(geometria)
From: gabrieldala...@gmail.com
To: obm-l@mat.puc-rio.br

Como eu falei tem como usar a fórmula de area (1/2).a.b.senCom ela e com 
semelhança você mostra que as diagonais dividem o trapézio em 4 triangulos de 
area A2, A3 e os outros dois tem area raiz(A2.A3), dessa maneira voce teria
A1=A2+2.raiz(A2.A3)+A3=(raiz(A2)+raiz(A3))^2
Gabriel Dalalio
Em 25 de outubro de 2012 09:23, marcone augusto araújo borges 
 escreveu:





Eu consegui.Beleza.Mas não gostei muito do modo como fiz.
Escrevi a soma das raízes das areas dos triangulos e elevei essa soma ao 
quadrado.
Comparei o resultado com a area do trapezio e comprovei a igualdade procurada.

A1 = (a+b)*(h1+h2)/2
Por semelhança h1*b = h2*a (1),ou seja,h1 = h2*a/b
Substituindo h1 em A2 = h1*a/2,temos A2=h2*a^2/2b.Calculando [(raiz(A2) + 
raiz(A3)]^2 e usando,de (1),(a+b)/b = (h1 + h2)/h2,encontrei A1.
Dai,conclui o resultado procurado.

Será que alguem(generosamente)apresentaria uma solução mais interessante? 
 
Date: Tue, 23 Oct 2012 09:28:00 -0300
Subject: [obm-l] Re: [obm-l] FW: Tentei e não consegui(geometria)
From: gabrieldala...@gmail.com

To: obm-l@mat.puc-rio.br

AB=aCD=baltura do trapezio=haltura de ABE em relação a AB=h1altura de CDE em 
relação a CD=h2

A1=(a+b)*h/2A2=h1*a/2A3=h2*b/2
h=h1+h2
e pela semelhança entre ABE e CDE:h1/a=h2/b
Com essas equações você pode provar o enunciadoTambém dá para conseguir o 
resultado usando formula da área com seno...


Gabriel Dalalio
2012/10/23 marcone augusto araújo borges 






 
From: marconeborge...@hotmail.com
To: obm-l@mat.puc-rio.br


Subject: Tentei e não consegui(geometria)
Date: Tue, 23 Oct 2012 02:22:05 +








Seja um trapezio ABCD de bases AB e CD.AS diagonais AC e BD se encontram em E.
Sejam A1,A2,A3 as areas do trapezio,do triangulo ABE e do triangulo 
CDE,respectivamente.
Mostre que raiz(A1) = raiz(A2) + raiz(A3). 



  

  

  

[obm-l] Re:[obm-l] FW: Tentei e não consegui(geometria)

2012-10-25 Por tôpico Eduardo Wilner
Pela semelhança dos triangulos ABE (base AB=a, altura h2) e CDE(base CD=b, 
altura h3) ,
  (a/h2) = (b/h3) = k.
Assim,  A2 = a*h2/2 = (k/2)(h2)^2  => h2 = \sqrt (2*A2/k) (I)

Analogamente  A3 = (k/2)(h3)^2   =>  h3 = \sqrt(2*A3/k)   (II)

 A1 = (h/2)(a+b) = (h/2)k(h2+h3) = (k/2)h^2 (pois  (IV) h2+h3=h) => h 
=\sqrt(2*A1/k) (III)

Substituindo  (I), (II) e (III) em (IV) e simplificando, obtemos a relaçao 
proposta.

[ ]'s


[obm-l] RE: [obm-l] Re: [obm-l] FW: Tentei e não consegui(geometria)

2012-10-25 Por tôpico marcone augusto araújo borges

Eu consegui.Beleza.Mas não gostei muito do modo como fiz.Escrevi a soma das 
raízes das areas dos triangulos e elevei essa soma ao quadrado.Comparei o 
resultado com a area do trapezio e comprovei a igualdade procurada.A1 = 
(a+b)*(h1+h2)/2Por semelhança h1*b = h2*a (1),ou seja,h1 = h2*a/bSubstituindo 
h1 em A2 = h1*a/2,temos A2=h2*a^2/2b.Calculando [(raiz(A2) + raiz(A3)]^2 e 
usando,de (1),(a+b)/b = (h1 + h2)/h2,encontrei A1.Dai,conclui o resultado 
procurado.Será que alguem(generosamente)apresentaria uma solução mais 
interessante? 
 Date: Tue, 23 Oct 2012 09:28:00 -0300
Subject: [obm-l] Re: [obm-l] FW: Tentei e não consegui(geometria)
From: gabrieldala...@gmail.com
To: obm-l@mat.puc-rio.br

AB=aCD=baltura do trapezio=haltura de ABE em relação a AB=h1altura de CDE em 
relação a CD=h2
A1=(a+b)*h/2A2=h1*a/2A3=h2*b/2
h=h1+h2
e pela semelhança entre ABE e CDE:h1/a=h2/b
Com essas equações você pode provar o enunciadoTambém dá para conseguir o 
resultado usando formula da área com seno...

Gabriel Dalalio
2012/10/23 marcone augusto araújo borges 






 
From: marconeborge...@hotmail.com
To: obm-l@mat.puc-rio.br

Subject: Tentei e não consegui(geometria)
Date: Tue, 23 Oct 2012 02:22:05 +








Seja um trapezio ABCD de bases AB e CD.AS diagonais AC e BD se encontram em E.
Sejam A1,A2,A3 as areas do trapezio,do triangulo ABE e do triangulo 
CDE,respectivamente.
Mostre que raiz(A1) = raiz(A2) + raiz(A3). 


  

  

RE: [obm-l] FW: Quadrados e cubos

2012-10-11 Por tôpico marcone augusto araújo borges

Onde tá escrito precedido deveria estar escrito seguido.
 From: marconeborge...@hotmail.com
To: obm-l@mat.puc-rio.br
Subject: [obm-l] FW: Quadrados e cubos
Date: Thu, 11 Oct 2012 00:27:26 +






 
From: marconeborge...@hotmail.com
To: obm-l@mat.puc-rio.br
Subject: Quadrados e cubos
Date: Wed, 10 Oct 2012 19:47:38 +








8 é um cubo precedido de um quadrado.Existem outros pares de inteiros positivos 
 n e n +1 tais que o primeiro é um cubo e o segundo,um quadrado?

  

[obm-l] Re: [obm-l] FW: solicitação

2012-07-27 Por tôpico geonir paulo schnorr
Considerando os conjuntos
A:  pessoas que tem 1,62 m ou mais;
B:  pessoas que tem 1,62 m ou menos;
A inter B = pessoas com 1,62 m.

Em teoria de probabilidade temos que:

A + B - (AinterB)  = AUB

assim,

95% + 8% - (AinterB) = 100%
(AinterB) = 103% - 100%
(AinterB) = 3%

*(AinterB = A intersecção com B)


Em 27 de julho de 2012 14:56, Luís Lopes  escreveu:

>  Sds,
>
> Reenviando.
>
> --
> From: qed_te...@hotmail.com
> To: obm-l@mat.puc-rio.br
> Subject: FW: solicitação
> Date: Fri, 27 Jul 2012 12:12:47 +
>
> Sds,
>
> Alguém pode ajudar? Obrigado.
>
> Luis
>
>
>
> Subject: solicitação
> Date: Fri, 27 Jul 2012 08:14:16 -0300
>
> Em uma escola 95% das pessoas tem 1,62 m ou mais e 8% 1,62,m ou menos.
> Calcule o % de quem tem 1,62 .Solicito uma ajuda nesta questão.
>
>


[obm-l] Re: [obm-l] FW: solicitação

2012-07-27 Por tôpico Andre Araujo ЄЭ
Luís,

100% das pessoas = 95% (> ou = 1,62m) + 8% (< ou = 1,62m) - x% (=1,62m).
Assim, x=3%.

Abs, AA.

Em sexta-feira, 27 de julho de 2012, Luís Lopes escreveu:

>  Sds,
>
> Alguém pode ajudar? Obrigado.
>
> Luis
>
>
>
> Subject: solicitação
> Date: Fri, 27 Jul 2012 08:14:16 -0300
>
>  Em uma escola 95% das pessoas tem 1,62 m ou mais e 8% 1,62,m ou menos.
> Calcule o % de quem tem 1,62 .Solicito uma ajuda nesta questão.
>
>
>


Re: [obm-l] FW: PROBLEMAS..... de concurso??

2012-06-26 Por tôpico Bruno França dos Reis
Diferem por uma constante quer dizer: "a diferença é uma constante". Em
símbolos: sejam P1 e P2 trinômios do segundo grau; dizer que P1 e P2
diferem por uma constante é equivalente a dizer que P1 - P2 = k, k real.

Note que "um é igual ao outro vezes um k" é algo completamente diferente.
(ex: P1(x) = x^2 + x + 1 e P2(x) = x^2 + x + 2, temos que (P2 - P1)(x) = 1
(ou seja, diferem por uma constante) e não existe k tal que P2 = kP1).

--
Bruno FRANÇA DOS REIS

msn: brunoreis...@hotmail.com
skype: brunoreis666
tel: +55 11 9961-7732

http://brunoreis.com
http://brunoreis.com/tech (en)
http://brunoreis.com/blog (pt)

GPG Key: http://brunoreis.com/bruno-public.key

e^(pi*i)+1=0


2012/6/26 marcone augusto araújo borges 

>  diferem por uma constante quer dizer que  um é igual ao outro vezes um k?
> nesse caso as raizes de um são iguais as raizes do outro
>
>  --
> From: qed_te...@hotmail.com
> To: obm-l@mat.puc-rio.br
> Subject: [obm-l] FW: PROBLEMAS. de concurso??
> Date: Mon, 25 Jun 2012 13:56:08 +
>
>
> Sauda,c~oes,
>
> Me mandaram os problemas abaixo com o gabarito.
> Que tirei para ver as respostas justificadas de vocês,
> sempre melhores e mais espertas do que as minhas.
>
> Faço isso por 3 razões:
>
> 1) para me ajudarem;
> 2) para dar uma melhor resposta ao Fernando;
> 3) para tirar a lista do silêncio e moviment'a-la
> um pouco.
>
> [ ]'s
> Lu'is
>
>
>
>
> Prezado Luis,
>
>
>
> Gostaria de sua ajuda para as seguintes questões:
>
>
>
> 1)Se dois trinômios do 2º grau possuem as mesmas raízes então:
> a) eles são necessariamente iguais.
> b) eles assumem necessariamente um mínimo ou um máximo no mesmo ponto.
> c) eles diferem por uma constante.
> d) suas concavidades são de mesmo sentido.
> e) nenhuma das anteriores.
> R. letra  "a letra d é f'acil de ser eliminada. hum
>
> a letra a também"
>
>
>
> 2)Dados três pontos no plano cartesiano, não colineares e com abscissas
> distintas duas a duas, o número de funções quadráticas que podem ser
> encontradas de maneira que esses pontos pertençam aos seus gráficos é:
>
> a) 0  b) 1  c) 2  d) 3
>
> R.letra .?
>
>
>


RE: [obm-l] FW: PROBLEMAS..... de concurso??

2012-06-26 Por tôpico marcone augusto araújo borges

diferem por uma constante quer dizer que  um é igual ao outro vezes um k?
nesse caso as raizes de um são iguais as raizes do outro
 



From: qed_te...@hotmail.com
To: obm-l@mat.puc-rio.br
Subject: [obm-l] FW: PROBLEMAS. de concurso??
Date: Mon, 25 Jun 2012 13:56:08 +




Sauda,c~oes, 


Me mandaram os problemas abaixo com o gabarito. 
Que tirei para ver as respostas justificadas de vocês, 
sempre melhores e mais espertas do que as minhas. 


Faço isso por 3 razões:





1) para me ajudarem; 
2) para dar uma melhor resposta ao Fernando; 
3) para tirar a lista do silêncio e moviment'a-la 
um pouco. 


[ ]'s 
Lu'is 




 
Prezado Luis,
 
Gostaria de sua ajuda para as seguintes questões:




1)Se dois trinômios do 2º grau possuem as mesmas raízes então:
a) eles são necessariamente iguais.
b) eles assumem necessariamente um mínimo ou um máximo no mesmo ponto.
c) eles diferem por uma constante.
d) suas concavidades são de mesmo sentido.
e) nenhuma das anteriores.
R. letra  "a letra d é f'acil de ser eliminada. hum 
a letra a também"
 
2)Dados três pontos no plano cartesiano, não colineares e com abscissas 
distintas duas a duas, o número de funções quadráticas que podem ser 
encontradas de maneira que esses pontos pertençam aos seus gráficos é:
a) 0  b) 1  c) 2  d) 3
R.letra .? 

  

Re: [obm-l] FW: PROBLEMAS..... de concurso??

2012-06-25 Por tôpico Bruno França dos Reis
Problema 1:

(a) Sejam P1 um trinômio de 2o. grau e P2 = kP1 (k real não nulo, k != 1)
são dois trinômios de 2o. grau distintos com as mesmas raízes ==> (a) é
falso

(b) Sejam P1(x) = (x-1)(x-2) e P2 = 2P1 (k real não nulo, k != 1), são dois
trinômios de 2o. grau com as mesmas raízes e extremos distintos (P1: mínimo
em (3/2, -1/4), P2: mínimo em (3/2, -1/2)).

(c) Sejam P1 e P2 = 2P1, P2 - P1 = P1 que não é constante

(d) Sejam P1 e P2 = -P1, concavidades opostas.

Portanto, NDA.


x^2 - 2x + 1

2x - 2 = 0
x = 1

--
Bruno FRANÇA DOS REIS

msn: brunoreis...@hotmail.com
skype: brunoreis666
tel: +55 11 9961-7732

http://brunoreis.com
http://brunoreis.com/tech (en)
http://brunoreis.com/blog (pt)

GPG Key: http://brunoreis.com/bruno-public.key

e^(pi*i)+1=0


2012/6/25 Bruno França dos Reis 

> Problema 2:
>
> Sejam P1(x1, y1), P2(x2, y2), P3(x3, y3).
>
> Hipóteses:
>  (1) P1, P2 e P3 são não-colineares
>  (2) xi != xj para i != j
>
> Queremos determinar o número de funções f tais que P1, P2, P3 \in {(x,
> f(x)); x \in R} da forma f(x) = ax^2 + bx + c.
>
> Sem perda de generalidade, podemos assumir x1 < x2 < x3, e, também, x1 =
> y1 = 0 (justificado através da possibilidade de analisar o problema em
> qualquer outro sistema de coordenadas que seja um deslocamento do original).
>
> Dessa forma, f(0) = 0 <==> c = 0. Logo, f é da forma f(x) = ax^2 + bx.
>
> Ora,
>  y2 = f(x2) = a(x2)^2 + b(x2)
>  y3 = f(x3) = a(x3)^2 + b(x3)
>
> Temos, então, um sistema linear em (a, b), cuja matriz de coeficientes é M
> = [[(x2)^2 (x2)]; [(x3)^2 (x3)]].
>
> Ora, det M = (x2)^2 * (x3) - (x3)^2 * (x2) = (x2)(x3)((x2) - (x3)). Pelas
> hipóteses, x2 != 0, x3 != 0 e x2 != x3, logo det M != 0, portanto existe
> solução e é única. Assim, existe uma, e apenas uma, parábola passando pelos
> 3 pontos em questão.
>
>
>
>
> --
> Bruno FRANÇA DOS REIS
>
> msn: brunoreis...@hotmail.com
> skype: brunoreis666
> tel: +55 11 9961-7732
>
> http://brunoreis.com
> http://brunoreis.com/tech (en)
> http://brunoreis.com/blog (pt)
>
> GPG Key: http://brunoreis.com/bruno-public.key
>
> e^(pi*i)+1=0
>
>
>
> 2012/6/25 Luís Lopes 
>
>>  Sauda,c~oes,
>>
>> Me mandaram os problemas abaixo com o gabarito.
>> Que tirei para ver as respostas justificadas de vocês,
>> sempre melhores e mais espertas do que as minhas.
>>
>> Faço isso por 3 razões:
>>
>> 1) para me ajudarem;
>> 2) para dar uma melhor resposta ao Fernando;
>> 3) para tirar a lista do silêncio e moviment'a-la
>> um pouco.
>>
>> [ ]'s
>> Lu'is
>>
>>
>>
>>
>> Prezado Luis,
>>
>>
>>
>> Gostaria de sua ajuda para as seguintes questões:
>>
>>
>>
>> 1)Se dois trinômios do 2º grau possuem as mesmas raízes então:
>> a) eles são necessariamente iguais.
>> b) eles assumem necessariamente um mínimo ou um máximo no mesmo ponto.
>> c) eles diferem por uma constante.
>> d) suas concavidades são de mesmo sentido.
>> e) nenhuma das anteriores.
>> R. letra  "a letra d é f'acil de ser eliminada. hum
>>
>> a letra a também"
>>
>>
>>
>> 2)Dados três pontos no plano cartesiano, não colineares e com abscissas
>> distintas duas a duas, o número de funções quadráticas que podem ser
>> encontradas de maneira que esses pontos pertençam aos seus gráficos é:
>>
>> a) 0  b) 1  c) 2  d) 3
>>
>> R.letra .?
>>
>>
>>
>


Re: [obm-l] FW: PROBLEMAS..... de concurso??

2012-06-25 Por tôpico Bruno França dos Reis
Problema 2:

Sejam P1(x1, y1), P2(x2, y2), P3(x3, y3).

Hipóteses:
 (1) P1, P2 e P3 são não-colineares
 (2) xi != xj para i != j

Queremos determinar o número de funções f tais que P1, P2, P3 \in {(x,
f(x)); x \in R} da forma f(x) = ax^2 + bx + c.

Sem perda de generalidade, podemos assumir x1 < x2 < x3, e, também, x1 = y1
= 0 (justificado através da possibilidade de analisar o problema em
qualquer outro sistema de coordenadas que seja um deslocamento do original).

Dessa forma, f(0) = 0 <==> c = 0. Logo, f é da forma f(x) = ax^2 + bx.

Ora,
 y2 = f(x2) = a(x2)^2 + b(x2)
 y3 = f(x3) = a(x3)^2 + b(x3)

Temos, então, um sistema linear em (a, b), cuja matriz de coeficientes é M
= [[(x2)^2 (x2)]; [(x3)^2 (x3)]].

Ora, det M = (x2)^2 * (x3) - (x3)^2 * (x2) = (x2)(x3)((x2) - (x3)). Pelas
hipóteses, x2 != 0, x3 != 0 e x2 != x3, logo det M != 0, portanto existe
solução e é única. Assim, existe uma, e apenas uma, parábola passando pelos
3 pontos em questão.




--
Bruno FRANÇA DOS REIS

msn: brunoreis...@hotmail.com
skype: brunoreis666
tel: +55 11 9961-7732

http://brunoreis.com
http://brunoreis.com/tech (en)
http://brunoreis.com/blog (pt)

GPG Key: http://brunoreis.com/bruno-public.key

e^(pi*i)+1=0


2012/6/25 Luís Lopes 

>  Sauda,c~oes,
>
> Me mandaram os problemas abaixo com o gabarito.
> Que tirei para ver as respostas justificadas de vocês,
> sempre melhores e mais espertas do que as minhas.
>
> Faço isso por 3 razões:
>
> 1) para me ajudarem;
> 2) para dar uma melhor resposta ao Fernando;
> 3) para tirar a lista do silêncio e moviment'a-la
> um pouco.
>
> [ ]'s
> Lu'is
>
>
>
>
> Prezado Luis,
>
>
>
> Gostaria de sua ajuda para as seguintes questões:
>
>
>
> 1)Se dois trinômios do 2º grau possuem as mesmas raízes então:
> a) eles são necessariamente iguais.
> b) eles assumem necessariamente um mínimo ou um máximo no mesmo ponto.
> c) eles diferem por uma constante.
> d) suas concavidades são de mesmo sentido.
> e) nenhuma das anteriores.
> R. letra  "a letra d é f'acil de ser eliminada. hum
>
> a letra a também"
>
>
>
> 2)Dados três pontos no plano cartesiano, não colineares e com abscissas
> distintas duas a duas, o número de funções quadráticas que podem ser
> encontradas de maneira que esses pontos pertençam aos seus gráficos é:
>
> a) 0  b) 1  c) 2  d) 3
>
> R.letra .?
>
>
>


Re: [obm-l] FW: Sistema de inteiros positivos

2012-03-11 Por tôpico terence thirteen
Isto dá uma série pra cada caso. Pensei que ele quisesse um caso geral.
Este sim é virtualmente impossível.

Em 11 de março de 2012 18:25, escreveu:

> **
>
> hum vamos tentar, representando os numeros da forma 3x com os expoentes
>
> da expressao x^3+x^6+x^9+x^12+...
>
> as de 2y com os expoentes de x^2+x^4+x^6+x^8+...
>
> e os de z com z+z^2+z^3+z^4...
>
> como nâo estamos interessados na convergencia da serie e sim nos resultados
>
> obtidos pelos expoentes da multiplicacao de
>
> (x^3+x^6+x^9+x^12+...)( x^2+x^4+x^6+x^8+...)(z+z^2+z^3+z^4...)  você
>
>  estaria interessado em de quantas formas poderíamos somar os expoentes  de
>
> maneira que sempre de 30,um bom exemplo seria (x^6)(x^4)(x^20)=x^30 sacou??
>
>  entao nossa resposta seria o coeficiente de x^30 nessa multiplicação,
> entao ficaria
>
>
>  
> [(x^3)/(1-x^3)][(x^2)/(1-x^2)][x/(1-x)]=(x^6)[(1-x^3)^-1][(1-x^2)^-1](1-x^-1)=(x^6)(x^24)
>
> logo o coeficiente de isso aqui [(1-x^3)^-1][(1-x^2)^-1](1-x^-1) que
> multiplica o x^24 dara
>
> nossa resposta escrevendo sob o termo geral de binomio de newton
>
> vamos fazer uma notacao de binomial assim o número binomial (4,2)=4!/2!.2!
>
> fica cada um assim binomial (-1,k).(-x^3)^k  (-1, t).(-x^2)^t
>  (-1,n).(-x^n), bom com isso estariamos interessados novamente
>
> só que em outra solucao a de 3k+2t+n=24 e que seria semelhante ao problema
> inicial só que 6 unidades menor ok.
>
> onde chegaríamos a 3a+2b+c=18, depois 3p+2r+s=12, entao 3w+2j+y=6 e nesta
>  última fica fácil (1,1,1) , (2,0,0), (0,3,0) , (0,0,6)
>
> e subistituindo voltando todas elas o resultado do problema teria um
> somatório de números binomiais que vou tentar depois ai tento concluir pra
> você
>
> no papel pq é muito ruim escrever aqui no pc direto!!
>
> Bom esta solucao que tentei foi no estilo funções geradoras!!!
>
> um grande abraço do  Douglas Oliveira.
>
>
>
> On Tue, 6 Mar 2012 16:33:39 -0300, João Maldonado wrote:
>
>  Existe alguma fórmula para se determinar a quantidade de soluções
> inteiras positivas para o sistema = K
> Ex: 3x + 2y + z = 30 (um sistema desses é fácil de se resolver, mas é
> possível generalizar  a fórmula para um somatório com n variáveis?)
> []'s
> João
>
>
>
>



-- 
/**/
神が祝福

Torres


Re: [obm-l] FW: Sistema de inteiros positivos

2012-03-11 Por tôpico douglas . oliveira
  

hum vamos tentar, representando os numeros da forma 3x com os
expoentes 

da expressao x^3+x^6+x^9+x^12+... 

as de 2y com os
expoentes de x^2+x^4+x^6+x^8+... 

e os de z com z+z^2+z^3+z^4... 

como
nâo estamos interessados na convergencia da serie e sim nos resultados


obtidos pelos expoentes da multiplicacao de 

(x^3+x^6+x^9+x^12+...)(
x^2+x^4+x^6+x^8+...)(z+z^2+z^3+z^4...) você 

 estaria interessado em de
quantas formas poderíamos somar os expoentes de 

maneira que sempre de
30,um bom exemplo seria (x^6)(x^4)(x^20)=x^30 sacou?? 

 entao nossa
resposta seria o coeficiente de x^30 nessa multiplicação, entao ficaria



[(x^3)/(1-x^3)][(x^2)/(1-x^2)][x/(1-x)]=(x^6)[(1-x^3)^-1][(1-x^2)^-1](1-x^-1)=(x^6)(x^24)


logo o coeficiente de isso aqui [(1-x^3)^-1][(1-x^2)^-1](1-x^-1) que
multiplica o x^24 dara 

nossa resposta escrevendo sob o termo geral de
binomio de newton 

vamos fazer uma notacao de binomial assim o número
binomial (4,2)=4!/2!.2! 

fica cada um assim binomial (-1,k).(-x^3)^k
(-1, t).(-x^2)^t (-1,n).(-x^n), bom com isso estariamos interessados
novamente  

só que em outra solucao a de 3k+2t+n=24 e que seria
semelhante ao problema inicial só que 6 unidades menor ok. 

onde
chegaríamos a 3a+2b+c=18, depois 3p+2r+s=12, entao 3w+2j+y=6 e nesta
última fica fácil (1,1,1) , (2,0,0), (0,3,0) , (0,0,6)  

e
subistituindo voltando todas elas o resultado do problema teria um
somatório de números binomiais que vou tentar depois ai tento concluir
pra você 

no papel pq é muito ruim escrever aqui no pc direto!! 

Bom
esta solucao que tentei foi no estilo funções geradoras!!! 

um grande
abraço do Douglas Oliveira. 

On Tue, 6 Mar 2012 16:33:39 -0300, João
Maldonado wrote: 

> Existe alguma fórmula para se determinar a
quantidade de soluções inteiras positivas para o sistema = K 
> Ex: 3x +
2y + z = 30 (um sistema desses é fácil de se resolver, mas é possível
generalizar a fórmula para um somatório com n variáveis?) 
> []'s 
>
João

  

Re: [obm-l] FW: Sistema de inteiros positivos

2012-03-11 Por tôpico terence thirteen
Acho difícil.

Se for algo muito geral, eu posso produzir coisas sem solução(que tal
2x+4y=1995, por exemplo?). Se for algo com pelo menos uma solução, parece
pior ainda: que tal saber quantas soluções tem
a+2b+3c+4d+5e+...+26z=(1+2+3+...+26)+2002^2? Pelo menos uma, ele tem, certo?

Este problema me lembra aquele problema de programação, que pede para
descobrir de quantas maneiras podemos trocar uma nota de 100 reais usando
notas e moedas do sistema brasileiro.

Não creio que exista uma solução fácil, mas por causa de um apelo à
autoridade: o problema se resumiria a uma fórmula que parece se modificar a
cada variável adicionada.


2012/3/6 João Maldonado 

>   Existe alguma fórmula para se determinar a quantidade de soluções
> inteiras positivas para o sistema = K
>
> Ex: 3x + 2y + z = 30 (um sistema desses é fácil de se resolver, mas é
> possível generalizar  a fórmula para um somatório com n variáveis?)
>
> []'s
> João
>



-- 
/**/
神が祝福

Torres


Re: [obm-l] FW: PROBLEMA DE LÓGICA

2011-12-09 Por tôpico Sergio Lima Netto
Caro Luís,

O mais fácil, penso eu, é procurar uma contradição nos
testemunhos, identificando um sub-conjunto de possíveis
mentirosos, e, a partir daí, encontrar o mentiroso
por tentativa-e-erro.

No caso, é simples perceber a contradição entre o que o
Rafael e o Leonardo disseram, já que um culpa o Chicão e
o outro culpa o Leonardo. Logo, o mentiroso
é o Rafael ou o Leonardo.

Isto confirma que os outros dois testemunhos eram verdadeiros,
e assim podemos afirmar, com certeza, que "não foi o Augusto"
e que o "Rafael não tem razão".

Agora, se o "Rafael não tem razão", então, o Leonardo
tem, e assim o caloteiro é o Chicão,
enquanto que o mentiroso é o Rafael.

Bem, no meu raciocínio, a resposta certa é a (B),
e, se aceitarem a brincadeira, há um segundo mentiroso:
o gabarito. De fato, seguindo o gabarito,
se o culpado fosse o Rafael, então Leonardo e Rafael
estariam mentindo.

Abraço,
sergio

On Fri, 9 Dec 2011 11:45:39 +, Luís Lopes wrote
> Sauda,c~oes,
> 
> O "Rafael" me pediu ajuda para tirá-lo da enrascada abaixo.
> 
> A resposta está certa? Qual a maneira rápida/elegante de 
> se resolver tais problemas?
> 
> Abraços, 
> Luís
> 
> =
> 
> Meu caro Luís,
> 
> Gostaria de sua ajuda para  a seguinte 
> questão:
> 
> EXERCÍCIO Nº 1: Quatro amigos vão ao rodeio em 
> Barretos e um
> deles entra sem pagar ingresso. Um fiscal do evento 
> quer saber
> quem foi o penetra: Eu não fui, diz o Augusto. Foi o 
> Chicão, diz o
> Leonardo. Foi o Leonardo, diz o Rafael. Rafael não 
> tem razão, diz
> o Chicão. Sabendo que só um deles mentiu. Quem não 
> pagou a 
> entrada?
> 
> a) Leonardo
> b) Chicão
> c) Rafael
> d) Augusto
> e) n.d.a
> Resposta: C.
> 
> 
> -- 
> This message has been scanned for viruses and
> dangerous content by MailScanner, and is
> believed to be clean.


Sergio Lima Netto
PEE-COPPE/DEL-Poli/UFRJ
POBox 68504, Rio de Janeiro, RJ
21941-972, BRAZIL
(+55 21) 2562-8164


-- 
This message has been scanned for viruses and
dangerous content by MailScanner, and is
believed to be clean.

=
Instru��es para entrar na lista, sair da lista e usar a lista em
http://www.mat.puc-rio.br/~obmlistas/obm-l.html
=


[obm-l] Re: [obm-l] FW: PROBLEMA DE LÓGICA

2011-12-09 Por tôpico João Antônio Justi
Como apenas um deles está mentindo e apenas um deles entrou sem pagar,
então ou Leonardo ou Rafael está mentindo, uma vez que suas afirmações se
contradizem. Logo, Augusto e Chicão estão falando a verdade. Como Chicão
está falando a verdade, então Rafael "não tem razão", ie, está mentindo. O
mentiroso, portanto, é Rafael, e Chicão entrou sem pagar, conforme disse
Leonardo. Como a questão pergunta quem não pagou, parece-me que a resposta
correta é (b), Chicão.

2011/12/9 Luís Lopes 

>  Sauda,c~oes,
>
> O "Rafael" me pediu ajuda para tirá-lo da enrascada abaixo.
>
> A resposta está certa? Qual a maneira rápida/elegante de
> se resolver tais problemas?
>
> Abraços,
> Luís
>
> =
> Meu caro Luís,
>
> Gostaria de sua ajuda para  a seguinte questão:
>
> * *
>
> EXERCÍCIO Nº 1: Quatro amigos vão ao rodeio em Barretos e um
>
> deles entra sem pagar ingresso. Um fiscal do evento quer saber
>
> quem foi o penetra: Eu não fui, diz o Augusto. Foi o Chicão, diz o
>
> Leonardo. Foi o Leonardo, diz o Rafael. Rafael não tem razão, diz
>
> o Chicão. Sabendo que só um deles mentiu. Quem não pagou a
>
> entrada?
>
> *
> *
>
> a) Leonardo
>
> b) Chicão
>
> c) Rafael
>
> d) Augusto
> e) n.d.a
> Resposta: C.
>
>
>


[obm-l] Re: [obm-l] FW: Múltiplos

2011-12-06 Por tôpico Pedro Cardoso
Oi, João.

Na decomposição por números primos, o número de "2" que aparece em n! é...

S = n/2 + n/4 + n/8 + ... + 1 = n * [ 1/2*(1 - 1/2^T)/(1-1/2) ]
onde T = número de termos da PG. Veja que n * (1/2)^T = 1.

Pense um pouco pra ver que isso vale pra qualquer primo (o +1 não, o +1 no
fim eu só posso colocar porque n é potência de 2 pras contas fecharem).

Daí,

S = n * [ 1/2*( 1 - 1/n) / (1/2) ] = n*(1-1/n) = n-1.

Assim, "2" aparece "n-1" vezes em n! e portanto 2^(n-1) divide n! (isso
tudo quando n é potência de 2, pro somatório ficar bonito).

2011/12/6 João Maldonado 

>
>
> --
> From: joao_maldona...@hotmail.com
> To: obm-l@mat.puc-rio.br
> Subject: Múltiplos
> Date: Tue, 6 Dec 2011 14:29:31 -0200
>
>
>
> Prove que se n é potênica de 2,  então n! é múltiplo de 2^(n-1)
>
> O jeito que eu fiz  é um pouquinho complicado,  consiste  principalmente
> em calcular o somatório (n-p).2^(p-1),  com p variando de 1 a n-1
> Deu certo mas  foi somente contas, existe algum jeito de pensar mais
> bonitinho   (até porque no contexto que vi esse problema não deve ser essa
> a solução mais fácil)?
>
> []'s
> João
>


[obm-l] RE: [obm-l] FW: [obm-l] Equações polinomiais

2011-09-25 Por tôpico marcone augusto araújo borges

Olá,João.Depois eu vi uma solução assim:
x^4 - 4x^2 + 4 = 5x^3 + 7x=>(x^2 - 2)^2 = 5x^3 + 7x
Se x<0,o segundo membro é negativo e o primeiro,positivo.
Então a equação não tem raízes negativas.
Abraço,
Marcone
 



From: joao_maldona...@hotmail.com
To: obm-l@mat.puc-rio.br
Subject: [obm-l] FW: [obm-l] Equações polinomiais
Date: Fri, 23 Sep 2011 19:47:08 -0300







Mais  uam vez acho que existe uma maneira  mais bonita de resolver






y=x^4 - 5x^3 - 4x^2 - 7x +  4 = 0


y' =   4x³  - 15x² -8x - 7
Se y' = 0  temos os pontos  de máximo e  mínimo momentaneo  de y


y''  =  12x²  -  30x  - 8
y'' = 0  temos os  pontos de máximo, mínimo de y'   
6x²-15x  -4=0  
x ~ 1/12 (15 - Sqrt[321])  e 1/12 (15 + Sqrt[321]) 




Analisando  y'  temos queao substituir [a primeira raiz,   y<0,  logo  
y' só  tem uma raiz


sendo  assim  y só tem um ponto mínimo,  que é positivo


Como 
x=3,  y<0
x=0, y>0


e x=10, y>0


temos  que todas as  2 raízes reais  são positivas


[]'s
João




From: marconeborge...@hotmail.com
To: obm-l@mat.puc-rio.br
Subject: [obm-l] Equações polinomiais 
Date: Fri, 23 Sep 2011 21:42:35 +




Quantas raízes negativas possui a equação x^4 - 5x^3 - 4x^2 - 7x +  4 = 0 ?
Se eu tivesse certeza de que as raízes são reais,tentaria as relaões de 
Girard...
  

[obm-l] Re: [obm-l] Re: [obm-l] FW: Combinatória

2011-07-22 Por tôpico Fernando A Candeias
Oi Ralph
substantivo femininoTemos em parte uma problema de semântica. No Houaiss
encontramos o verbete:  O que mostra que pode ser também utilizada para
indicar probabilidade, como no exemplo que dá em verde. No
primeiro significado ela indica simplesmente a existencia;. Como alguem que
perguntasse,. é possivel a ordem de chegada ABC ? E a respopsta seria sim ,
par 1q





"condição do que é possível, do que pode acontecer

Ex.: muita, pouca p."

 O que mostra
Em 20 de julho de 2011 12:19, Ralph Teixeira  escreveu:

> Eu gosto MUITO da solucao do Fernando, tambem acho que eh a mais elegante.
>
> Isto dito, vou ser chato muito muito chato: nada no problema sugere nem
> pede probabilidades. Entao vamos usar a otima ideia, mas mudar a linguagem:
>
> "Para cada possibilidade que tenha a ordem ABC, teremos possibilidades
> correspondentes com as ordens ACB, CAB, etc. Ou seja, o numero de
> classificacoes possiveis eh 6 vezes o numero de classificacoes pedido.
> Assim, existem 6!/6=120 classificacoes possiveis onde Felipe se torna
> campeao."
>
> E notem que isto NAO significa que a probabilidade de Felipe ser
> campeao eh 1/6, jah que ninguem sabe se as "possibilidades" sao
> realmente igualmente provaveis.
>
> (Alias, considerando o momento da Red Bull e do Felipe... :) :) :) :) )
>
> Abraco,
>   Ralph
>
> 2011/7/20 João Maldonado 
>
>>  Olá.
>>
>> Eu não vejo erro, aliás é até mais prática do que as soluções já
>> apresentradas.
>>
>> Só esclarecendo  para os outros, demorei um pouco para entender:
>>
>>
>> ABC  significa que A está a frente de B que está a frente de C  (não
>> importa de que jeito)
>> Ex:
>> ADBECF ou ABDEFC, DEFABC, etc
>>
>> --
>> Date: Tue, 19 Jul 2011 11:30:17 -0300
>> Subject: Combinatória
>> From: facande...@gmail.com
>> To: joao_maldona...@hotmail.com
>>
>> Prezador colega.
>>
>> Outra maneira seria a seguinte, usando sua convenção para o nome dos
>> concorrentes.
>> Os possíveis resultados seriam uma das seis permutações,   ABC, ACB, BAC,
>> BCA, CAB, CBA. todas equiprováveis. Como a ordem  favorável é  ABC,
>> e probabilidade de sua ocorrência será 1/6, que será também a de Felipe ser
>> campeão,  como você achou
>> Está certa essa linha de raciocínio?
>> Sds.
>> Fernando Candeias
>>
>>
>>
>>
>


[obm-l] Re: [obm-l] FW: Combinatória

2011-07-20 Por tôpico Ralph Teixeira
Eu gosto MUITO da solucao do Fernando, tambem acho que eh a mais elegante.

Isto dito, vou ser chato muito muito chato: nada no problema sugere nem pede
probabilidades. Entao vamos usar a otima ideia, mas mudar a linguagem:

"Para cada possibilidade que tenha a ordem ABC, teremos possibilidades
correspondentes com as ordens ACB, CAB, etc. Ou seja, o numero de
classificacoes possiveis eh 6 vezes o numero de classificacoes pedido.
Assim, existem 6!/6=120 classificacoes possiveis onde Felipe se torna
campeao."

E notem que isto NAO significa que a probabilidade de Felipe ser
campeao eh 1/6, jah que ninguem sabe se as "possibilidades" sao
realmente igualmente provaveis.

(Alias, considerando o momento da Red Bull e do Felipe... :) :) :) :) )

Abraco,
  Ralph

2011/7/20 João Maldonado 

>  Olá.
>
> Eu não vejo erro, aliás é até mais prática do que as soluções já
> apresentradas.
>
> Só esclarecendo  para os outros, demorei um pouco para entender:
>
>
> ABC  significa que A está a frente de B que está a frente de C  (não
> importa de que jeito)
> Ex:
> ADBECF ou ABDEFC, DEFABC, etc
>
> --
> Date: Tue, 19 Jul 2011 11:30:17 -0300
> Subject: Combinatória
> From: facande...@gmail.com
> To: joao_maldona...@hotmail.com
>
> Prezador colega.
>
> Outra maneira seria a seguinte, usando sua convenção para o nome dos
> concorrentes.
> Os possíveis resultados seriam uma das seis permutações,   ABC, ACB, BAC,
> BCA, CAB, CBA. todas equiprováveis. Como a ordem  favorável é  ABC,
> e probabilidade de sua ocorrência será 1/6, que será também a de Felipe ser
> campeão,  como você achou
> Está certa essa linha de raciocínio?
> Sds.
> Fernando Candeias
>
>
>
>


[obm-l] Res: [obm-l] Enc: Re: [obm-l] FW: [obm-l] Dúvida em Geometria

2011-06-15 Por tôpico Paulo Barclay Ribeiro
Oi Eduardo, desculpe a demora em responder.Ando com problemas de conexão.
Entendi bem asua  solução  eachei,também, muito original.
Agradeço muito a sua ajuda..

Um forte abraço

paulo




De: Eduardo Wilner 
Para: obm-l@mat.puc-rio.br
Enviadas: Sexta-feira, 10 de Junho de 2011 0:25:27
Assunto: [obm-l] Enc: Re: [obm-l] FW: [obm-l] Dúvida em Geometria


Estou repetindo a mensagem pois o que apareceu na lista está muito "deformado" 
em relação ao que eue enviei antes; os simbolos vetoriais devem estar em 
negrito, que talvez o "copilador" não aceite. Mas o determinante saiu todo 
desmontado... 


--- Em qui, 9/6/11, Eduardo Wilner  escreveu:


>De: Eduardo Wilner 
>Assunto: Re: [obm-l] FW: [obm-l] Dúvida em Geometria
>Para: obm-l@mat.puc-rio.br
>Data: Quinta-feira, 9 de Junho de 2011, 17:01
>
>
>É uma boa oportunidade de aplicar vetores; o produto escalar dos versores das 
>normais às faces fornece o oposto do cosseno do ângulo diedro por elas formado 
>(oposto porquê o ângulo entre elas é suplementar ao ângulo diedro).
>Considerando um sistema de coordenadas cartesianas com origem no centro da 
>base 
>da piramide , eixo Oz contendo a altura e Ox e Oy perpendiculares aos lados do 
>quadrado da base pertencentes às faces em pauta,(sejam AB e BC)  o produto 
>vetorial
>
>    | i    j     k |   
>(A-V) X (B - V ) = |4   -4   -2| = 16 i + 32 k = 16.sqrt(5) n1
>    |4    4   -2|
>
> onde n1 é o versor da normal à face BVA (V é o vertice superior).
>
>Uma rotação de 90º em torno de Oz fornece a normal à face BVC:
>
>                                          n2 =   ( 16 j +32 k ) /[16*sqrt(5)].
>
>Logo  o cosseno do ângulo diedro = - n1 . n2 = -(32*32)/(16*16*5) = - 4/5.
>
>[ ]'s
> 
>
>
>
>
>>
>>
>>
>>
>>

>>
>>
>>  

[obm-l] Enc: Re: [obm-l] FW: [obm-l] Dúvida em Geometria

2011-06-09 Por tôpico Eduardo Wilner
Estou repetindo a mensagem pois o que apareceu na lista está muito 
"deformado" em relação ao que eue enviei antes; os simbolos vetoriais 
devem estar em negrito, que talvez o "copilador" não aceite. Mas o 
determinante saiu todo desmontado... 

--- Em qui, 9/6/11, Eduardo Wilner  escreveu:

De: Eduardo Wilner 
Assunto: Re: [obm-l] FW: [obm-l] Dúvida em Geometria
Para: obm-l@mat.puc-rio.br
Data: Quinta-feira, 9 de Junho de 2011, 17:01

É uma boa oportunidade de aplicar vetores; o produto escalar dos versores das 
normais às faces fornece o oposto do cosseno do ângulo diedro por elas formado 
(oposto porquê o ângulo entre elas é suplementar ao ângulo diedro).
Considerando um sistema de coordenadas cartesianas com origem no centro da base 
da piramide , eixo Oz contendo a altura e Ox e Oy perpendiculares aos lados do 
quadrado da base pertencentes às faces em pauta,(sejam AB e BC)  o produto 
vetorial

    | i    j     k |   
(A-V) X (B - V ) = |4   -4   -2| = 16 i + 32 k = 16.sqrt(5) n1
    |4    4   -2|

 onde n1 é o versor da normal à face BVA (V é o vertice superior).

Uma rotação de 90º em torno de Oz fornece a normal à face BVC:

                                          n2 =   ( 16 j +32 k ) /[16*sqrt(5)].

Logo  o cosseno do ângulo diedro = - n1 . n2 = -(32*32)/(16*16*5) = - 4/5.

[ ]'s
 







  


[obm-l] Re: [obm-l] FW: [obm-l] Dúvida em Geometria

2011-06-09 Por tôpico Eduardo Wilner
É uma boa oportunidade de aplicar vetores; o produto escalar dos versores das 
normais às faces fornece o oposto do cosseno do ângulo diedro por elas formado 
(oposto porquê o ângulo entre elas é suplementar ao ângulo diedro).
Considerando um sistema de coordenadas cartesianas com origem no centro da base 
da piramide , eixo Oz contendo a altura e Ox e Oy perpendiculares aos lados do 
quadrado da base pertencentes às faces em pauta,(sejam AB e BC)  o produto 
vetorial

                                            | i     j     k |    
   (A-V) X (B-V) =  | 4   -4    -2| = 16 i +32 k = 16*sqrt(5) n1
    | 4    4    -2|

 onde n1 é o versor da normal à face BVA.

Uma rotação de 90º em torno de Oz fornece a normal à face BVC:

                                          n2 =   ( 16 j +32 k ) /[16*sqrt(5)].

Logo  o cosseno do ângulo diedro = - n1 . n2 = -(32*32)/(16*16*5) = - 4/5.

[ ]'s
 


--- Em dom, 29/5/11, João Maldonado  escreveu:

De: João Maldonado 
Assunto: [obm-l] FW: [obm-l] Dúvida em Geometria
Para: obm-l@mat.puc-rio.br
Data: Domingo, 29 de Maio de 2011, 17:30





 


From: joao_maldona...@hotmail.com
To: obm-l@mat.puc-rio.br
Subject: RE: [obm-l] Dúvida em Geometria
Date: Sun, 29 May 2011 01:19:37 -0300



 
 

#yiv285393420 .yiv285393420ExternalClass .yiv285393420ecxhmmessage P
{padding:0px;}
#yiv285393420 .yiv285393420ExternalClass body.yiv285393420ecxhmmessage
{font-size:10pt;font-family:Tahoma;}




Bom, sou  estudante de ensino medio, logo minha resposta pode estar  errada :D
Fazendo z1, z2, z3, z4 como os vértices da base e z5 como o  vértice da 
pirâmide/O como o centro da base, 
O cosseno do ângulo da base é fácil calcular, já que Oz1 = 4sqrt(2) e Oz5 = 2, 
z1z5 = 6, logo cos(

[obm-l] Re: [obm-l] Re: [obm-l] Re: [obm-l] Re: [obm-l] FW: Teoria dos números

2011-05-30 Por tôpico Johann Dirichlet
O que eu posso fazer se eu sou lento e preguiçoso no computador?

Ah, deixa eu ficar ninja no dvorak... huahuahuahua!

Em 27/05/11, Rogerio Ponce escreveu:
> Pois e', Dirichlet, o Ralph tem este pessimo habito...
> :)
>
> []'s
> Rogerio Ponce
>
> Em 27 de maio de 2011 17:39, Ralph Teixeira  escreveu:
>
>> Yeah! Ninjei de novo! :) :) :) ;)
>> 2011/5/27 Johann Dirichlet 
>>
>>> Poxa! O Ralph destruiu minha mensagem! Mas acabei respondendo do mesmo
>>> jeito (ou nao!:))
>>>
>>> Em 27/05/11, Johann Dirichlet escreveu:
>>> > Ce já estudou congruencias? Um bom começo é pegar a Eureka! 2 na
>>> > página da OBM, www.obm.org.br (ou comprar da OBM! É baratinho, uma
>>> > anuidade de uns 30 reais e uns 4 contos por cada atrasado que quiser).
>>> > Anyway, vou tentar deixar fácil...
>>> >
>>> > 1)
>>> > 2^n=(x-1)(x^2+x+1)
>>> >
>>> > Vamos tentar calcular o MDC:
>>> > d|x-1
>>> > d|x^2+x+1
>>> >
>>> > x =1 (mod d)
>>> > x^2+x+1=0 (mod d)
>>> >
>>> > primeira na segunda, d|3. Como d=3 é impossível (potencias de 2 nao
>>> > tem fatores 3 :) ), d=1.
>>> >
>>> > Em especial, x-1=1 ou x^2+x+1=1 (ambos sao potencias de 2, e o MDC é
>>> > 1, logo um deles é 1).
>>> > Ou seja, x=2 ou 0. Substitui e chora!
>>> >
>>> > 2)
>>> > 7|4n^2-3
>>> > Multiplica por 2
>>> > 7|8n^2-6=n^2+1+(7n^2-7)
>>> >
>>> > 7|n^2+1
>>> > Por congruências, é possível provar que basta testar n de 0 a 6.Mas
>>> > vou usar descenso infinito.
>>> >
>>> > Teste de 0 a 6 (larga a mão de ser preguiçoso!). Vai falhar (eu acho :)
>>> ).
>>> >
>>> > Se funcionar para algum cara maior que 6, seja F o menor dos caras
>>> > para os quais funciona (se existe, existe o menor, este é o lema da
>>> > boa ordem).
>>> >
>>> > Seja J=F-7. Então J é maior ou igual a 0.
>>> > 7|(J+7)^2+1=J^2+2*7*J+7^2+1=7*(um termo chato que não interessa)+J^2+1
>>> > 7|J^2+1
>>> >
>>> > Mas epa! Achei um cara (J) menor que o menor(F)!
>>> > E este é um absurdo, que surgiu quando eu disse que funcionava para
>>> > algum cara maior que 6!
>>> > Então, só faltaria testar para caras menores que 7. Você já testou,
>>> > então sabe que não funciona!
>>> >
>>> > É isso.
>>> >
>>> > P.S.: otruque de multiplicar por 2 facilita a vida pacas, mas não
>>> > precisava aplica-lo: a ideia do descenso infinito ainda daria conta.
>>> >
>>> >
>>> > Em 27/05/11, marcone augusto araújo
>>> > borges escreveu:
>>> >>
>>> >>
>>> >>
>>> >>
>>> >>
>>> >> From: marconeborge...@hotmail.com
>>> >> To: obm-l@mat.puc-rio.br
>>> >> Subject: Teoria dos números
>>> >> Date: Fri, 27 May 2011 12:28:34 +
>>> >>
>>> >>
>>> >>
>>> >>
>>> >>  1) Mostrar que para nenhum número natural n ,( 2^n)+1 nunca é
>>> um
>>> >> cubo.
>>> >>
>>> >>  Pensei:2^n=x^3-1=(x-1)(x^2+x+1).Se eu conseguisse mostrar q
>>> >> mdc((x-1,x^2+x+1)=1 e que x-1 e
>>> >>x^2+x+1 não podem ser cubos ao mesmo tempo,acredito q
>>> >> resolveria
>>> a
>>> >> questão.
>>> >>  Tentei outras formas também ,mas não consegui.
>>> >>
>>> >>  2) Provar q não exiiste número natural n tal q 7 divide
>>> 4n^2-3.
>>> >>
>>> >>   Considerei n= 7k+ 1 ou 7k-1 ou 7k+2 ou 7k-2 ou 7k+3 ou 7k-3
>>> >> e
>>> >> verifiquei q 4n^2-3 não é múltiplo de 7.
>>> >>   Sei q há outras formas(e talvez mais interessantes).
>>> >>
>>> >
>>> >
>>> > --
>>> > /**/
>>> > 神が祝福
>>> >
>>> > Torres
>>> >
>>>
>>>
>>> --
>>> /**/
>>> 神が祝福
>>>
>>> Torres
>>>
>>> =
>>> Instru�ões para entrar na lista, sair da lista e usar a lista em
>>> http://www.mat.puc-rio.br/~obmlistas/obm-l.html
>>> =
>>>
>>
>>
>


-- 
/**/
神が祝福

Torres

=
Instru��es para entrar na lista, sair da lista e usar a lista em
http://www.mat.puc-rio.br/~obmlistas/obm-l.html
=


[obm-l] Re: [obm-l] Re: [obm-l] Re: [obm-l] FW: Teoria dos números

2011-05-27 Por tôpico Rogerio Ponce
Pois e', Dirichlet, o Ralph tem este pessimo habito...
:)

[]'s
Rogerio Ponce

Em 27 de maio de 2011 17:39, Ralph Teixeira  escreveu:

> Yeah! Ninjei de novo! :) :) :) ;)
> 2011/5/27 Johann Dirichlet 
>
>> Poxa! O Ralph destruiu minha mensagem! Mas acabei respondendo do mesmo
>> jeito (ou nao!:))
>>
>> Em 27/05/11, Johann Dirichlet escreveu:
>> > Ce já estudou congruencias? Um bom começo é pegar a Eureka! 2 na
>> > página da OBM, www.obm.org.br (ou comprar da OBM! É baratinho, uma
>> > anuidade de uns 30 reais e uns 4 contos por cada atrasado que quiser).
>> > Anyway, vou tentar deixar fácil...
>> >
>> > 1)
>> > 2^n=(x-1)(x^2+x+1)
>> >
>> > Vamos tentar calcular o MDC:
>> > d|x-1
>> > d|x^2+x+1
>> >
>> > x =1 (mod d)
>> > x^2+x+1=0 (mod d)
>> >
>> > primeira na segunda, d|3. Como d=3 é impossível (potencias de 2 nao
>> > tem fatores 3 :) ), d=1.
>> >
>> > Em especial, x-1=1 ou x^2+x+1=1 (ambos sao potencias de 2, e o MDC é
>> > 1, logo um deles é 1).
>> > Ou seja, x=2 ou 0. Substitui e chora!
>> >
>> > 2)
>> > 7|4n^2-3
>> > Multiplica por 2
>> > 7|8n^2-6=n^2+1+(7n^2-7)
>> >
>> > 7|n^2+1
>> > Por congruências, é possível provar que basta testar n de 0 a 6.Mas
>> > vou usar descenso infinito.
>> >
>> > Teste de 0 a 6 (larga a mão de ser preguiçoso!). Vai falhar (eu acho :)
>> ).
>> >
>> > Se funcionar para algum cara maior que 6, seja F o menor dos caras
>> > para os quais funciona (se existe, existe o menor, este é o lema da
>> > boa ordem).
>> >
>> > Seja J=F-7. Então J é maior ou igual a 0.
>> > 7|(J+7)^2+1=J^2+2*7*J+7^2+1=7*(um termo chato que não interessa)+J^2+1
>> > 7|J^2+1
>> >
>> > Mas epa! Achei um cara (J) menor que o menor(F)!
>> > E este é um absurdo, que surgiu quando eu disse que funcionava para
>> > algum cara maior que 6!
>> > Então, só faltaria testar para caras menores que 7. Você já testou,
>> > então sabe que não funciona!
>> >
>> > É isso.
>> >
>> > P.S.: otruque de multiplicar por 2 facilita a vida pacas, mas não
>> > precisava aplica-lo: a ideia do descenso infinito ainda daria conta.
>> >
>> >
>> > Em 27/05/11, marcone augusto araújo
>> > borges escreveu:
>> >>
>> >>
>> >>
>> >>
>> >>
>> >> From: marconeborge...@hotmail.com
>> >> To: obm-l@mat.puc-rio.br
>> >> Subject: Teoria dos números
>> >> Date: Fri, 27 May 2011 12:28:34 +
>> >>
>> >>
>> >>
>> >>
>> >>  1) Mostrar que para nenhum número natural n ,( 2^n)+1 nunca é
>> um
>> >> cubo.
>> >>
>> >>  Pensei:2^n=x^3-1=(x-1)(x^2+x+1).Se eu conseguisse mostrar q
>> >> mdc((x-1,x^2+x+1)=1 e que x-1 e
>> >>x^2+x+1 não podem ser cubos ao mesmo tempo,acredito q resolveria
>> a
>> >> questão.
>> >>  Tentei outras formas também ,mas não consegui.
>> >>
>> >>  2) Provar q não exiiste número natural n tal q 7 divide
>> 4n^2-3.
>> >>
>> >>   Considerei n= 7k+ 1 ou 7k-1 ou 7k+2 ou 7k-2 ou 7k+3 ou 7k-3 e
>> >> verifiquei q 4n^2-3 não é múltiplo de 7.
>> >>   Sei q há outras formas(e talvez mais interessantes).
>> >>
>> >
>> >
>> > --
>> > /**/
>> > 神が祝福
>> >
>> > Torres
>> >
>>
>>
>> --
>> /**/
>> 神が祝福
>>
>> Torres
>>
>> =
>> Instru�ões para entrar na lista, sair da lista e usar a lista em
>> http://www.mat.puc-rio.br/~obmlistas/obm-l.html
>> =
>>
>
>


[obm-l] Re: [obm-l] Re: [obm-l] FW: Teoria dos números

2011-05-27 Por tôpico Ralph Teixeira
Yeah! Ninjei de novo! :) :) :) ;)
2011/5/27 Johann Dirichlet 

> Poxa! O Ralph destruiu minha mensagem! Mas acabei respondendo do mesmo
> jeito (ou nao!:))
>
> Em 27/05/11, Johann Dirichlet escreveu:
> > Ce já estudou congruencias? Um bom começo é pegar a Eureka! 2 na
> > página da OBM, www.obm.org.br (ou comprar da OBM! É baratinho, uma
> > anuidade de uns 30 reais e uns 4 contos por cada atrasado que quiser).
> > Anyway, vou tentar deixar fácil...
> >
> > 1)
> > 2^n=(x-1)(x^2+x+1)
> >
> > Vamos tentar calcular o MDC:
> > d|x-1
> > d|x^2+x+1
> >
> > x =1 (mod d)
> > x^2+x+1=0 (mod d)
> >
> > primeira na segunda, d|3. Como d=3 é impossível (potencias de 2 nao
> > tem fatores 3 :) ), d=1.
> >
> > Em especial, x-1=1 ou x^2+x+1=1 (ambos sao potencias de 2, e o MDC é
> > 1, logo um deles é 1).
> > Ou seja, x=2 ou 0. Substitui e chora!
> >
> > 2)
> > 7|4n^2-3
> > Multiplica por 2
> > 7|8n^2-6=n^2+1+(7n^2-7)
> >
> > 7|n^2+1
> > Por congruências, é possível provar que basta testar n de 0 a 6.Mas
> > vou usar descenso infinito.
> >
> > Teste de 0 a 6 (larga a mão de ser preguiçoso!). Vai falhar (eu acho :)
> ).
> >
> > Se funcionar para algum cara maior que 6, seja F o menor dos caras
> > para os quais funciona (se existe, existe o menor, este é o lema da
> > boa ordem).
> >
> > Seja J=F-7. Então J é maior ou igual a 0.
> > 7|(J+7)^2+1=J^2+2*7*J+7^2+1=7*(um termo chato que não interessa)+J^2+1
> > 7|J^2+1
> >
> > Mas epa! Achei um cara (J) menor que o menor(F)!
> > E este é um absurdo, que surgiu quando eu disse que funcionava para
> > algum cara maior que 6!
> > Então, só faltaria testar para caras menores que 7. Você já testou,
> > então sabe que não funciona!
> >
> > É isso.
> >
> > P.S.: otruque de multiplicar por 2 facilita a vida pacas, mas não
> > precisava aplica-lo: a ideia do descenso infinito ainda daria conta.
> >
> >
> > Em 27/05/11, marcone augusto araújo
> > borges escreveu:
> >>
> >>
> >>
> >>
> >>
> >> From: marconeborge...@hotmail.com
> >> To: obm-l@mat.puc-rio.br
> >> Subject: Teoria dos números
> >> Date: Fri, 27 May 2011 12:28:34 +
> >>
> >>
> >>
> >>
> >>  1) Mostrar que para nenhum número natural n ,( 2^n)+1 nunca é
> um
> >> cubo.
> >>
> >>  Pensei:2^n=x^3-1=(x-1)(x^2+x+1).Se eu conseguisse mostrar q
> >> mdc((x-1,x^2+x+1)=1 e que x-1 e
> >>x^2+x+1 não podem ser cubos ao mesmo tempo,acredito q resolveria
> a
> >> questão.
> >>  Tentei outras formas também ,mas não consegui.
> >>
> >>  2) Provar q não exiiste número natural n tal q 7 divide 4n^2-3.
> >>
> >>   Considerei n= 7k+ 1 ou 7k-1 ou 7k+2 ou 7k-2 ou 7k+3 ou 7k-3 e
> >> verifiquei q 4n^2-3 não é múltiplo de 7.
> >>   Sei q há outras formas(e talvez mais interessantes).
> >>
> >
> >
> > --
> > /**/
> > 神が祝福
> >
> > Torres
> >
>
>
> --
> /**/
> 神が祝福
>
> Torres
>
> =
> Instru�ões para entrar na lista, sair da lista e usar a lista em
> http://www.mat.puc-rio.br/~obmlistas/obm-l.html
> =
>


[obm-l] Re: [obm-l] FW: Teoria dos números

2011-05-27 Por tôpico Johann Dirichlet
Poxa! O Ralph destruiu minha mensagem! Mas acabei respondendo do mesmo
jeito (ou nao!:))

Em 27/05/11, Johann Dirichlet escreveu:
> Ce já estudou congruencias? Um bom começo é pegar a Eureka! 2 na
> página da OBM, www.obm.org.br (ou comprar da OBM! É baratinho, uma
> anuidade de uns 30 reais e uns 4 contos por cada atrasado que quiser).
> Anyway, vou tentar deixar fácil...
>
> 1)
> 2^n=(x-1)(x^2+x+1)
>
> Vamos tentar calcular o MDC:
> d|x-1
> d|x^2+x+1
>
> x =1 (mod d)
> x^2+x+1=0 (mod d)
>
> primeira na segunda, d|3. Como d=3 é impossível (potencias de 2 nao
> tem fatores 3 :) ), d=1.
>
> Em especial, x-1=1 ou x^2+x+1=1 (ambos sao potencias de 2, e o MDC é
> 1, logo um deles é 1).
> Ou seja, x=2 ou 0. Substitui e chora!
>
> 2)
> 7|4n^2-3
> Multiplica por 2
> 7|8n^2-6=n^2+1+(7n^2-7)
>
> 7|n^2+1
> Por congruências, é possível provar que basta testar n de 0 a 6.Mas
> vou usar descenso infinito.
>
> Teste de 0 a 6 (larga a mão de ser preguiçoso!). Vai falhar (eu acho :) ).
>
> Se funcionar para algum cara maior que 6, seja F o menor dos caras
> para os quais funciona (se existe, existe o menor, este é o lema da
> boa ordem).
>
> Seja J=F-7. Então J é maior ou igual a 0.
> 7|(J+7)^2+1=J^2+2*7*J+7^2+1=7*(um termo chato que não interessa)+J^2+1
> 7|J^2+1
>
> Mas epa! Achei um cara (J) menor que o menor(F)!
> E este é um absurdo, que surgiu quando eu disse que funcionava para
> algum cara maior que 6!
> Então, só faltaria testar para caras menores que 7. Você já testou,
> então sabe que não funciona!
>
> É isso.
>
> P.S.: otruque de multiplicar por 2 facilita a vida pacas, mas não
> precisava aplica-lo: a ideia do descenso infinito ainda daria conta.
>
>
> Em 27/05/11, marcone augusto araújo
> borges escreveu:
>>
>>
>>
>>
>>
>> From: marconeborge...@hotmail.com
>> To: obm-l@mat.puc-rio.br
>> Subject: Teoria dos números
>> Date: Fri, 27 May 2011 12:28:34 +
>>
>>
>>
>>
>>  1) Mostrar que para nenhum número natural n ,( 2^n)+1 nunca é um
>> cubo.
>>
>>  Pensei:2^n=x^3-1=(x-1)(x^2+x+1).Se eu conseguisse mostrar q
>> mdc((x-1,x^2+x+1)=1 e que x-1 e
>>x^2+x+1 não podem ser cubos ao mesmo tempo,acredito q resolveria a
>> questão.
>>  Tentei outras formas também ,mas não consegui.
>>
>>  2) Provar q não exiiste número natural n tal q 7 divide 4n^2-3.
>>
>>   Considerei n= 7k+ 1 ou 7k-1 ou 7k+2 ou 7k-2 ou 7k+3 ou 7k-3 e
>> verifiquei q 4n^2-3 não é múltiplo de 7.
>>   Sei q há outras formas(e talvez mais interessantes).
>>  
>
>
> --
> /**/
> 神が祝福
>
> Torres
>


-- 
/**/
神が祝福

Torres

=
Instru��es para entrar na lista, sair da lista e usar a lista em
http://www.mat.puc-rio.br/~obmlistas/obm-l.html
=


[obm-l] Re: [obm-l] FW: Teoria dos números

2011-05-27 Por tôpico Johann Dirichlet
Ce já estudou congruencias? Um bom começo é pegar a Eureka! 2 na
página da OBM, www.obm.org.br (ou comprar da OBM! É baratinho, uma
anuidade de uns 30 reais e uns 4 contos por cada atrasado que quiser).
Anyway, vou tentar deixar fácil...

1)
2^n=(x-1)(x^2+x+1)

Vamos tentar calcular o MDC:
d|x-1
d|x^2+x+1

x =1 (mod d)
x^2+x+1=0 (mod d)

primeira na segunda, d|3. Como d=3 é impossível (potencias de 2 nao
tem fatores 3 :) ), d=1.

Em especial, x-1=1 ou x^2+x+1=1 (ambos sao potencias de 2, e o MDC é
1, logo um deles é 1).
Ou seja, x=2 ou 0. Substitui e chora!

2)
7|4n^2-3
Multiplica por 2
7|8n^2-6=n^2+1+(7n^2-7)

7|n^2+1
Por congruências, é possível provar que basta testar n de 0 a 6.Mas
vou usar descenso infinito.

Teste de 0 a 6 (larga a mão de ser preguiçoso!). Vai falhar (eu acho :) ).

Se funcionar para algum cara maior que 6, seja F o menor dos caras
para os quais funciona (se existe, existe o menor, este é o lema da
boa ordem).

Seja J=F-7. Então J é maior ou igual a 0.
7|(J+7)^2+1=J^2+2*7*J+7^2+1=7*(um termo chato que não interessa)+J^2+1
7|J^2+1

Mas epa! Achei um cara (J) menor que o menor(F)!
E este é um absurdo, que surgiu quando eu disse que funcionava para
algum cara maior que 6!
Então, só faltaria testar para caras menores que 7. Você já testou,
então sabe que não funciona!

É isso.

P.S.: otruque de multiplicar por 2 facilita a vida pacas, mas não
precisava aplica-lo: a ideia do descenso infinito ainda daria conta.


Em 27/05/11, marcone augusto araújo
borges escreveu:
>
>
>
>
>
> From: marconeborge...@hotmail.com
> To: obm-l@mat.puc-rio.br
> Subject: Teoria dos números
> Date: Fri, 27 May 2011 12:28:34 +
>
>
>
>
>  1) Mostrar que para nenhum número natural n ,( 2^n)+1 nunca é um
> cubo.
>
>  Pensei:2^n=x^3-1=(x-1)(x^2+x+1).Se eu conseguisse mostrar q
> mdc((x-1,x^2+x+1)=1 e que x-1 e
>x^2+x+1 não podem ser cubos ao mesmo tempo,acredito q resolveria a
> questão.
>  Tentei outras formas também ,mas não consegui.
>
>  2) Provar q não exiiste número natural n tal q 7 divide 4n^2-3.
>
>   Considerei n= 7k+ 1 ou 7k-1 ou 7k+2 ou 7k-2 ou 7k+3 ou 7k-3 e
> verifiquei q 4n^2-3 não é múltiplo de 7.
>   Sei q há outras formas(e talvez mais interessantes).
>   


-- 
/**/
神が祝福

Torres

=
Instru��es para entrar na lista, sair da lista e usar a lista em
http://www.mat.puc-rio.br/~obmlistas/obm-l.html
=


[obm-l] Re: [obm-l] FW: Teoria dos números

2011-05-27 Por tôpico Ralph Teixeira
1) Começou bem. Mas você não quer mostrar que o m.d.c é 1, ou que eles são
(ou não) cubos. Você quer mostrar que eles não podem ser POTÊNCIAS DE 2 ao
mesmo tempo (você já cuidou do "cubo" na fatoração...). Neste
caso, pensando na paridade de x :)

2) Sim, o seu jeito é possivelmente o mais direto e simples. Para juntar
tudo num caso só, escreva assim: n=7k+r onde r é um dos números
{+-3,+-2,+-1,0}. Então 4n^2-3=4(49k^2+14k)+4r^2-3 deixa o mesmo resto que
r^2-3 na divisão por 7. Agora é só ver que se r={+-3,+-2,+-1,0} então
r^2-3={6,1,-2,-3}, e nenhum destes é divisível por 7.

Ou, melhor ainda, na linguagem das congruências:
n=r (mod 7) (onde r=+-3,+-2,+-1 ou 0)
4n^2-3=4r^2-3=6,1,2 ou -3 (mod 7), que nunca é 0.
---///---

Em outras palavras... Vamos resolver:
4n^2-3=0 (mod 7)
4n^2=3 (mod 7)
Dividindo por 4, isto é, multiplicando por 2 (que é o inverso de 4 (mod 7))
n^2=6 (mod 7)

Os "resíduos quadráticos" mod 7 são {0^2=0,1^1=1,2^2=4,3^2=2}. Então 6 não é
resíduo quadrático, isto é, n^2=6 não tem solução (módulo 7).

(Note: aqui fizemos listando explicitamente todos os resíduos quadráticos
mod 7; em geral, há maneiras mais bacanas de achar se um número é resíduo
quadrático mod p ou não... Se você estiver curioso, pode começar olhando
aqui: http://en.wikipedia.org/wiki/Quadratic_residue)

Abraço,
  Ralph
2011/5/27 marcone augusto araújo borges 

>
>
> --
> From: marconeborge...@hotmail.com
> To: obm-l@mat.puc-rio.br
> Subject: Teoria dos números
> Date: Fri, 27 May 2011 12:28:34 +
>
>  1) Mostrar que para nenhum número natural n ,( 2^n)+1 nunca é um
> cubo.
>
>  Pensei:2^n=x^3-1=(x-1)(x^2+x+1).Se eu conseguisse mostrar q
> mdc((x-1,x^2+x+1)=1 e que x-1 e
>x^2+x+1 não podem ser cubos ao mesmo tempo,acredito q resolveria a
> questão.
>  Tentei outras formas também ,mas não consegui.
>
>  2) Provar q não exiiste número natural n tal q 7 divide 4n^2-3.
>
>   Considerei n= 7k+ 1 ou 7k-1 ou 7k+2 ou 7k-2 ou 7k+3 ou 7k-3 e
> verifiquei q 4n^2-3 não é múltiplo de 7.
>   Sei q há outras formas(e talvez mais interessantes).
>


[obm-l] RE: [obm-l] FW: Pontuação mínima em campeonato

2011-05-25 Por tôpico Paulo Santa Rita

Olá Marcone e demais colegas desta lista ... OBM-L,
Muito complicado porque não há resposta para o seu problema e a formulação está 
errada, pois :
1) Existir algum critério de desempate é necessário2) Esta pontuação mínima 
depende do "ponto" onde está o campeonato.
Explico.
Para facilitar a sua visualização, considere 8 clubes que disputam 2 vagas em 
um campeonato de1 turno. Pode ocorrer que no final do campeonato 3 deles tenham 
tido um único empate e vencidoos demais jogos. Sejam E1, E2 e E3 estas equipes. 
Pode ocorrer o seguinte :
E1 empatou com E2E2 empatou com E3E3 empatou com E1
Cada uma dessas equipes tem exatamente 4 pontos perdidos. Todas as demais 
equipes tem, aomenos, 9 pontos perdidos ( pois, ao menos, perderam para E1, E2 
e E3 ). Logo, E1,  E2  e  E3terminam empatados em primeiro lugar. Como não há 
critério de desempate, como escolherquais serão as duas equipes que terão 
direito as duas vagas ?
Isso mostra que um critério de desempate é necessário.
Suponha agora que os 8 clubes disputam 3 vagas e que há um critério de 
desempate. ANTESDO CAMPEONATO COMEÇAR é correto dizer que o número maximo de 
pontos que podemser perdidos para assegurar uma das vagas é 4, isto é, 5 ou 
mais pontos perdidos NÃO ASSEGURAuma das vagas ( verifique )
Entretanto, se ao final da primeira rodada todos os jogos terminarem empatados, 
os numerosacima mudam, ou seja, a quantidade de pontos que voce procura DEPENDE 
DA RODADA, eleé uma função do ponto em que está o campeonato.
PROBLEMA1) . Seja N o número de clubes e P o número de vagas, P < [N/2], em um 
campeonato de turno único. Mostre que se um clube, ao final do campeonato, 
tiver D derrotas e E empates, entãoo número máximo de outros clubes que podem 
ter uma pontuação maior ou igual a dele é 2*D + E
OBS1 : [N/2] -> função pisoOBS1 : Admita que há critérios de desempate
PROBLEMA 2) Generalize o problema acima para o caso de campeonatos com mais de 
um turno.
Um AbraçoPSR,42505110940
From: marconeborge...@hotmail.com
To: obm-l@mat.puc-rio.br
Subject: [obm-l] FW: Pontuação mínima em campeonato
Date: Wed, 25 May 2011 11:58:21 +









 



From: marconeborge...@hotmail.com
To: obm-l@mat.puc-rio.br
Subject: Pontuação mínima em campeonato 
Date: Wed, 25 May 2011 10:42:02 +




Como calcular o mínimo de pontos para uma equipe estar livre do rebaixamento 
(independente de qualquer critério de desempate)em um campeonato de 20 times em 
que os quatro últimos colocados são rebaixados?Cada time enfrenta seus 19 
adversários,jogando 2 vezes com cada um deles e a vitória vale 3 pontos,o 
empate vale 1 ponto e a derrota,zero.
É muito complicado?
 
  

[obm-l] RE: [obm-l] FW: Equação polinomial(ajuda)

2011-04-23 Por tôpico João Maldonado

Note que sen(5a) = 1/2 não tem só uma soluçãosen 30 = 1/2, sen 150 = 1/2, sen 
390 = 1/2, etc.sen 6 não é a única  solução
 Temos x = 30 + 360k ou x = 150 + 360k
Dividindo por 5 temosx = 6 + 72k, daonde vem as soluções 6, 78, 222, e 294 
(note que x = 150 implicaria  senx = 1/2)
x = 30 +72k, daonde vem as soluções 102, 174, 246 e 318 ( note que x = 30  
implicaria senx =   1/2)
Mas sen(102) = sen(78sen(174) = sen(6)sen(246) = sen(294)sen(318) = sen(222)
Logo as soluções são  sen(6), sen(78)  - sen(42) e - sen(66)
From: marconeborge...@hotmail.com
To: obm-l@mat.puc-rio.br
Subject: [obm-l] FW: Equação polinomial(ajuda)
Date: Sat, 23 Apr 2011 21:26:04 +









 



From: marconeborge...@hotmail.com
To: obm-l@mat.puc-rio.br
Subject: FW: Equação polinomial(ajuda)
Date: Sat, 23 Apr 2011 21:17:13 +





 


From: marconeborge...@hotmail.com
To: obm-l@mat.puc-rio.br
Subject: Equação polinomial(ajuda)
Date: Sat, 23 Apr 2011 00:59:49 +



Não quero a solução,gostaria de esclarecimentos ou dicas .
Achar as raízes de 16x^4+8x^3-16x^2-8x+1=0.
Sugestão:Escreva sen(5a) em termos de sen(a).
Eu encontrei sen(5a)=16(sena)^5-20(sena)^3+5sena
Dividindo um polinômio pelo outro obtive:
16x^5-20x^3+5x=(16x^4+8x^3-16x^2-8x+1)(x-1/2) +1/2
Pensei:se x diferente de 1/2 e 16x^5-20x^3+5x=1/2,então
16x^4+8x^3-16x^2-8x+1=0,dai,fazendo x=sena, sen(5a)=1/2 e x=sen6 é solução.
Se isso está certo,e as outras raízes?
Tentei as possíveis raízes racionais,mas não deu.
 
 
  

[obm-l] Re: [obm-l] Re: [obm-l] FW: [obm-l] Re: [obm-l] RE: [obm-l] Olim píadas cearenses(geometria)

2011-04-21 Por tôpico Julio César Saldaña



Essa propriedade é valida, não só para trapecios, mas para qualquer cuadilátero.

No cuadrilátero que tem por vertices os pontos medios do primeiro, cada par de
lados opostos são paralelos a uma diagonal do primeiro, e iguai à metade de
esta. Logo, é um paralelogramo.

Julio Saldaña


-- Mensaje original ---
De : obm-l@mat.puc-rio.br
Para : obm-l@mat.puc-rio.br
Fecha : Thu, 21 Apr 2011 18:30:00 -0300
Asunto : [obm-l] Re: [obm-l] FW: [obm-l] Re: [obm-l] RE: [obm-l] Olim píadas
cearenses(geometria)

Tem razão, Marcone e João... relendo agora entendi melhor a questão.
Abs.

Hugo.

Em 21 de abril de 2011 14:28, marcone augusto araújo borges <
marconeborge...@hotmail.com> escreveu:




--
From: marconeborge...@hotmail.com
To: obm-l@mat.puc-rio.br
Subject: RE: [obm-l] Re: [obm-l] RE: [obm-l] Olimpíadas
cearenses(geometria)
Date: Thu, 21 Apr 2011 16:05:12 +


 O João está certo.E minha pergunta no final foi se basta provar que DOIS
lados opostos são paralelos e congruentes.Um abraço.
--
Date: Wed, 20 Apr 2011 23:10:44 -0300
Subject: [obm-l] Re: [obm-l] RE: [obm-l] Olimpíadas cearenses(geometria)
From: hfernande...@gmail.com

To: obm-l@mat.puc-rio.br

Não entendi bem sua solução, João.

Pelo que diz o enunciado, os vértices do trapézio são os pontos médios de
um quadrilátero convexo.
Da maneira como você fez, parece que você considerou o quadrilátero formado
pelos pontos médios dos lados do trapézio.
É isso mesmo, ou estou enganado?

Abs.

Hugo.

Em 20 de abril de 2011 20:38, João Maldonado

escreveu:


*Na verdade basta provar que os lados opostos são iguais, automaticamente
serão paralelos.*
*Todo uq**adrilátero com ladosopostos iguais é um paralelogramo.*
*
*
*Prova:*
*Faça dois lados (a e b) de um quadrilátero qualquer saindo de um vértice
V. Para que os lados opostos sejm iguais podemos traçar uma circunferência a
partir do fim dos lados a e b, com raio igual ao lado oposto. Desse modo
teríamos 2 circunferências, que se intersectam em 2 pontos. Um dos pontos
gera uma configuração de quadrilátero não convexo, a  outra gera um
quadrilátero convexo. Logo os lados são paralelos.*
*
*
*Mas voltando ao problema,*
*
*
*
*
*Fazendo o trapézio ABCD com lados paralelos AB e CD. Os pontos médios de
AB=X, BC=Y, CD=Z, DA=W. A altura do trapézio h que parte de A intersecta CD
em P e a altura do trapézio que parte de B instersecta CD em Q (neste caso
fazendo P e Q dentro do segmento CD (fica para você provar quando um está
fora). Chamando AB/2 de d, PD de a e QC de b, temos que:*
*1) Em relação a CD, a coordenada y de W é  h/2, e a coordenada x é
(2d+a+b)/2 - a/2 = d+b/2*
*2) Em relação a AB (que é paralela a CD), logo em relação  a CD, a
coordenada y de W é h/2 e a coordenada x é d+b/2, logo os Ângulos formados
com  AB são iguais e as retas WZ e XY são paralelas e de mesma  medida*
*Analogamente para YZ e WX.*
*
*
*Logo se trata de um paralelogramo*
*
*
*
*
*[]\'s*
*João*
*
*
 *
*
 --
From: marconeborge...@hotmail.com
To: obm-l@mat.puc-rio.br
Subject: [obm-l] Olimpíadas cearenses(geometria)
Date: Wed, 20 Apr 2011 22:06:10 +


Prove que um quadrilatero convexo cujos vertices sao os pontos medios dos
lados de um trapezio qualquer é um paralelogramo
Bastaria provar que dos lados opostos são paralelos e congruentes?






__
Si desea recibir, semanalmente, el Boletín Electrónico de la PUCP, ingrese a:
http://www.pucp.edu.pe/puntoedu/suscribete/

=
Instruções para entrar na lista, sair da lista e usar a lista em
http://www.mat.puc-rio.br/~obmlistas/obm-l.html
=


[obm-l] Re: [obm-l] FW: [obm-l] Re: [obm-l] RE: [obm-l] Olimpíadas cearenses(geometria)

2011-04-21 Por tôpico Hugo Fernando Marques Fernandes
Tem razão, Marcone e João... relendo agora entendi melhor a questão.
Abs.

Hugo.

Em 21 de abril de 2011 14:28, marcone augusto araújo borges <
marconeborge...@hotmail.com> escreveu:

>
>
> --
> From: marconeborge...@hotmail.com
> To: obm-l@mat.puc-rio.br
> Subject: RE: [obm-l] Re: [obm-l] RE: [obm-l] Olimpíadas
> cearenses(geometria)
> Date: Thu, 21 Apr 2011 16:05:12 +
>
>
>  O João está certo.E minha pergunta no final foi se basta provar que DOIS
> lados opostos são paralelos e congruentes.Um abraço.
> --
> Date: Wed, 20 Apr 2011 23:10:44 -0300
> Subject: [obm-l] Re: [obm-l] RE: [obm-l] Olimpíadas cearenses(geometria)
> From: hfernande...@gmail.com
>
> To: obm-l@mat.puc-rio.br
>
> Não entendi bem sua solução, João.
>
> Pelo que diz o enunciado, os vértices do trapézio são os pontos médios de
> um quadrilátero convexo.
> Da maneira como você fez, parece que você considerou o quadrilátero formado
> pelos pontos médios dos lados do trapézio.
> É isso mesmo, ou estou enganado?
>
> Abs.
>
> Hugo.
>
> Em 20 de abril de 2011 20:38, João Maldonado 
> escreveu:
>
> *Na verdade basta provar que os lados opostos são iguais, automaticamente
> serão paralelos.*
> *Todo uq**adrilátero com ladosopostos iguais é um paralelogramo.*
> *
> *
> *Prova:*
> *Faça dois lados (a e b) de um quadrilátero qualquer saindo de um vértice
> V. Para que os lados opostos sejm iguais podemos traçar uma circunferência a
> partir do fim dos lados a e b, com raio igual ao lado oposto. Desse modo
> teríamos 2 circunferências, que se intersectam em 2 pontos. Um dos pontos
> gera uma configuração de quadrilátero não convexo, a  outra gera um
> quadrilátero convexo. Logo os lados são paralelos.*
> *
> *
> *Mas voltando ao problema,*
> *
> *
> *
> *
> *Fazendo o trapézio ABCD com lados paralelos AB e CD. Os pontos médios de
> AB=X, BC=Y, CD=Z, DA=W. A altura do trapézio h que parte de A intersecta CD
> em P e a altura do trapézio que parte de B instersecta CD em Q (neste caso
> fazendo P e Q dentro do segmento CD (fica para você provar quando um está
> fora). Chamando AB/2 de d, PD de a e QC de b, temos que:*
> *1) Em relação a CD, a coordenada y de W é  h/2, e a coordenada x é
> (2d+a+b)/2 - a/2 = d+b/2*
> *2) Em relação a AB (que é paralela a CD), logo em relação  a CD, a
> coordenada y de W é h/2 e a coordenada x é d+b/2, logo os Ângulos formados
> com  AB são iguais e as retas WZ e XY são paralelas e de mesma  medida*
> *Analogamente para YZ e WX.*
> *
> *
> *Logo se trata de um paralelogramo*
> *
> *
> *
> *
> *[]'s*
> *João*
> *
> *
>  *
> *
>  --
> From: marconeborge...@hotmail.com
> To: obm-l@mat.puc-rio.br
> Subject: [obm-l] Olimpíadas cearenses(geometria)
> Date: Wed, 20 Apr 2011 22:06:10 +
>
>
> Prove que um quadrilatero convexo cujos vertices sao os pontos medios dos
> lados de um trapezio qualquer é um paralelogramo
> Bastaria provar que dos lados opostos são paralelos e congruentes?
>
>
>


[obm-l] RE: [obm-l] Re: [obm-l] RE: [obm-l] FW: Progressão aritmética

2011-04-21 Por tôpico João Maldonado

k, Realmente não sei daonde  eu fui tirar PG, ;D

Mas enfim,

Na PA, tendo o primeiro termo a e a razão k:

1) a³ = a+7k
2) a² = a+nk, 1<=n<=6
3) a^4 = a+mk,m>7
 
De 2), resolvendo a equação do segndo grau, temos a=(1+sqrt(4nk + 1))/2
Chamando sqrt(4nk+ 1) de x para simplificar e substituindo em 1) 
x³ + 3x² -x = 3 + 56k
Substiuindo x
4nk(sqrt(4nk+1)) = 4K(14 - 3n) -> n²(4nk+1) = (14-3n) ² -> k =[  ((14-3n)/n)²   
-   1]/4n = (14 - 2n)(14 - 4n)/4n³
Substituindo n por 1,2,3,4,5 ou 6, vemos que k só é inteiro com n = 1, de onde 
k = 30 e a = 6
a² seria 36 e a^4 1296 = 6 + 1290 = 6 + 43k que  é o 44° termo
Logo  o segundo termo é 36

[]s
João


Date: Wed, 20 Apr 2011 23:08:17 -0300
Subject: [obm-l] Re: [obm-l] RE: [obm-l] FW: Progressão aritmética
From: hfernande...@gmail.com
To: obm-l@mat.puc-rio.br

O problema fala em progressão aritmética, não geométrica, João.

Abs.

Hugo.

Em 20 de abril de 2011 21:02, João Maldonado  
escreveu:






Primeiramente note que o primeiro é positivo e a razão também.

Chamando o primeiro termo de a e a razão de k, o termo n vale a.k^(n-1)

logo temos:

1) a³ = a.k^72) a² pertence à progreesão
3) a^4 pertence à progressão
De 1) a = k^(7/2)

Temos que a² ou a^4 está entre o primeiro e o oitavo termo. 

Fazendo a² = a.k^(n-1) -> a = k^(n-1) -> k^(7/2) = k^(n-1) -> k=1

Fazendo a^4, a mesma coisa

Logo o segundo termo é 1.

Possivelmente  errei em alguma coisa porque nnunca vi progressão geometrica com 
razão 1 .

Mas se o resultado bateu com o seu acho queo problema é o enunciado mesmo. Vou 
rever de novo minha solução, qualquer coisa posto outra vez.


AbraçoJoão

From: marconeborge...@hotmail.com

To: obm-l@mat.puc-rio.br
Subject: [obm-l] FW: Progressão aritmética
Date: Wed, 20 Apr 2011 22:21:55 +










 



From: marconeborge...@hotmail.com
To: obm-l@mat.puc-rio.br

Subject: Progressão aritmética
Date: Wed, 20 Apr 2011 21:58:18 +




Numa progressao aritmetica de  numeros inteiros positivos,o oitavo termo é 
igual ao cubo do primeiro.Sabendo que a segunda e a quarta potencias do 
primeiro termo pertencem a progressao,determinar o segundo termo.

Agradeço antecipadamente a quem puder resolver.
  

  

[obm-l] Re: [obm-l] RE: [obm-l] FW: Progressão aritmética

2011-04-20 Por tôpico Hugo Fernando Marques Fernandes
O problema fala em progressão aritmética, não geométrica, João.

Abs.

Hugo.

Em 20 de abril de 2011 21:02, João Maldonado
escreveu:

>  *Primeiramente note que o primeiro é positivo e a razão também.
>
> *
> *Chamando o primeiro termo de a e a razão de k, o termo n vale a.k^(n-1)*
> *
> *
> *logo temos:*
> *
> *
> *1) a³ = a.k^7*
> *2) a² pertence à progreesão*
> *3) a^4 pertence à progressão*
> *
> *
> *De 1) a = k^(7/2)*
> *
> *
> *Temos que a² ou a^4 está entre o primeiro e o oitavo termo. *
> *
> *
> *Fazendo a² = a.k^(n-1) -> a = k^(n-1) -> k^(7/2) = k^(n-1) -> k=1*
> *
> *
> *Fazendo a^4, a mesma coisa*
> *
> *
> *Logo o segundo termo é 1.*
> *
> *
> *Possivelmente  errei em alguma coisa porque nnunca vi progressão
> geometrica com razão 1 .*
> *
> *
> *Mas se o resultado bateu com o seu acho queo problema é o enunciado
> mesmo. Vou rever de novo minha solução, qualquer coisa posto outra vez.*
> *
> *
> *
> Abraço*
> *João*
>
> --
> From: marconeborge...@hotmail.com
> To: obm-l@mat.puc-rio.br
> Subject: [obm-l] FW: Progressão aritmética
> Date: Wed, 20 Apr 2011 22:21:55 +
>
>
>
>
> --
> From: marconeborge...@hotmail.com
> To: obm-l@mat.puc-rio.br
> Subject: Progressão aritmética
> Date: Wed, 20 Apr 2011 21:58:18 +
>
>  Numa progressao aritmetica de  numeros inteiros positivos,o oitavo termo é
> igual ao cubo do primeiro.Sabendo que a segunda e a quarta potencias do
> primeiro termo pertencem a progressao,determinar o segundo termo.
> Agradeço antecipadamente a quem puder resolver.
>


[obm-l] RE: [obm-l] FW: Progressão aritmética

2011-04-20 Por tôpico João Maldonado

Primeiramente note que o primeiro é positivo e a razão também.

Chamando o primeiro termo de a e a razão de k, o termo n vale a.k^(n-1)
logo temos:
1) a³ = a.k^72) a² pertence à progreesão3) a^4 pertence à progressão
De 1) a = k^(7/2)
Temos que a² ou a^4 está entre o primeiro e o oitavo termo. 
Fazendo a² = a.k^(n-1) -> a = k^(n-1) -> k^(7/2) = k^(n-1) -> k=1
Fazendo a^4, a mesma coisa
Logo o segundo termo é 1.
Possivelmente  errei em alguma coisa porque nnunca vi progressão geometrica com 
razão 1 .
Mas se o resultado bateu com o seu acho queo problema é o enunciado mesmo. Vou 
rever de novo minha solução, qualquer coisa posto outra vez.

AbraçoJoão
From: marconeborge...@hotmail.com
To: obm-l@mat.puc-rio.br
Subject: [obm-l] FW: Progressão aritmética
Date: Wed, 20 Apr 2011 22:21:55 +









 



From: marconeborge...@hotmail.com
To: obm-l@mat.puc-rio.br
Subject: Progressão aritmética
Date: Wed, 20 Apr 2011 21:58:18 +




Numa progressao aritmetica de  numeros inteiros positivos,o oitavo termo é 
igual ao cubo do primeiro.Sabendo que a segunda e a quarta potencias do 
primeiro termo pertencem a progressao,determinar o segundo termo.
Agradeço antecipadamente a quem puder resolver.
  

[obm-l] RE: [obm-l] RE: [obm-l] RE: [obm-l] RE: [obm-l] RE: [obm-l] RE: [obm-l] FW: Equação do segundo grau(raiz inteira)

2011-01-23 Por tôpico João Maldonado

Realmente estava vendo   aqui e para m > n + 1, n² > 2n, e não dá para fazer 
nenhuma suposição.
Ainda falta provar que as únicas raízes inteiras de 8a² + 1 = k² são -1, 0 e 1
Estou tentando aqui e cheguei  em t² = 2.(v²+1), mas ainda tenho que provar que 
os únicos valores de v são -1 e 1 (claro, se isso for mais fácil  que provar 
que as únicas solções de t² = 2v² + 1 é 0, -2 2) .
Se chegar em algo aviso (fica aí pra quem conseguir provar)

Abrr.

From: marconeborge...@hotmail.com
To: obm-l@mat.puc-rio.br
Subject: [obm-l] RE: [obm-l] RE: [obm-l] RE: [obm-l] RE: [obm-l] RE: [obm-l] 
FW: Equação do segundo grau(raiz inteira)
Date: Mon, 24 Jan 2011 00:48:16 +








Desculpe...mas ainda não entendi por que -n^2 > 1 - 2n -1 nem por que 2a = -2
 



From: joao_maldona...@hotmail.com
To: obm-l@mat.puc-rio.br
Subject: [obm-l] RE: [obm-l] RE: [obm-l] RE: [obm-l] RE: [obm-l] FW: Equação do 
segundo grau(raiz inteira)
Date: Sun, 23 Jan 2011 20:04:39 -0200




Olá Marcone,  

Realmente onde está escrito n² = 2n² seria n² = 2n
Quando escrevi m = n + (1 + k), tinha me esquecido que já tinha usado o k 
antes, esse k não tem nada a ver com aquele k do começo, usei ele para indicar 
uma contante, pode substituir por z por exemplo, para m >= n + 2, m = (1   +z), 
z inteiro maior ou igual a 1.



From: marconeborge...@hotmail.com
To: obm-l@mat.puc-rio.br
Subject: [obm-l] RE: [obm-l] RE: [obm-l] RE: [obm-l] FW: Equação do segundo 
grau(raiz inteira)
Date: Sat, 22 Jan 2011 01:56:44 +



Onde ta escrito n^2 =2^2n deve ser n^2 = 2n
Nao entendi por que m = n + ( 1 + k), o que comprometeu o entendimento do resto
O que significa k^2 - 1 ~ k^2 ?
Obrigado. 
  


From: joao_maldona...@hotmail.com
To: obm-l@mat.puc-rio.br
Subject: [obm-l] RE: [obm-l] RE: [obm-l] FW: Equação do segundo grau(raiz 
inteira)
Date: Fri, 21 Jan 2011 20:48:15 -0200



Decompondo essa equação chegamos a:
(a² + b²)x² + (-4ab - 1)x + (a² + b²) = 0 
delta = (4ab + 1)² -4(a² + b²)² -> diferença de dois quadrados
delta = (4ab+ 1 - 2a² - 2b²).(4ab + 1 + 2a² + 2b²) =  (-2.(a-b)² + 1).(2.(a+b)² 
+ 1) = k²
Se a>b ou a=1 e (-2.(a-b)² + 1) é negativo, e como (2.(a+b)² + 1) é 
sempre positivo, temos que a=b.


para a=b: 8a² + 1 = k²
8a² = k² - 1 ~ k²
2.(2a)² = k² - 1
Para faiilitar, faça 2a = n, k = m
m² - 2n² = 1
Imediatamente m > n
Se m = n + 1
(n+1)² - 2n² = 1
n² + 2n + 1 - 2n² = 1
n² = 2n²
n=0-> m = +-1, n = 2, m = +- 3
Se m >= n + 2, m = n + (1+k)
-n² > 1 -2n - 1
-n² > -2n 
n² < 2n   >>>> n < 0,impossível ( exitiria também a solução positiva) ou n 
< 2, temos n = 1
m² = 3, impossível
Logo temos 2a = 0, a = 0
  2a = -2, a = -1
2a = 2, a = 1 
a = b = 0 -> x = 0
a = b = -1 -> x = 0,5 e 2
a = b = 1 -> x = 0,5 e 2
raizes = 0   0,5   2

Abraço



From: marconeborge...@hotmail.com
To: obm-l@mat.puc-rio.br
Subject: [obm-l] RE: [obm-l] FW: Equação do segundo grau(raiz inteira)
Date: Fri, 21 Jan 2011 15:20:26 +



Desculpem,essa mensagem já havia saido e o marcelo postou uma solução 
interessante,faltando apenas esclarecer um detalhe



From: marconeborge...@hotmail.com
To: obm-l@mat.puc-rio.br
Subject: [obm-l] FW: Equação do segundo grau(raiz inteira)
Date: Sat, 15 Jan 2011 19:47:29 +




 


From: marconeborge...@hotmail.com
To: obm-l@mat.puc-rio.br
Subject: Equação do segundo grau(raiz inteira)
Date: Sun, 9 Jan 2011 02:27:05 +



Sejam a,b números inteiros .Sabendo que  a equação (ax - b)^2 + ( bx - a)^2 = x 
tem uma raiz inteira,encontre os valores de suas raizes.
Conto com a habitual atenção de todos,pela qual agradeço antecipadamente.
  

[obm-l] RE: [obm-l] RE: [obm-l] RE: [obm-l] RE: [obm-l] RE: [obm-l] FW: Equação do segundo grau(raiz inteira)

2011-01-23 Por tôpico marcone augusto araújo borges

Desculpe...mas ainda não entendi por que -n^2 > 1 - 2n -1 nem por que 2a = -2
 


From: joao_maldona...@hotmail.com
To: obm-l@mat.puc-rio.br
Subject: [obm-l] RE: [obm-l] RE: [obm-l] RE: [obm-l] RE: [obm-l] FW: Equação do 
segundo grau(raiz inteira)
Date: Sun, 23 Jan 2011 20:04:39 -0200




Olá Marcone,  

Realmente onde está escrito n² = 2n² seria n² = 2n
Quando escrevi m = n + (1 + k), tinha me esquecido que já tinha usado o k 
antes, esse k não tem nada a ver com aquele k do começo, usei ele para indicar 
uma contante, pode substituir por z por exemplo, para m >= n + 2, m = (1   +z), 
z inteiro maior ou igual a 1.



From: marconeborge...@hotmail.com
To: obm-l@mat.puc-rio.br
Subject: [obm-l] RE: [obm-l] RE: [obm-l] RE: [obm-l] FW: Equação do segundo 
grau(raiz inteira)
Date: Sat, 22 Jan 2011 01:56:44 +



Onde ta escrito n^2 =2^2n deve ser n^2 = 2n
Nao entendi por que m = n + ( 1 + k), o que comprometeu o entendimento do resto
O que significa k^2 - 1 ~ k^2 ?
Obrigado. 
  


From: joao_maldona...@hotmail.com
To: obm-l@mat.puc-rio.br
Subject: [obm-l] RE: [obm-l] RE: [obm-l] FW: Equação do segundo grau(raiz 
inteira)
Date: Fri, 21 Jan 2011 20:48:15 -0200



Decompondo essa equação chegamos a:
(a² + b²)x² + (-4ab - 1)x + (a² + b²) = 0 
delta = (4ab + 1)² -4(a² + b²)² -> diferença de dois quadrados
delta = (4ab+ 1 - 2a² - 2b²).(4ab + 1 + 2a² + 2b²) =  (-2.(a-b)² + 1).(2.(a+b)² 
+ 1) = k²
Se a>b ou a=1 e (-2.(a-b)² + 1) é negativo, e como (2.(a+b)² + 1) é 
sempre positivo, temos que a=b.


para a=b: 8a² + 1 = k²
8a² = k² - 1 ~ k²
2.(2a)² = k² - 1
Para faiilitar, faça 2a = n, k = m
m² - 2n² = 1
Imediatamente m > n
Se m = n + 1
(n+1)² - 2n² = 1
n² + 2n + 1 - 2n² = 1
n² = 2n²
n=0-> m = +-1, n = 2, m = +- 3
Se m >= n + 2, m = n + (1+k)
-n² > 1 -2n - 1
-n² > -2n 
n² < 2n   >>>> n < 0,impossível ( exitiria também a solução positiva) ou n 
< 2, temos n = 1
m² = 3, impossível
Logo temos 2a = 0, a = 0
  2a = -2, a = -1
2a = 2, a = 1 
a = b = 0 -> x = 0
a = b = -1 -> x = 0,5 e 2
a = b = 1 -> x = 0,5 e 2
raizes = 0   0,5   2

Abraço



From: marconeborge...@hotmail.com
To: obm-l@mat.puc-rio.br
Subject: [obm-l] RE: [obm-l] FW: Equação do segundo grau(raiz inteira)
Date: Fri, 21 Jan 2011 15:20:26 +



Desculpem,essa mensagem já havia saido e o marcelo postou uma solução 
interessante,faltando apenas esclarecer um detalhe



From: marconeborge...@hotmail.com
To: obm-l@mat.puc-rio.br
Subject: [obm-l] FW: Equação do segundo grau(raiz inteira)
Date: Sat, 15 Jan 2011 19:47:29 +




 


From: marconeborge...@hotmail.com
To: obm-l@mat.puc-rio.br
Subject: Equação do segundo grau(raiz inteira)
Date: Sun, 9 Jan 2011 02:27:05 +



Sejam a,b números inteiros .Sabendo que  a equação (ax - b)^2 + ( bx - a)^2 = x 
tem uma raiz inteira,encontre os valores de suas raizes.
Conto com a habitual atenção de todos,pela qual agradeço antecipadamente.
  

[obm-l] RE: [obm-l] RE: [obm-l] RE: [obm-l] RE: [obm-l] FW: Equação do segundo grau(raiz inteira)

2011-01-23 Por tôpico João Maldonado

Olá Marcone,  

Realmente onde está escrito n² = 2n² seria n² = 2n
Quando escrevi m = n + (1 + k), tinha me esquecido que já tinha usado o k 
antes, esse k não tem nada a ver com aquele k do começo, usei ele para indicar 
uma contante, pode substituir por z por exemplo, para m >= n + 2, m = (1   +z), 
z inteiro maior ou igual a 1.

From: marconeborge...@hotmail.com
To: obm-l@mat.puc-rio.br
Subject: [obm-l] RE: [obm-l] RE: [obm-l] RE: [obm-l] FW: Equação do segundo 
grau(raiz inteira)
Date: Sat, 22 Jan 2011 01:56:44 +








Onde ta escrito n^2 =2^2n deve ser n^2 = 2n
Nao entendi por que m = n + ( 1 + k), o que comprometeu o entendimento do resto

O que significa k^2 - 1 ~ k^2 ?

Obrigado. 

  



From: joao_maldona...@hotmail.com
To: obm-l@mat.puc-rio.br
Subject: [obm-l] RE: [obm-l] RE: [obm-l] FW: Equação do segundo grau(raiz 
inteira)
Date: Fri, 21 Jan 2011 20:48:15 -0200




Decompondo essa equação chegamos a:
(a² + b²)x² + (-4ab - 1)x + (a² + b²) = 0 
delta = (4ab + 1)² -4(a² + b²)² -> diferença de dois quadrados
delta = (4ab+ 1 - 2a² - 2b²).(4ab + 1 + 2a² + 2b²) =  (-2.(a-b)² + 1).(2.(a+b)² 
+ 1) = k²
Se a>b ou a=1 e (-2.(a-b)² + 1) é negativo, e como (2.(a+b)² + 1) é 
sempre positivo, temos que a=b.


para a=b: 8a² + 1 = k²
8a² = k² - 1 ~ k²
2.(2a)² = k² - 1
Para faiilitar, faça 2a = n, k = m
m² - 2n² = 1
Imediatamente m > n
Se m = n + 1
(n+1)² - 2n² = 1
n² + 2n + 1 - 2n² = 1
n² = 2n²
n=0-> m = +-1, n = 2, m = +- 3
Se m >= n + 2, m = n + (1+k)
-n² > 1 -2n - 1
-n² > -2n 
n² < 2n   >>>> n < 0,impossível ( exitiria também a solução positiva) ou n 
< 2, temos n = 1
m² = 3, impossível
Logo temos 2a = 0, a = 0
  2a = -2, a = -1
2a = 2, a = 1 
a = b = 0 -> x = 0
a = b = -1 -> x = 0,5 e 2
a = b = 1 -> x = 0,5 e 2
raizes = 0   0,5   2

Abraço



From: marconeborge...@hotmail.com
To: obm-l@mat.puc-rio.br
Subject: [obm-l] RE: [obm-l] FW: Equação do segundo grau(raiz inteira)
Date: Fri, 21 Jan 2011 15:20:26 +



Desculpem,essa mensagem já havia saido e o marcelo postou uma solução 
interessante,faltando apenas esclarecer um detalhe



From: marconeborge...@hotmail.com
To: obm-l@mat.puc-rio.br
Subject: [obm-l] FW: Equação do segundo grau(raiz inteira)
Date: Sat, 15 Jan 2011 19:47:29 +




 


From: marconeborge...@hotmail.com
To: obm-l@mat.puc-rio.br
Subject: Equação do segundo grau(raiz inteira)
Date: Sun, 9 Jan 2011 02:27:05 +



Sejam a,b números inteiros .Sabendo que  a equação (ax - b)^2 + ( bx - a)^2 = x 
tem uma raiz inteira,encontre os valores de suas raizes.
Conto com a habitual atenção de todos,pela qual agradeço antecipadamente.
  

[obm-l] RE: [obm-l] RE: [obm-l] RE: [obm-l] RE: [obm-l] FW: Equação do segundo grau(raiz inteira)

2011-01-21 Por tôpico marcone augusto araújo borges

onde aparece n^2 = 2n^2 deveria aparecer n^2 = 2n.
 


From: marconeborge...@hotmail.com
To: obm-l@mat.puc-rio.br
Subject: [obm-l] RE: [obm-l] RE: [obm-l] RE: [obm-l] FW: Equação do segundo 
grau(raiz inteira)
Date: Sat, 22 Jan 2011 01:56:44 +




Onde ta escrito n^2 =2^2n deve ser n^2 = 2n
Nao entendi por que m = n + ( 1 + k), o que comprometeu o entendimento do resto
O que significa k^2 - 1 ~ k^2 ?
Obrigado. 
  


From: joao_maldona...@hotmail.com
To: obm-l@mat.puc-rio.br
Subject: [obm-l] RE: [obm-l] RE: [obm-l] FW: Equação do segundo grau(raiz 
inteira)
Date: Fri, 21 Jan 2011 20:48:15 -0200



Decompondo essa equação chegamos a:
(a² + b²)x² + (-4ab - 1)x + (a² + b²) = 0 
delta = (4ab + 1)² -4(a² + b²)² -> diferença de dois quadrados
delta = (4ab+ 1 - 2a² - 2b²).(4ab + 1 + 2a² + 2b²) =  (-2.(a-b)² + 1).(2.(a+b)² 
+ 1) = k²
Se a>b ou a=1 e (-2.(a-b)² + 1) é negativo, e como (2.(a+b)² + 1) é 
sempre positivo, temos que a=b.


para a=b: 8a² + 1 = k²
8a² = k² - 1 ~ k²
2.(2a)² = k² - 1
Para faiilitar, faça 2a = n, k = m
m² - 2n² = 1
Imediatamente m > n
Se m = n + 1
(n+1)² - 2n² = 1
n² + 2n + 1 - 2n² = 1
n² = 2n²
n=0-> m = +-1, n = 2, m = +- 3
Se m >= n + 2, m = n + (1+k)
-n² > 1 -2n - 1
-n² > -2n 
n² < 2n   >>>> n < 0,impossível ( exitiria também a solução positiva) ou n 
< 2, temos n = 1
m² = 3, impossível
Logo temos 2a = 0, a = 0
  2a = -2, a = -1
2a = 2, a = 1 
a = b = 0 -> x = 0
a = b = -1 -> x = 0,5 e 2
a = b = 1 -> x = 0,5 e 2
raizes = 0   0,5   2

Abraço



From: marconeborge...@hotmail.com
To: obm-l@mat.puc-rio.br
Subject: [obm-l] RE: [obm-l] FW: Equação do segundo grau(raiz inteira)
Date: Fri, 21 Jan 2011 15:20:26 +



Desculpem,essa mensagem já havia saido e o marcelo postou uma solução 
interessante,faltando apenas esclarecer um detalhe



From: marconeborge...@hotmail.com
To: obm-l@mat.puc-rio.br
Subject: [obm-l] FW: Equação do segundo grau(raiz inteira)
Date: Sat, 15 Jan 2011 19:47:29 +




 


From: marconeborge...@hotmail.com
To: obm-l@mat.puc-rio.br
Subject: Equação do segundo grau(raiz inteira)
Date: Sun, 9 Jan 2011 02:27:05 +



Sejam a,b números inteiros .Sabendo que  a equação (ax - b)^2 + ( bx - a)^2 = x 
tem uma raiz inteira,encontre os valores de suas raizes.
Conto com a habitual atenção de todos,pela qual agradeço antecipadamente.
  

[obm-l] RE: [obm-l] RE: [obm-l] RE: [obm-l] RE: [obm-l] FW: Equação do segundo grau(raiz inteira)

2011-01-21 Por tôpico marcone augusto araújo borges

corrigindo: onde aparece n^2 = 2n^2 deveria ter n^2 = 2n
 


From: marconeborge...@hotmail.com
To: obm-l@mat.puc-rio.br
Subject: [obm-l] RE: [obm-l] RE: [obm-l] RE: [obm-l] FW: Equação do segundo 
grau(raiz inteira)
Date: Sat, 22 Jan 2011 01:56:44 +




Onde ta escrito n^2 =2^2n deve ser n^2 = 2n
Nao entendi por que m = n + ( 1 + k), o que comprometeu o entendimento do resto
O que significa k^2 - 1 ~ k^2 ?
Obrigado. 
  


From: joao_maldona...@hotmail.com
To: obm-l@mat.puc-rio.br
Subject: [obm-l] RE: [obm-l] RE: [obm-l] FW: Equação do segundo grau(raiz 
inteira)
Date: Fri, 21 Jan 2011 20:48:15 -0200



Decompondo essa equação chegamos a:
(a² + b²)x² + (-4ab - 1)x + (a² + b²) = 0 
delta = (4ab + 1)² -4(a² + b²)² -> diferença de dois quadrados
delta = (4ab+ 1 - 2a² - 2b²).(4ab + 1 + 2a² + 2b²) =  (-2.(a-b)² + 1).(2.(a+b)² 
+ 1) = k²
Se a>b ou a=1 e (-2.(a-b)² + 1) é negativo, e como (2.(a+b)² + 1) é 
sempre positivo, temos que a=b.


para a=b: 8a² + 1 = k²
8a² = k² - 1 ~ k²
2.(2a)² = k² - 1
Para faiilitar, faça 2a = n, k = m
m² - 2n² = 1
Imediatamente m > n
Se m = n + 1
(n+1)² - 2n² = 1
n² + 2n + 1 - 2n² = 1
n² = 2n²
n=0-> m = +-1, n = 2, m = +- 3
Se m >= n + 2, m = n + (1+k)
-n² > 1 -2n - 1
-n² > -2n 
n² < 2n   >>>> n < 0,impossível ( exitiria também a solução positiva) ou n 
< 2, temos n = 1
m² = 3, impossível
Logo temos 2a = 0, a = 0
  2a = -2, a = -1
2a = 2, a = 1 
a = b = 0 -> x = 0
a = b = -1 -> x = 0,5 e 2
a = b = 1 -> x = 0,5 e 2
raizes = 0   0,5   2

Abraço



From: marconeborge...@hotmail.com
To: obm-l@mat.puc-rio.br
Subject: [obm-l] RE: [obm-l] FW: Equação do segundo grau(raiz inteira)
Date: Fri, 21 Jan 2011 15:20:26 +



Desculpem,essa mensagem já havia saido e o marcelo postou uma solução 
interessante,faltando apenas esclarecer um detalhe



From: marconeborge...@hotmail.com
To: obm-l@mat.puc-rio.br
Subject: [obm-l] FW: Equação do segundo grau(raiz inteira)
Date: Sat, 15 Jan 2011 19:47:29 +




 


From: marconeborge...@hotmail.com
To: obm-l@mat.puc-rio.br
Subject: Equação do segundo grau(raiz inteira)
Date: Sun, 9 Jan 2011 02:27:05 +



Sejam a,b números inteiros .Sabendo que  a equação (ax - b)^2 + ( bx - a)^2 = x 
tem uma raiz inteira,encontre os valores de suas raizes.
Conto com a habitual atenção de todos,pela qual agradeço antecipadamente.
  

[obm-l] RE: [obm-l] RE: [obm-l] RE: [obm-l] FW: Equação do segundo grau(raiz inteira)

2011-01-21 Por tôpico marcone augusto araújo borges

Onde ta escrito n^2 =2^2n deve ser n^2 = 2n
Nao entendi por que m = n + ( 1 + k), o que comprometeu o entendimento do resto
O que significa k^2 - 1 ~ k^2 ?
Obrigado. 
  


From: joao_maldona...@hotmail.com
To: obm-l@mat.puc-rio.br
Subject: [obm-l] RE: [obm-l] RE: [obm-l] FW: Equação do segundo grau(raiz 
inteira)
Date: Fri, 21 Jan 2011 20:48:15 -0200




Decompondo essa equação chegamos a:
(a² + b²)x² + (-4ab - 1)x + (a² + b²) = 0 
delta = (4ab + 1)² -4(a² + b²)² -> diferença de dois quadrados
delta = (4ab+ 1 - 2a² - 2b²).(4ab + 1 + 2a² + 2b²) =  (-2.(a-b)² + 1).(2.(a+b)² 
+ 1) = k²
Se a>b ou a=1 e (-2.(a-b)² + 1) é negativo, e como (2.(a+b)² + 1) é 
sempre positivo, temos que a=b.


para a=b: 8a² + 1 = k²
8a² = k² - 1 ~ k²
2.(2a)² = k² - 1
Para faiilitar, faça 2a = n, k = m
m² - 2n² = 1
Imediatamente m > n
Se m = n + 1
(n+1)² - 2n² = 1
n² + 2n + 1 - 2n² = 1
n² = 2n²
n=0-> m = +-1, n = 2, m = +- 3
Se m >= n + 2, m = n + (1+k)
-n² > 1 -2n - 1
-n² > -2n 
n² < 2n   >>>> n < 0,impossível ( exitiria também a solução positiva) ou n 
< 2, temos n = 1
m² = 3, impossível
Logo temos 2a = 0, a = 0
  2a = -2, a = -1
2a = 2, a = 1 
a = b = 0 -> x = 0
a = b = -1 -> x = 0,5 e 2
a = b = 1 -> x = 0,5 e 2
raizes = 0   0,5   2

Abraço



From: marconeborge...@hotmail.com
To: obm-l@mat.puc-rio.br
Subject: [obm-l] RE: [obm-l] FW: Equação do segundo grau(raiz inteira)
Date: Fri, 21 Jan 2011 15:20:26 +



Desculpem,essa mensagem já havia saido e o marcelo postou uma solução 
interessante,faltando apenas esclarecer um detalhe



From: marconeborge...@hotmail.com
To: obm-l@mat.puc-rio.br
Subject: [obm-l] FW: Equação do segundo grau(raiz inteira)
Date: Sat, 15 Jan 2011 19:47:29 +




 


From: marconeborge...@hotmail.com
To: obm-l@mat.puc-rio.br
Subject: Equação do segundo grau(raiz inteira)
Date: Sun, 9 Jan 2011 02:27:05 +



Sejam a,b números inteiros .Sabendo que  a equação (ax - b)^2 + ( bx - a)^2 = x 
tem uma raiz inteira,encontre os valores de suas raizes.
Conto com a habitual atenção de todos,pela qual agradeço antecipadamente.
  

[obm-l] RE: [obm-l] RE: [obm-l] FW: Equação do segundo grau(raiz inteira)

2011-01-21 Por tôpico João Maldonado

Decompondo essa equação chegamos a:
(a² + b²)x² + (-4ab - 1)x + (a² + b²) = 0 
delta = (4ab + 1)² -4(a² + b²)² -> diferença de dois quadrados
delta = (4ab+ 1 - 2a² - 2b²).(4ab + 1 + 2a² + 2b²) =  (-2.(a-b)² + 1).(2.(a+b)² 
+ 1) = k²
Se a>b ou a=1 e (-2.(a-b)² + 1) é negativo, e como (2.(a+b)² + 1) é 
sempre positivo, temos que a=b.


para a=b: 8a² + 1 = k²
8a² = k² - 1 ~ k²
2.(2a)² = k² - 1
Para faiilitar, faça 2a = n, k = m
m² - 2n² = 1
Imediatamente m > n
Se m = n + 1
(n+1)² - 2n² = 1
n² + 2n + 1 - 2n² = 1
n² = 2n²
n=0-> m = +-1, n = 2, m = +- 3
Se m >= n + 2, m = n + (1+k)
-n² > 1 -2n - 1
-n² > -2n 
n² < 2n   >>>> n < 0,impossível ( exitiria também a solução positiva) ou n 
< 2, temos n = 1
m² = 3, impossível
Logo temos 2a = 0, a = 0
  2a = -2, a = -1
2a = 2, a = 1 
a = b = 0 -> x = 0
a = b = -1 -> x = 0,5 e 2
a = b = 1 -> x = 0,5 e 2
raizes = 0   0,5   2

Abraço

From: marconeborge...@hotmail.com
To: obm-l@mat.puc-rio.br
Subject: [obm-l] RE: [obm-l] FW: Equação do segundo grau(raiz inteira)
Date: Fri, 21 Jan 2011 15:20:26 +








Desculpem,essa mensagem já havia saido e o marcelo postou uma solução 
interessante,faltando apenas esclarecer um detalhe




From: marconeborge...@hotmail.com
To: obm-l@mat.puc-rio.br
Subject: [obm-l] FW: Equação do segundo grau(raiz inteira)
Date: Sat, 15 Jan 2011 19:47:29 +





 


From: marconeborge...@hotmail.com
To: obm-l@mat.puc-rio.br
Subject: Equação do segundo grau(raiz inteira)
Date: Sun, 9 Jan 2011 02:27:05 +



Sejam a,b números inteiros .Sabendo que  a equação (ax - b)^2 + ( bx - a)^2 = x 
tem uma raiz inteira,encontre os valores de suas raizes.
Conto com a habitual atenção de todos,pela qual agradeço antecipadamente.
  

[obm-l] RE: [obm-l] FW: Equação do segundo grau(raiz inteira)

2011-01-21 Por tôpico marcone augusto araújo borges

Desculpem,essa mensagem já havia saido e o marcelo postou uma solução 
interessante,faltando apenas esclarecer um detalhe
 


From: marconeborge...@hotmail.com
To: obm-l@mat.puc-rio.br
Subject: [obm-l] FW: Equação do segundo grau(raiz inteira)
Date: Sat, 15 Jan 2011 19:47:29 +





 


From: marconeborge...@hotmail.com
To: obm-l@mat.puc-rio.br
Subject: Equação do segundo grau(raiz inteira)
Date: Sun, 9 Jan 2011 02:27:05 +



Sejam a,b números inteiros .Sabendo que  a equação (ax - b)^2 + ( bx - a)^2 = x 
tem uma raiz inteira,encontre os valores de suas raizes.
Conto com a habitual atenção de todos,pela qual agradeço antecipadamente.
  

[obm-l] RES: [obm-l] Re: [obm-l] FW: Raízes irracionais

2011-01-20 Por tôpico Artur Costa Steiner
Basta demonstrar que, se a e b > 0 são racionais tais que raiz(b) é
irracional, então, se a + raiz(b) é raiz de P (P com coeficientes
racionais), então a - raiz(b) é também raiz de P. Sem perda de generalidade,
basta demonstrar para o caso a = 0. (Demonstrado para este caso, se a +
raiz(b) for raiz de P, então raiz(b) é raiz de Q(x) = P(x + a), cujos
coeficientes são racionais. Logo, -raiz(b) é raiz de Q, de modo que P(a -
raiz(b)) = Q(-raiz(b)) = 0).

Como b é racional e raiz(b) é irracional, então potências pares de raiz(b)
são racionais e potências ímpares são irracionais da forma b^k raiz(b), com
k inteiro >= 0. Sendo n o grau de P, se agruparmos as potências pares e
ímpares de raiz(b) (deixo para vc os detalhes), vamos obter algo do tipo

P(raiz(b)) = A + B raiz(b)
P(-raiz(b)) = A - B raiz(b)

Onde A e B são racionais. Como raiz(b) é irracional e P(raiz(b)) = 0, temos
necessariamente que A = B = 0, o que implica P(-raiz(b)) = 0.

Abraços
Artur


-Mensagem original-
De: owner-ob...@mat.puc-rio.br [mailto:owner-ob...@mat.puc-rio.br] Em nome
de Bernardo Freitas Paulo da Costa
Enviada em: quinta-feira, 20 de janeiro de 2011 07:58
Para: obm-l@mat.puc-rio.br
Assunto: [obm-l] Re: [obm-l] FW: Raízes irracionais

2011/1/20 marcone augusto araújo borges :
> Se um número como 3 + raíz(2),por exemplo, é raiz de uma equação do
segundo
> grau,então 3 - raíz(2) também é.
> Isso vale ,em geral,para uma equação de grau n?SE vale,como provar?

Bom, primeiro, você tem que acertar o enunciado... se a equação for

x^2 - 6 x + 11 + 2 raiz(2) x + 6 raiz(2) = (x - 3 - raiz(2))^2 = 0,
você não terá 3 - raiz(2)

Algo mais exato talvez seja "Se P(x) é um polinômio com coeficientes
inteiros / racionais, e se a + b*raiz(c) é uma raiz de P(x) = 0, com
a, b e c racionais, então a - b * raiz(c) também é raiz de P(x) = 0".

A melhor forma de provar isso é começar como você pensou na equação de
segundo grau. Você deve conseguir, usando a fórmula explícita da
solução, provar que as "raízes conjugadas sempre aparecem".

Para demonstrar mais geralmente, você talvez tenha que saber um pouco
de álgebra. Aliás, isso é bem parecido com o fato de "se uma raiz de
um polinômio com coeficientes reais for complexa, então tem outra raiz
complexa, a conjugada *complexa*", e vale a pena ver como funcionam
esses dois casos.

Abraços
-- 
Bernardo Freitas Paulo da Costa

=
Instruções para entrar na lista, sair da lista e usar a lista em
http://www.mat.puc-rio.br/~obmlistas/obm-l.html
=


=
Instruções para entrar na lista, sair da lista e usar a lista em
http://www.mat.puc-rio.br/~obmlistas/obm-l.html
=


[obm-l] Re: [obm-l] FW: Raízes irracionais

2011-01-20 Por tôpico Bernardo Freitas Paulo da Costa
2011/1/20 marcone augusto araújo borges :
> Se um número como 3 + raíz(2),por exemplo, é raiz de uma equação do segundo
> grau,então 3 - raíz(2) também é.
> Isso vale ,em geral,para uma equação de grau n?SE vale,como provar?

Bom, primeiro, você tem que acertar o enunciado... se a equação for

x^2 - 6 x + 11 + 2 raiz(2) x + 6 raiz(2) = (x - 3 - raiz(2))^2 = 0,
você não terá 3 - raiz(2)

Algo mais exato talvez seja "Se P(x) é um polinômio com coeficientes
inteiros / racionais, e se a + b*raiz(c) é uma raiz de P(x) = 0, com
a, b e c racionais, então a - b * raiz(c) também é raiz de P(x) = 0".

A melhor forma de provar isso é começar como você pensou na equação de
segundo grau. Você deve conseguir, usando a fórmula explícita da
solução, provar que as "raízes conjugadas sempre aparecem".

Para demonstrar mais geralmente, você talvez tenha que saber um pouco
de álgebra. Aliás, isso é bem parecido com o fato de "se uma raiz de
um polinômio com coeficientes reais for complexa, então tem outra raiz
complexa, a conjugada *complexa*", e vale a pena ver como funcionam
esses dois casos.

Abraços
-- 
Bernardo Freitas Paulo da Costa

=
Instruções para entrar na lista, sair da lista e usar a lista em
http://www.mat.puc-rio.br/~obmlistas/obm-l.html
=


RE: [obm-l] FW: Alguns problemas da prova da OBM (errata)

2010-09-22 Por tôpico Lucas Colucci

Por favor, não comentem online a prova até o dia 21. Há pessoas que só a farão 
na segunda-feira, dia 20.
Obrigado.

From: lucashagemais...@msn.com
To: obm-l@mat.puc-rio.br
Subject: RE: [obm-l] FW: Alguns problemas da prova da OBM (errata)
Date: Sat, 18 Sep 2010 22:13:46 -0300









 Correção: é

x² - (r+s)x + rs + 2010 = 0

 

Abraço!

From: joao_maldona...@hotmail.com
To: obm-l@mat.puc-rio.br
Subject: [obm-l] FW: Alguns problemas da prova da OBM (errata)
Date: Sat, 18 Sep 2010 21:56:40 -0300








From: joao_maldona...@hotmail.com
To: obm-l@mat.puc-rio.br
Subject: Alguns problemas da prova da OBM (errata)
Date: Sat, 18 Sep 2010 18:53:21 -0300



No ex: 1 é para N maior de 0

Lembrei do ex: 2

Na seguinte equação:

x² + (r+s)x + rs - 2010 = 0

Para x, r e s inteiros, a quantidade de valores de |r-s| é.

Abraço :D

  

RE: [obm-l] FW: Alguns problemas da prova da OBM (errata)

2010-09-18 Por tôpico Lucas Hagemaister


 Correção: é

x² - (r+s)x + rs + 2010 = 0

 

Abraço!

From: joao_maldona...@hotmail.com
To: obm-l@mat.puc-rio.br
Subject: [obm-l] FW: Alguns problemas da prova da OBM (errata)
Date: Sat, 18 Sep 2010 21:56:40 -0300








From: joao_maldona...@hotmail.com
To: obm-l@mat.puc-rio.br
Subject: Alguns problemas da prova da OBM (errata)
Date: Sat, 18 Sep 2010 18:53:21 -0300



No ex: 1 é para N maior de 0

Lembrei do ex: 2

Na seguinte equação:

x² + (r+s)x + rs - 2010 = 0

Para x, r e s inteiros, a quantidade de valores de |r-s| é.

Abraço :D

  

Re: [obm-l] FW: Nosso calendario

2010-08-29 Por tôpico Marcelo Salhab Brogliato
Sim, apesar de ser imediato.
Pois fevereiro ganha 1 dia.. logo, basta somar 1 nos devidos locais e ver
que ainda
temos todos os resíduos módulo 7.

abraços,
Salhab


2010/8/29 Hugo Fernando Marques Fernandes 

> Não faltou considerar os anos bissextos?
>
> Abraços.
>
> Hugo.
>
> Em 29 de agosto de 2010 00:26, marcone augusto araújo borges <
> marconeborge...@hotmail.com> escreveu:
>
>  Obrigado,abraços.
>>
>> --
>> Date: Sat, 28 Aug 2010 23:41:47 -0300
>> Subject: Re: [obm-l] FW: Nosso calendario
>> From: msbro...@gmail.com
>>
>> To: obm-l@mat.puc-rio.br
>>
>> Vamos ver a qtde de dias de cada mês, em ordem:
>> 31, 28, 31, 30, 31, 30, 31, 31, 30, 31, 30, 31
>>
>> Analisando isso módulo 7, visto que são 7 dias da semana, temos:
>> 28 == 0 (mod 7)
>> 30 == 2 (mod 7)
>> 31 == 3 (mod 7)
>>
>> Desta maneira, temos:
>> 3, 0, 3, 2, 3, 2, 3, 3, 2, 3, 2, 3
>>
>> Supondo que o primeiro dia 13 esteja em k, temos:
>> k, k+3, k+3, k+6, k+8, k+11, k+13, k+16, k+19, k+21, k+24, k+26
>>
>> Analisando mod 7, temos:
>> k, k+3, k+3, k+6, k+1, k+4, k+6, k+2, k+5, k, k+3, k+5
>>
>> Veja que temos todos os inteiros mod 7 somando com k.
>> Desta maneira, sempre há uma sexta feira 13.
>>
>> abraços,
>> Salhab
>>
>>
>>
>> 2010/8/28 marcone augusto araújo borges 
>>
>>
>>
>> --
>> From: marconeborge...@hotmail.com
>> To: obm-l@mat.puc-rio.br
>> Subject: Nosso calendario
>> Date: Sun, 29 Aug 2010 02:12:05 +
>>
>> Mostre q em qualquer ano existe pelo menos uma sexta-feira 13.Eu acho q
>> consegui resolver,mas gostaria de ver outra solução.
>> Fiz assim:se o dia 13 de janeiro  é um domingo,entao o dia 13 de setembro
>> é uma sexta pois,contando apenas o numero
>> de dias q passam de 28 em cada mes,a partir de janeiro(até
>> agosto),encontramos 19 dias(um multiplo de 7 mais 5),dai,conside-
>> rando o domingo como dia 1,temos 1+5=6(sexta).Usei o mesmo raciocinio para
>> o caso do dia 13 de janeiro ser segunda,terça,quarta,quinta ou sabado e
>> encontrei para cada caso uma sexta feira 13 no mesmo ano.
>>
>>
>>
>


Re: [obm-l] FW: Nosso calendario

2010-08-28 Por tôpico Hugo Fernando Marques Fernandes
Não faltou considerar os anos bissextos?

Abraços.

Hugo.

Em 29 de agosto de 2010 00:26, marcone augusto araújo borges <
marconeborge...@hotmail.com> escreveu:

>  Obrigado,abraços.
>
> --
> Date: Sat, 28 Aug 2010 23:41:47 -0300
> Subject: Re: [obm-l] FW: Nosso calendario
> From: msbro...@gmail.com
>
> To: obm-l@mat.puc-rio.br
>
> Vamos ver a qtde de dias de cada mês, em ordem:
> 31, 28, 31, 30, 31, 30, 31, 31, 30, 31, 30, 31
>
> Analisando isso módulo 7, visto que são 7 dias da semana, temos:
> 28 == 0 (mod 7)
> 30 == 2 (mod 7)
> 31 == 3 (mod 7)
>
> Desta maneira, temos:
> 3, 0, 3, 2, 3, 2, 3, 3, 2, 3, 2, 3
>
> Supondo que o primeiro dia 13 esteja em k, temos:
> k, k+3, k+3, k+6, k+8, k+11, k+13, k+16, k+19, k+21, k+24, k+26
>
> Analisando mod 7, temos:
> k, k+3, k+3, k+6, k+1, k+4, k+6, k+2, k+5, k, k+3, k+5
>
> Veja que temos todos os inteiros mod 7 somando com k.
> Desta maneira, sempre há uma sexta feira 13.
>
> abraços,
> Salhab
>
>
>
> 2010/8/28 marcone augusto araújo borges 
>
>
>
> --
> From: marconeborge...@hotmail.com
> To: obm-l@mat.puc-rio.br
> Subject: Nosso calendario
> Date: Sun, 29 Aug 2010 02:12:05 +
>
> Mostre q em qualquer ano existe pelo menos uma sexta-feira 13.Eu acho q
> consegui resolver,mas gostaria de ver outra solução.
> Fiz assim:se o dia 13 de janeiro  é um domingo,entao o dia 13 de setembro é
> uma sexta pois,contando apenas o numero
> de dias q passam de 28 em cada mes,a partir de janeiro(até
> agosto),encontramos 19 dias(um multiplo de 7 mais 5),dai,conside-
> rando o domingo como dia 1,temos 1+5=6(sexta).Usei o mesmo raciocinio para
> o caso do dia 13 de janeiro ser segunda,terça,quarta,quinta ou sabado e
> encontrei para cada caso uma sexta feira 13 no mesmo ano.
>
>
>


RE: [obm-l] FW: Nosso calendario

2010-08-28 Por tôpico marcone augusto araújo borges

Obrigado,abraços.
 


Date: Sat, 28 Aug 2010 23:41:47 -0300
Subject: Re: [obm-l] FW: Nosso calendario
From: msbro...@gmail.com
To: obm-l@mat.puc-rio.br


Vamos ver a qtde de dias de cada mês, em ordem:
31, 28, 31, 30, 31, 30, 31, 31, 30, 31, 30, 31


Analisando isso módulo 7, visto que são 7 dias da semana, temos:
28 == 0 (mod 7)
30 == 2 (mod 7)
31 == 3 (mod 7)


Desta maneira, temos:
3, 0, 3, 2, 3, 2, 3, 3, 2, 3, 2, 3


Supondo que o primeiro dia 13 esteja em k, temos:
k, k+3, k+3, k+6, k+8, k+11, k+13, k+16, k+19, k+21, k+24, k+26


Analisando mod 7, temos:
k, k+3, k+3, k+6, k+1, k+4, k+6, k+2, k+5, k, k+3, k+5


Veja que temos todos os inteiros mod 7 somando com k.
Desta maneira, sempre há uma sexta feira 13.


abraços,
Salhab






2010/8/28 marcone augusto araújo borges 



 


From: marconeborge...@hotmail.com
To: obm-l@mat.puc-rio.br
Subject: Nosso calendario
Date: Sun, 29 Aug 2010 02:12:05 +

Mostre q em qualquer ano existe pelo menos uma sexta-feira 13.Eu acho q 
consegui resolver,mas gostaria de ver outra solução.
Fiz assim:se o dia 13 de janeiro  é um domingo,entao o dia 13 de setembro é uma 
sexta pois,contando apenas o numero 
de dias q passam de 28 em cada mes,a partir de janeiro(até agosto),encontramos 
19 dias(um multiplo de 7 mais 5),dai,conside-
rando o domingo como dia 1,temos 1+5=6(sexta).Usei o mesmo raciocinio para o 
caso do dia 13 de janeiro ser segunda,terça,quarta,quinta ou sabado e encontrei 
para cada caso uma sexta feira 13 no mesmo ano.

  

Re: [obm-l] FW: Nosso calendario

2010-08-28 Por tôpico Marcelo Salhab Brogliato
Vamos ver a qtde de dias de cada mês, em ordem:
31, 28, 31, 30, 31, 30, 31, 31, 30, 31, 30, 31

Analisando isso módulo 7, visto que são 7 dias da semana, temos:
28 == 0 (mod 7)
30 == 2 (mod 7)
31 == 3 (mod 7)

Desta maneira, temos:
3, 0, 3, 2, 3, 2, 3, 3, 2, 3, 2, 3

Supondo que o primeiro dia 13 esteja em k, temos:
k, k+3, k+3, k+6, k+8, k+11, k+13, k+16, k+19, k+21, k+24, k+26

Analisando mod 7, temos:
k, k+3, k+3, k+6, k+1, k+4, k+6, k+2, k+5, k, k+3, k+5

Veja que temos todos os inteiros mod 7 somando com k.
Desta maneira, sempre há uma sexta feira 13.

abraços,
Salhab



2010/8/28 marcone augusto araújo borges 

>
>
> --
> From: marconeborge...@hotmail.com
> To: obm-l@mat.puc-rio.br
> Subject: Nosso calendario
> Date: Sun, 29 Aug 2010 02:12:05 +
>
> Mostre q em qualquer ano existe pelo menos uma sexta-feira 13.Eu acho q
> consegui resolver,mas gostaria de ver outra solução.
> Fiz assim:se o dia 13 de janeiro  é um domingo,entao o dia 13 de setembro é
> uma sexta pois,contando apenas o numero
> de dias q passam de 28 em cada mes,a partir de janeiro(até
> agosto),encontramos 19 dias(um multiplo de 7 mais 5),dai,conside-
> rando o domingo como dia 1,temos 1+5=6(sexta).Usei o mesmo raciocinio para
> o caso do dia 13 de janeiro ser segunda,terça,quarta,quinta ou sabado e
> encontrei para cada caso uma sexta feira 13 no mesmo ano.
>


[obm-l] RE: [obm-l] Re: [obm -l] Re: [obm-l] FW: [obm-l] Números inte iros

2010-04-23 Por tôpico marcone augusto araújo borges

Nem sei o q dizer diante de tanta satisfação.Para o Adalberto e para o 
Ralph,obrigado!
 


Date: Fri, 23 Apr 2010 12:50:23 -0300
Subject: [obm-l] Re: [obm-l] Re: [obm-l] FW: [obm-l] Números inteiros
From: ralp...@gmail.com
To: obm-l@mat.puc-rio.br


Achei uma maneira meio "complicada" de fazer o problema:
 
Somar 1 nao altera a "integridade" (:P) do numero:
 
(n^3+1)/(mn-1) +1 = (n^3+mn)/(mn-1) = n (n^2+m)/(mn-1)
 
Mas n e mn-1 sao primos entre si. Entao isto eh inteiro sse
(a) (n^2+m)/(mn-1) for inteiro, sse
(b) m(n^2+m)/(mn-1) inteiro (lembra que m e mn-1 sao primos entre si), sse
(c) m(n^2+m)/(mn-1) -n = (m^2+n)/(mn-1) for inteiro.
 
Olhando as linhas (a) e (c) acima, vemos que (m,n) "serve" se e somente se 
(n,m) serve tambem. Assim, daqui para a frente, vou procurar apenas as solucoes 
com m>=n (as outras vao ser obtidas invertendo a ordem).
 
Entao olhe para (n^2+m)/(mn-1), agora pensando que m>=n. Voce nao tem a 
sensacao de que o denominador eh grande demais? Afinal, eh m.n versus n^2, os 
outros termos sendo "de ordem menor"...
 
Formalmente, tomando m=n+c (com c>=0), temos:
 
D=mn-1=n^2+cn-1
N=n^2+m=n^2+n+c
D-N=cn-n-c-1=(c-1)(n-1)-2
 
Lembre que n>=1 e c>=0! Assim, em geral, temos D>N>0 e N/D nao poderah ser 
inteiro. Bom, as unicas excecoes seriam os casos em que (c-1)(n-1)<=2, isto eh:
 
CASO 1: n=1
CASO 2: n=2 e c=0,1,2,3
CASO 3: n=3 e c=0,1,2
 
Estes casos sao bem mais simples, e geram as solucoes que o Adalberto jah tinha 
achado. Nao ha outras (lembrando que para cada solucao (a,b) destes casos 
existe a solucao (b,a) correspondente).

Abraco, Ralph.
 
2010/4/23 Adalberto Dornelles 


Oi Marco,

> Alguem poderia ajudar com ideias para a resolução da questão:Determinar
> todos os pares de inteiros positivos(m,n) tais que (n^3+1)/(mn-1) seja um
> inteiro?Ate agora eu observei apenas que  m=n=2 satisfaz e os pares (2,1) e
> (1,2), tambem.Agradeço antecipadamente.

Achei (por força bruta) os seguintes pares com m,n < 1000
a = (n^3+1)/(mn-1)

   mn a
1 2 9
1 314
2 1 2
2 2 3
2 514
3 1 1
3 5 9
5 2 1
5 3 2

Observe que m e n são primos. Talvez isso ajude na resolução...

Abraço,
Adalberto 
_
O seu navegador também te ajuda a ficar longe de vírus. Leia mais sobre 
segurança.
http://www.microsoft.com/brasil/windows/internet-explorer/?WT.mc_id=1500

[obm-l] Re: [obm-l] Re: [obm-l] FW: [obm-l] Números inteiro s

2010-04-23 Por tôpico Ralph Teixeira
Achei uma maneira meio "complicada" de fazer o problema:

Somar 1 nao altera a "integridade" (:P) do numero:

(n^3+1)/(mn-1) +1 = (n^3+mn)/(mn-1) = n (n^2+m)/(mn-1)

Mas n e mn-1 sao primos entre si. Entao isto eh inteiro sse
(a) (n^2+m)/(mn-1) for inteiro, sse
(b) m(n^2+m)/(mn-1) inteiro (lembra que m e mn-1 sao primos entre si), sse
(c) m(n^2+m)/(mn-1) -n = (m^2+n)/(mn-1) for inteiro.

Olhando as linhas (a) e (c) acima, vemos que (m,n) "serve" se e somente se
(n,m) serve tambem. Assim, daqui para a frente, vou procurar apenas as
solucoes com m>=n (as outras vao ser obtidas invertendo a ordem).

Entao olhe para (n^2+m)/(mn-1), agora pensando que m>=n. Voce nao tem a
sensacao de que o denominador eh grande demais? Afinal, eh m.n versus n^2,
os outros termos sendo "de ordem menor"...

Formalmente, tomando m=n+c (com c>=0), temos:

D=mn-1=n^2+cn-1
N=n^2+m=n^2+n+c
D-N=cn-n-c-1=(c-1)(n-1)-2

Lembre que n>=1 e c>=0! Assim, em geral, temos D>N>0 e N/D nao poderah ser
inteiro. Bom, as unicas excecoes seriam os casos em que (c-1)(n-1)<=2, isto
eh:

CASO 1: n=1
CASO 2: n=2 e c=0,1,2,3
CASO 3: n=3 e c=0,1,2

Estes casos sao bem mais simples, e geram as solucoes que o Adalberto jah
tinha achado. Nao ha outras (lembrando que para cada solucao (a,b) destes
casos existe a solucao (b,a) correspondente).

Abraco, Ralph.

2010/4/23 Adalberto Dornelles 

> Oi Marco,
>
> > Alguem poderia ajudar com ideias para a resolução da questão:Determinar
> > todos os pares de inteiros positivos(m,n) tais que (n^3+1)/(mn-1) seja um
> > inteiro?Ate agora eu observei apenas que  m=n=2 satisfaz e os pares (2,1)
> e
> > (1,2), tambem.Agradeço antecipadamente.
>
> Achei (por força bruta) os seguintes pares com m,n < 1000
> a = (n^3+1)/(mn-1)
>
>mn a
> 1 2 9
> 1 314
> 2 1 2
> 2 2 3
> 2 514
> 3 1 1
> 3 5 9
> 5 2 1
> 5 3 2
>
> Observe que m e n são primos. Talvez isso ajude na resolução...
>
> Abraço,
> Adalberto


[obm-l] Re: [obm-l] FW: [obm-l] Números inteiros

2010-04-23 Por tôpico Adalberto Dornelles
Oi Marco,

> Alguem poderia ajudar com ideias para a resolução da questão:Determinar
> todos os pares de inteiros positivos(m,n) tais que (n^3+1)/(mn-1) seja um
> inteiro?Ate agora eu observei apenas que  m=n=2 satisfaz e os pares (2,1) e
> (1,2), tambem.Agradeço antecipadamente.

Achei (por força bruta) os seguintes pares com m,n < 1000
a = (n^3+1)/(mn-1)

mn a
 1 2 9
 1 314
 2 1 2
 2 2 3
 2 514
 3 1 1
 3 5 9
 5 2 1
 5 3 2

Observe que m e n são primos. Talvez isso ajude na resolução...

Abraço,
Adalberto


> 
> Transforme-se em personagens engraçados e coloque no Messenger. Clique e
> veja como.
> 
> Quer transformar suas fotos em emoticons para o Messenger? Clique aqui e
> veja como.

=
Instruções para entrar na lista, sair da lista e usar a lista em
http://www.mat.puc-rio.br/~obmlistas/obm-l.html
=


[obm-l] Re: [obm-l] FW: PASSATEMPOS MATEMÁTICOS!

2009-11-17 Por tôpico Rogerio Ponce
Olá Jorge e colegas da lista,

Sejam A,B e C os amigos, e consideremos que a garrafa sera' distribuida em 3
copos.
A serve o primeiro copo.
B escolhe entre entregar o copo a A e pegar a garrafa, ou pegar o copo e
deixar a garrafa com A.
Em seguida, quem estiver com a garrafa serve os outros dois copos.
Entao, C escolhe um deles para si mesmo, deixando o outro copo para quem
estiver com a garrafa.

[]'s
Rogerio Ponce.


2009/11/15 Jorge Luis Rodrigues e Silva Luis 

>
>
> --
> From: jorgelrs1...@hotmail.com
> To: obm-l@mat.puc-rio.br
> Subject: PASSATEMPOS MATEMÁTICOS!
> Date: Sun, 15 Nov 2009 00:52:43 +
>
> Dois franceses estavam prestes a dividir uma garrafa de champanhe, quando
> chegou um terceiro amigo. Como homens educados, eles sabiam que para dividir
> irmamente o champanhe entre duas pessoas, de modo que ambas considerem a
> divisão justa, era uma delas repartir o conteúdo em duas partes que
> considere iguais e a outra pessoa escolher uma destas partes. Mas com a
> chegada do terceiro amigo, como proceder?
>
>


[obm-l] Re: [obm-l] FW: FÍSICA DIVERTIDA E CURIOSA!

2009-10-11 Por tôpico Bruno França dos Reis
Esse seu "enigma da barcaça" é muito interessante mesmo.

Aqui vai uma citação dele, conforme mensagem do Jorge há quase 4 anos atrás:


"Um barquinho flutua numa piscina; dentro dele estão uma pessoa e uma pedra.

A pessoa joga a pedra dentro da piscina. O nível da água na piscina sobe,
desce ou não se altera? E se ao invés da pedra fosse um cubo de gelo que
derretesse sòmente após ser jogado dentro da piscina, como se comportaria o
nível da água? (Essa é ótima!) "


--
Bruno FRANÇA DOS REIS

msn: brunoreis...@hotmail.com
skype: brunoreis666
tel: +33 (0)6 28 43 42 16

http://brunoreis.com
http://blog.brunoreis.com

GPG Key: http://brunoreis.com/bruno-public.key

e^(pi*i)+1=0


2009/10/12 Jorge Luis Rodrigues e Silva Luis 

>
>
> --
> From: jorgelrs1...@hotmail.com
> To: obm-l@mat.puc-rio.br
> Subject: FÍSICA DIVERTIDA E CURIOSA!
> Date: Sat, 10 Oct 2009 19:02:05 +
>
> Turma! Em caráter excepcional, devido ao clima de feriado prolongado, vamos
> nos divertir um pouco, já que nem só de integrais e derivadas se ocupa um
> bom matemático...
>
> Dois atletas correm e andam na chuva a mesma velocidade, quem se molha
> mais; será o que corre a metade da distância e anda a outra metade ou o que
> corre a metade do tempo e anda o restante?
>
> Quando descemos num elevador, nosso peso diminui? E se tivermos subindo ou
> descendo em cima de uma balança, ela vai acusar pesos diferentes?
>
> Um mesmo navio descarregado e carregado. Qual deles flutua mais baixo na
> água? Como o peso de sua carga se compara ao peso extra de água deslocada?
>
> Se o carro supersônico Thrust SSC corresse a 1220 Km/h por 15 minutos, em
> quantos quilômetros ficaria a frente do seu ruido?
>
> Um macaco está pendurado na extremidade de uma corda de 6m que passa por
> uma roldana e é equilibrada por um peso amarrado na outra extremidade. Para
> que o peso suba até atingir a roldana, quantos metros o macaco terá que
> subir na corda?
>
> Uma caixa em que ardeu uma vela pesa o mesmo que a caixa e a vela antes de
> esta ter ardido?
>
> Em um país foi medido a temperatura de zero grau centígrado. No dia
> seguinte essa temperatura caiu pela metade. Como isso é possível? Qual a
> nova temperatura? (Campeão!)
>
> Vocês sabiam! que a vazão em um cano de 2 polegadas é, aproximadamente, 6,4
> vezes maior que em um cano de 1 polegada. Incrível, não! A influência do
> atrito neste processo é algo fantástico! Agora, mais incrível ainda é
> sentirmos fria a água a ferver quando metemos as mãos dentro dela.
> Experimentem!
>
> Gostaria de convidar os simpatizantes destes pueris assuntos a revisarem o
> "Enigma da Barcaça".
>
>
> Divirtam-se!
>
> --
> Novo Internet Explorer 8: traduza com apenas um clique. Baixe agora, é
> grátis!
> --
> Com o Novo Internet Explorer 8 suas abas se organizam por cor. Baixe
> agora, é 
> grátis!
>


[obm-l] Re: [obm-l] FW: ANÁLIS E COMBINAT ÓRIA!

2009-06-29 Por tôpico Luciana Rodrigues
 
Carpe Dien
Em 29/06/2009 11:57, Jorge Luis Rodrigues e Silva Luis < jorgelrs1...@hotmail.com > escreveu:

.hmmessage P { margin:0px; padding:0px } body.hmmessage { font-size: 10pt; font-family:Verdana }
 

From: jorgelrs1...@hotmail.comTo: obm-l@mat.puc-rio.brSubject: ANÁLISE COMBINATÓRIA!Date: Mon, 29 Jun 2009 13:45:02 +
.ExternalClass .EC_hmmessage P {padding:0px;} .ExternalClass body.EC_hmmessage {font-size:10pt;font-family:Verdana;}
Olá, Pessoal! Um exame consta de 4 provas. Os graus em cada matéria variam de 0 a 10, aproximados até décimos. Qual o número mínimo de candidatos que nos permitirá afirmar a existência de dois que tenham obtido notas idênticas? Quantos milhares sem algarismos repetidos podem ser formados com 2 algarismos pares e 2 ímpares significativos? Em quantas permutações dos algarismos 1, 2, 3, 4, 5 e 6 os equidistantes dos extremos somam 7? Quantos diferentes colares usando 13 pedras distintas podem ser feitos se virar o colar ao invés de rodar? Qual o número de maneiras que podemos colocar quatro bolas indistingüíveis em seis compartimentos separados? A propósito, quantos números tem todos os seus dígitos de igual paridade? Afinal! Qual o maior número  de interseções de 5 circunferências?  Abraços!

Novo Internet Explorer 8: mais rápido e muito mais seguro. Baixe agora, é grátis!

Instale o novo Internet Explorer 8 otimizado para o MSN. Download aqui 
=
Instruções para entrar na lista, sair da lista e usar a lista em
http://www.mat.puc-rio.br/~obmlistas/obm-l.html
=


[obm-l] Re: [obm-l] FW: TERRA DOS MATEMÁTICOS!

2009-05-20 Por tôpico lucianarodriggues

Em 20/05/2009 09:11, Paulo Santa Rita < paulo.santar...@gmail.com > escreveu:Ola Jorge e demais colegasdesta lista ... OBM-L,Voce gostou das "Investigacoes Aritmeticas" ? Fico feliz e obrigadopelo elogio. Em verdade esta mensagem e a exposicao de estudos que eufiz quando ainda era muito jovem, crianca ainda. E apenas uma parte deum estudo mais amplo. Na epoca o meu objetivo era encontrar as colunasocultas ( ou faces ocultas ) do traingulo de Pascal. Hoje eu sei comfazer isso. Inclusive ja publiquei aqui algumas investigacoes nestesentido.Se voce verificar minhas primeiras mensagens para esta lista vai notarque la eu digo que "havia descoberto coisas que nao estao nos livros".Na verdade foram muitas coisas, pois sempre e naturalmente gostei depensar. Acho que e
  natural que todo jovem disciplinado e dedicado, querealmente gosta de Matematica faca (re)descobertas de fatos que osmatematicos do passado ja fizeram. Por exemplo, o Gugu redescobriu umtipo de solucao para equacoes do 3 grau ja descoberto pelo Euler e oNicolau ja disse aqui que redescobriu o algoritmo do calculo de raizesquadradas. Acho que isso e natural e esperavel, nao signifcando nadaalem disso !Seria possivel dizer o menor N tal que 1 + 1/2 + 1/3 + ... + 1/N > P,para um P dado, sem usaraproximacoes com a constante de Euler Macheroni ? Era isso que eu queria saber.Nao entendi a passagem abaixo :> Agora, quanto à série dos inversos dos primos...A Série Harmônica é um caso> patológico de divergência. Se você somar os inversos dos naturais elevados a> qualquer potência maior do que 1, a soma será convergente. Se for 1 ou menor> do que 1 será divergente. Então, não existe um "
 menor" expoente r para o> qual a soma dos inversos dos naturais elevados a r seja convergente. Como os> primos são um subconjunto dos naturais, também não existe um "menor"> expoente para o qual a soma dos inversos dos primos elevados a r seja> convergente. Qualquer r maior do que 1 basta. O mesmo Euler provou, em 1736,> que a soma dos inversos dos primos é divergente.Eu me referi a soma das r-esimas potencias dos inversos dos primos.Como a soma dos inversos dos primos e divergente entao, com certeza,existe um r >1 tal quea serie :1 + (1/2) ^r + (1/3)^r + (1/5)^r + (1/7)^r + ... + (1/P) ^r +  ...converge. Qual o menor "r" que atende esta condicao ? Euler mostrouque r=1 nao serve, pois ele provou que a soma dos inversos dos primose divergente. Assim, r > 1. Qual o menor r ? Sera alguma dasconstantes que conhecemos ? Sera um novo numero irracional importante?Bem falastes ! A serie har
 monica !Eu nao me canso de admira-la ! Ela e "altamente" sensivel. Vocecolocou um expoente "um pouquinho" maior que 1 em seus termos, elaconverge. Se mudar o sinal de + para - dos termos cujos denominadoresformam uma PA, ela converge. De alguma forma ele deve servir como umaespecie de "medida" ou termometro de convergencia, mas eu nao atineicomo fazer isso. Eu apreciaria muito se alguem pudesse falar algo arespeito.Um Abraco a Todos !PSR,4200509090B2009/5/19 Jorge Luis Rodrigues e Silva Luis :>>> > From: jorgelrs1...@hotmail.com> To: obm-l@mat.puc-rio.br> Subject: TERRA DOS MATEMÁTICOS!> Date: Tue, 19 May 2009 15:36:35 +>> Ok! Nehab, bom progresso para quem já foi denominada de "Terra dos> Humoristas". Não é à toa que o autor da mais engenhosa distribuição das 3> barras
  de chocolate entre quatro crianças é um Cearense, aluno do curso de> licenciatura em matemática-UECE. Foi também o pioneiro a discordar da> afirmação do colega Takiyama "1/x*x#x*1/x" na calculadora do> feirante...Experimentem com seus pupilos a pueril situação: Entre as frações> 1/5 e 1/3 temos 16 divisões iguais. Em qual das divisões se encontra a> fração 1/4?>> Grande Paulo! Parabéns pela enquete "Investigações Aritméticas", pois me> passou despercebida, na época. Uma verdadeira pérola.Campeão!>> Quanto à questão do menor N tal que 1+(1/2)+...+(1/N)>P, Euler demonstrou> que a soma dos termos da Série Harmônica, para N tendendo ao infinito, é> lnN+0,5772..., ou seja para atingir um inteiro P razoavelmente grande basta> fazer lnN=P-0,5772... onde N é (2,718281828...) elevado a P-0,5772... Esse> caminho permite obter uma ordem de grandeza bastante boa, mas para saber> exatam
 ente o menor N, teremos que trabalhar com muitas, mas muitas casas> decimais.>> Agora, quanto à série dos inversos dos primos...A Série Harmônica é um caso> patológico de divergência. Se você somar os inversos dos naturais elevados a> qualquer potência maior do que 1, a soma será convergente. Se for 1 ou menor> do que 1 será divergente. Então, não existe um "menor" expoente r para o> qual a soma dos inversos dos naturais elevados a r seja convergente. Como os> primos são um subconjunto dos naturais, também não existe um "menor"> expoente para o qual a soma dos inversos dos primos elevados a r seja> convergente. Qualquer r maior do que 1 basta. O mesm

[obm-l] Re: [obm-l] FW: TERRA DOS MATEMÁTICOS!

2009-05-20 Por tôpico Paulo Santa Rita
Ola Jorge e demais colegas
desta lista ... OBM-L,

Voce gostou das "Investigacoes Aritmeticas" ? Fico feliz e obrigado
pelo elogio. Em verdade esta mensagem e a exposicao de estudos que eu
fiz quando ainda era muito jovem, crianca ainda. E apenas uma parte de
um estudo mais amplo. Na epoca o meu objetivo era encontrar as colunas
ocultas ( ou faces ocultas ) do traingulo de Pascal. Hoje eu sei com
fazer isso. Inclusive ja publiquei aqui algumas investigacoes neste
sentido.

Se voce verificar minhas primeiras mensagens para esta lista vai notar
que la eu digo que "havia descoberto coisas que nao estao nos livros".
Na verdade foram muitas coisas, pois sempre e naturalmente gostei de
pensar. Acho que e natural que todo jovem disciplinado e dedicado, que
realmente gosta de Matematica faca (re)descobertas de fatos que os
matematicos do passado ja fizeram. Por exemplo, o Gugu redescobriu um
tipo de solucao para equacoes do 3 grau ja descoberto pelo Euler e o
Nicolau ja disse aqui que redescobriu o algoritmo do calculo de raizes
quadradas. Acho que isso e natural e esperavel, nao signifcando nada
alem disso !

Seria possivel dizer o menor N tal que 1 + 1/2 + 1/3 + ... + 1/N > P,
para um P dado, sem usar
aproximacoes com a constante de Euler Macheroni ? Era isso que eu queria saber.

Nao entendi a passagem abaixo :

> Agora, quanto à série dos inversos dos primos...A Série Harmônica é um caso
> patológico de divergência. Se você somar os inversos dos naturais elevados a
> qualquer potência maior do que 1, a soma será convergente. Se for 1 ou menor
> do que 1 será divergente. Então, não existe um "menor" expoente r para o
> qual a soma dos inversos dos naturais elevados a r seja convergente. Como os
> primos são um subconjunto dos naturais, também não existe um "menor"
> expoente para o qual a soma dos inversos dos primos elevados a r seja
> convergente. Qualquer r maior do que 1 basta. O mesmo Euler provou, em 1736,
> que a soma dos inversos dos primos é divergente.

Eu me referi a soma das r-esimas potencias dos inversos dos primos.
Como a soma dos inversos dos primos e divergente entao, com certeza,
existe um r >1 tal que
a serie :
1 + (1/2) ^r + (1/3)^r + (1/5)^r + (1/7)^r + ... + (1/P) ^r +  ...
converge. Qual o menor "r" que atende esta condicao ? Euler mostrou
que r=1 nao serve, pois ele provou que a soma dos inversos dos primos
e divergente. Assim, r > 1. Qual o menor r ? Sera alguma das
constantes que conhecemos ? Sera um novo numero irracional importante
?

Bem falastes ! A serie harmonica !

Eu nao me canso de admira-la ! Ela e "altamente" sensivel. Voce
colocou um expoente "um pouquinho" maior que 1 em seus termos, ela
converge. Se mudar o sinal de + para - dos termos cujos denominadores
formam uma PA, ela converge. De alguma forma ele deve servir como uma
especie de "medida" ou termometro de convergencia, mas eu nao atinei
como fazer isso. Eu apreciaria muito se alguem pudesse falar algo a
respeito.

Um Abraco a Todos !
PSR,4200509090B





2009/5/19 Jorge Luis Rodrigues e Silva Luis :
>
>
> 
> From: jorgelrs1...@hotmail.com
> To: obm-l@mat.puc-rio.br
> Subject: TERRA DOS MATEMÁTICOS!
> Date: Tue, 19 May 2009 15:36:35 +
>
> Ok! Nehab, bom progresso para quem já foi denominada de "Terra dos
> Humoristas". Não é à toa que o autor da mais engenhosa distribuição das 3
> barras de chocolate entre quatro crianças é um Cearense, aluno do curso de
> licenciatura em matemática-UECE. Foi também o pioneiro a discordar da
> afirmação do colega Takiyama "1/x*x#x*1/x" na calculadora do
> feirante...Experimentem com seus pupilos a pueril situação: Entre as frações
> 1/5 e 1/3 temos 16 divisões iguais. Em qual das divisões se encontra a
> fração 1/4?
>
> Grande Paulo! Parabéns pela enquete "Investigações Aritméticas", pois me
> passou despercebida, na época. Uma verdadeira pérola.Campeão!
>
> Quanto à questão do menor N tal que 1+(1/2)+...+(1/N)>P, Euler demonstrou
> que a soma dos termos da Série Harmônica, para N tendendo ao infinito, é
> lnN+0,5772..., ou seja para atingir um inteiro P razoavelmente grande basta
> fazer lnN=P-0,5772... onde N é (2,718281828...) elevado a P-0,5772... Esse
> caminho permite obter uma ordem de grandeza bastante boa, mas para saber
> exatamente o menor N, teremos que trabalhar com muitas, mas muitas casas
> decimais.
>
> Agora, quanto à série dos inversos dos primos...A Série Harmônica é um caso
> patológico de divergência. Se você somar os inversos dos naturais elevados a
> qualquer potência maior do que 1, a soma será convergente. Se for 1 ou menor
> do que 1 será divergente. Então, não existe um "menor" expoente r para o
> qual a soma dos inversos dos naturais elevados a r seja convergente. Como os
> primos são um subconjunto dos naturais, também não existe um "menor"
> expoente para o qual a soma dos inversos dos primos elevados a r seja
> convergente. Qualquer r maior do que 1 basta. O mesmo Euler provou, em 1736,
> que a soma dos inversos do

[obm-l] Re: [obm-l] FW: TERRA DOS MATEMÁ TICOS!

2009-05-19 Por tôpico lucianarodriggues

Em 19/05/2009 21:00, Jorge Luis Rodrigues e Silva Luis < jorgelrs1...@hotmail.com > escreveu:


.hmmessage P
{
margin:0px;
padding:0px
}
body.hmmessage
{
font-size: 10pt;
font-family:Verdana
}



From: jorgelrs1...@hotmail.comTo: obm-l@mat.puc-rio.brSubject: TERRA DOS MATEMÁTICOS!Date: Tue, 19 May 2009 15:36:35 +




.ExternalClass .EC_hmmessage P
{padding:0px;}
.ExternalClass body.EC_hmmessage
{font-size:10pt;font-family:Verdana;}



Ok! Nehab, bom progresso para quem já foi denominada de "Terra dos Humoristas". Não é à toa que o autor da mais engenhosa distribuição das 3 barras de chocolate entre quatro crianças é um Cearense, aluno do curso de licenciatura em matemática-UECE. Foi também o pioneiro a discordar da afirmação do colega Takiyama "1/x*x#x*1/x" na calculadora do feirante...Experimentem com seus pupilos a pueril situação: Entre as frações 1/5 e 1/3 temos 16 divisões iguais. Em qual das divisões se encontra a fração 1/4?Grande Paulo! Parabéns pela enquete "Investigações Aritméticas", pois me passou despercebida, na época. Uma verdadeira pérola.Campeão!Quanto à questão do menor N tal que 1+(1/2)+...+(1/N)>P, Euler demonstrou que a soma dos termos da Série Harmônica, para N tendendo ao infinito, é lnN+0,5772..., ou seja para atingir um inteiro P razoavelmente grande basta fazer lnN=P-0,5772... onde N é (2,718281828...) elevado a P-0,5772... 
 Esse caminho permite obter uma ordem de grandeza bastante boa, mas para saber exatamente o menor N, teremos que trabalhar com muitas, mas muitas casas decimais.Agora, quanto à série dos inversos dos primos...A Série Harmônica é um caso patológico de divergência. Se você somar os inversos dos naturais elevados a qualquer potência maior do que 1, a soma será convergente. Se for 1 ou menor do que 1 será divergente. Então, não existe um "menor" expoente r para o qual a soma dos inversos dos naturais elevados a r seja convergente. Como os primos são um subconjunto dos naturais, também não existe um "menor" expoente para o qual a soma dos inversos dos primos elevados a r seja convergente. Qualquer r maior do que 1 basta. O mesmo Euler provou, em 1736, que a soma dos inversos dos primos é divergente. Inteligente, este rapaz que tinha tudo para ter sido mais um Cearense...!A propósito, sendo a>0 por quê, quando n cresce indefinidamente,
  a^1/n tende a 1?Um abraço à todos!Quer uma internet mais segura? Baixe agora o novo Internet Explorer 8. É grátis!Novo Internet Explorer 8: mais rápido e muito mais seguro. Baixe agora, é grátis!


=
Instruções para entrar na lista, sair da lista e usar a lista em
http://www.mat.puc-rio.br/~obmlistas/obm-l.html
=


[obm-l] Re: [obm-l] FW: DIVERS ÕES COMBIN ATÓRIAS!

2009-04-01 Por tôpico lucianarodriggues
Pessoal
Não estou com tempo para ler os emails da lista, já solicitei o meu desligamento, mas as mensagens continuam chegando.
Gostaria de me desligar!
Abraços à todos
Em 01/04/2009 16:57, Jorge Luis Rodrigues e Silva Luis < jorgelrs1...@hotmail.com > escreveu:

.hmmessage P { margin:0px; padding:0px } body.hmmessage { font-size: 10pt; font-family:Verdana }
  

From: jorgelrs1...@hotmail.comTo: obm-l@mat.puc-rio.brSubject: DIVERSÕES COMBINATÓRIAS!Date: Wed, 1 Apr 2009 13:12:28 +
.ExternalClass .EC_hmmessage P {padding:0px;} .ExternalClass body.EC_hmmessage {font-size:10pt;font-family:Verdana;}
Turma! Alguém aí já resolveu o probleminha proposto pelo colega Thélio Gama quanto aos números de 6 algarismos distintos que podemos formar de modo que um algarismo par esteja sempre ao lado de pelo menos um algarismo ímpar? Fácil, não! Afinal! Quantos milhares sem algarismos repetidos podem ser formados com 2 algarismos pares e 2 ímpares significativos? Mais fácil ainda, não! Vejam abaixo outros bonitinhos... Dez homens e dez mulheres de alturas diferentes se colocam em fila, de todas as maneiras possíveis. Em quantas delas os homens se encontram dispostos por ordem crescente de alturas? Em quantas delas tanto homens como mulheres se acham dispostos por ordem crescente de alturas? Em 2 planos são marcados 5 e 6 pontos, respectivamente, nunca 3 deles em linha reta. A interseção dos 2 planos contém dois deles. Tomando como vértice um ponto da interseção, formamos pirâmides de base triangular com os vértices restantes fora da interse
 ção. Quantas pirâmides podemos formar? Um jogo de armar consta de 25 quadrículas que convenientemente justapostas formam um quadrado onde se desenhou uma paisagem. Quantas arrumações apresentam, pelo menos, as quadrículas das cantoneiras no lugar certo? (Essa é boa!)  Divirtam-se! 

Turbine seu Messenger com emoticons! Clique já, é GRÁTIS!

Turbine seu Messenger com emoticons! Clique já, é GRÁTIS! 
=
Instruções para entrar na lista, sair da lista e usar a lista em
http://www.mat.puc-rio.br/~obmlistas/obm-l.html
=


[obm-l] Re: [obm-l] FW: FRACIONALIZAÇÃO!

2009-01-30 Por tôpico Tarso de Moura Leitão
Obrigado pela atenção, apesar das dificuldades devemos sim procurar situações 
significativas para as crianças sem jamais pensar em abolir o ensino das 
frações, conforme tenho lido em algumas publicações.
Um abraço
Tarso.



[obm-l] Re: [obm-l] FW: RES: [obm-l] RE: [obm-l ] Ajuda Problema de Teoria dos Números

2008-08-21 Por tôpico luiz silva
Albert,
 
Obrigado pela ajuda.
 
Um Abraço,
Felipe

--- Em qui, 21/8/08, Albert Bouskela <[EMAIL PROTECTED]> escreveu:

De: Albert Bouskela <[EMAIL PROTECTED]>
Assunto: [obm-l] FW: RES: [obm-l] RE: [obm-l] Ajuda Problema de Teoria dos 
Números
Para: obm-l@mat.puc-rio.br
Data: Quinta-feira, 21 de Agosto de 2008, 14:10




#yiv1543679991 .hmmessage P
{
margin:0px;padding:0px;}
#yiv1543679991 {
FONT-SIZE:10pt;FONT-FAMILY:Tahoma;}


Veja a "COMPLEMENTAÇÃO" da minha reposta - está, também, abaixo (MELHORADA!):
 
  
Complementando e MELHORANDO minha resposta anterior:
 
x^3 + 3y = z^3
 
Logo:   z^3 – x^3 = 3y
 
Logo   (z^3 – x^3) é múltiplo de 3
 
“m” e “n” são inteiros quaisquer.
 
Logo   (x, z) = { (3n, 3m) ; (3n+1, 3m+1) ; (3n+2, 3m+2) } . Verifique!
 
I.e., verifique que se   (x, z) = { (3n, 3m+1) ; (3n, 3m+2) ; (3n+1, 3m) ; 
(3n+1, 3m+2) ; (3n+2, 3m) ; (3n+2, 3m+1) } , então   (z^3 – x^3)   não é 
múltiplo de “ 3” .
 
Apenas um exemplo:
x = 3n   ;   z = 3m+1
z^3 – x^3 = 3[9(m^3 + m^2 – n^3) + 3m] + 1   i.e.,   resto = 1   na divisão por 
“3”
 
Veja que analisei TODAS as condições de contorno possíveis!
 
y = (z^3 – x^3)/3
 
Finalmente:
[x, z, y] = { [3n, 3m, 9(m^3 – n^3)] ; [3n+1, 3m+1 , 9(m^3 + m^2 – n^3 – n^2) 
+3(m – n)] ; [3n+2, 3m+2, 9(m^3 + 2m^2 – n^3 – 2n^2) + 12(m – n)] }
 
Verifique TODAS as contas!

AB
[EMAIL PROTECTED]




De: [EMAIL PROTECTED] [mailto:[EMAIL PROTECTED] Em nome de luiz silva
Enviada em: quinta-feira, 21 de agosto de 2008 12:49
Para: obm-l@mat.puc-rio.br
Assunto: [obm-l] RE: [obm-l] Ajuda Problema de Teoria dos Números








Albert,
 
Valeu. 
 
Alguém saberia como chegar nessa resposta, provando que esta é a solução geral 
da equação ?
 
Abs
Felipe

--- Em qui, 21/8/08, Albert Bouskela <[EMAIL PROTECTED]> escreveu:

De: Albert Bouskela <[EMAIL PROTECTED]>
Assunto: RE: [obm-l] Ajuda Problema de Teoria dos Números
Para: obm-l@mat.puc-rio.br
Data: Quinta-feira, 21 de Agosto de 2008, 11:29




#yiv1543679991 .ExternalClass #EC_yiv722995640 .EC_hmmessage P
{padding-right:0px;padding-left:0px;padding-bottom:0px;padding-top:0px;}
#yiv1543679991 .ExternalClass #EC_yiv722995640
{font-size:10pt;font-family:Tahoma;}

Posso inferir que:
 
x = 3n
y = -9(n^3) + 9(m^3)
z = 3m

"m" e "n" são inteiros.
 
Acredito que esta seja a solução mais geral possível.

AB
[EMAIL PROTECTED]






Date: Thu, 21 Aug 2008 05:27:59 -0700
From: [EMAIL PROTECTED]
Subject: [obm-l] Ajuda Problema de Teoria dos Números
To: obm-l@mat.puc-rio.br







Alguém sabe como resolver  (dar a forma geral da solução..não uma solução 
específica) a equação diofantina abaixo :
 

x3  + 3y =  z3 



Um Abraço 

Felipe 







Novos endereços, o Yahoo! que você conhece. Crie um email novo com a sua cara 
@ymail.com ou @rocketmail.com.


Instale a Barra de Ferramentas com Desktop Search e ganhe EMOTICONS para o 
Messenger! É GRÁTIS! 


Novos endereços, o Yahoo! que você conhece. Crie um email novo com a sua cara 
@ymail.com ou @rocketmail.com.


Receba GRÁTIS as mensagens do Messenger no seu celular quando você estiver 
offline. Conheça o MSN Mobile! Crie já o seu! 


  Novos endereços, o Yahoo! que você conhece. Crie um email novo com a sua 
cara @ymail.com ou @rocketmail.com.
http://br.new.mail.yahoo.com/addresses

Re: [obm-l] Fw: [obm-l] soma de série

2008-03-11 Por tôpico saulo nilson
apos reduzir temos o caso, perdao.

2008/3/11 saulo nilson <[EMAIL PROTECTED]>:

> 1+1/2+1/3+1/4=(24+12+8+6)/24=par /par
>
> 2008/3/11 Luiz Alberto Duran Salomão <[EMAIL PROTECTED]>:
>
>
> > - Original Message - *From:* Luiz Alberto Duran Salomão<[EMAIL 
> > PROTECTED]>
> > *To:* obm-l@mat.puc-rio.br
> > *Sent:* Tuesday, March 11, 2008 10:21 AM
> > *Subject:* Re: [obm-l] soma de série
> >
> > Caros amigos,
> > Seja n um inteiro, com n>1. O que se quer provar é que
> > 1+1/2+1/3+ . . . +1/n   não é inteiro.
> > Seja 2^a a maior potência de 2 tal que 2^a é menor do que ou igual a n.
> > Assim, 1/2^a   aparece no somatório acima mas 1/2^(a+1)  não aparece.
> > Observe que o mínimo múltiplo comum dos denominadores dos
> > termos do somatório tem a potência 2^a como fator. Agora, no
> > numerador de cada  fração, já com denominador igual ao mínimo
> > múltiplo comum, temos sempre um número par, com exceção do
> > termo correspondente a 1/2^a. Logo, a soma dos numeradores
> > é ímpar o que nos leva a concluir que o somatório não é  um
> > número inteiro.
> > Abraços,
> > Luiz Alberto
> >
> >  - Original Message -
> > *From:* MauZ <[EMAIL PROTECTED]>
> > *To:* obm-l@mat.puc-rio.br
> > *Sent:* Monday, March 10, 2008 6:13 PM
> >  *Subject:* [obm-l] soma de série
> >
> > mostrar que 1+1/2+1/3+...+1/n não é inteiro pra qquer N>1.
> >
> > Obrigado!
> >
> > --
> >
> > No virus found in this incoming message.
> > Checked by AVG.
> > Version: 7.5.518 / Virus Database: 269.21.7/1324 - Release Date:
> > 10/3/2008 19:27
> >
> >
>


Re: [obm-l] Fw: [obm-l] soma de série

2008-03-11 Por tôpico Rodrigo Renji
mostrar que 1+1/2+1/3+...+1/n não é inteiro pra qquer N>1.

a questão original é uma soma finita e não uma série
a série sabemos que diverge, o que se queria  na demonstração e que a
soma finita acima
nunca é inteira
soma [k=1, n] 1/k

Em 11/03/08, saulo nilson<[EMAIL PROTECTED]> escreveu:
> 1+1/2+1/3+1/4=(24+12+8+6)/24=par /par
>
>
> 2008/3/11 Luiz Alberto Duran Salomão <[EMAIL PROTECTED]>:
>
>
> >
> >
> >
> >
> > - Original Message -
> > From: Luiz Alberto Duran Salomão
> > To: obm-l@mat.puc-rio.br
> > Sent: Tuesday, March 11, 2008 10:21 AM
> > Subject: Re: [obm-l] soma de série
> >
> >
> > Caros amigos,
> > Seja n um inteiro, com n>1. O que se quer provar é que
> > 1+1/2+1/3+ . . . +1/n   não é inteiro.
> > Seja 2^a a maior potência de 2 tal que 2^a é menor do que ou igual a n.
> > Assim, 1/2^a   aparece no somatório acima mas 1/2^(a+1)  não aparece.
> > Observe que o mínimo múltiplo comum dos denominadores dos
> > termos do somatório tem a potência 2^a como fator. Agora, no
> > numerador de cada  fração, já com denominador igual ao mínimo
> > múltiplo comum, temos sempre um número par, com exceção do
> > termo correspondente a 1/2^a. Logo, a soma dos numeradores
> > é ímpar o que nos leva a concluir que o somatório não é  um
> > número inteiro.
> > Abraços,
> > Luiz Alberto
> >
> >
> > - Original Message -
> > From: MauZ
> > To: obm-l@mat.puc-rio.br
> > Sent: Monday, March 10, 2008 6:13 PM
> >
> > Subject: [obm-l] soma de série
> >
> > mostrar que 1+1/2+1/3+...+1/n não é inteiro pra qquer N>1.
> >
> > Obrigado!
> >
> >
> >
> > 
>
> >
> > No virus found in this incoming message.
> > Checked by AVG.
> > Version: 7.5.518 / Virus Database: 269.21.7/1324 - Release Date: 10/3/2008
> 19:27
> >
> >
> >
> >
> >
>
>

=
Instruções para entrar na lista, sair da lista e usar a lista em
http://www.mat.puc-rio.br/~obmlistas/obm-l.html
=


Re: [obm-l] Fw: [obm-l] soma de série

2008-03-11 Por tôpico saulo nilson
1+1/2+1/3+1/4=(24+12+8+6)/24=par /par

2008/3/11 Luiz Alberto Duran Salomão <[EMAIL PROTECTED]>:

>
> - Original Message - *From:* Luiz Alberto Duran Salomão<[EMAIL 
> PROTECTED]>
> *To:* obm-l@mat.puc-rio.br
> *Sent:* Tuesday, March 11, 2008 10:21 AM
> *Subject:* Re: [obm-l] soma de série
>
> Caros amigos,
> Seja n um inteiro, com n>1. O que se quer provar é que
> 1+1/2+1/3+ . . . +1/n   não é inteiro.
> Seja 2^a a maior potência de 2 tal que 2^a é menor do que ou igual a n.
> Assim, 1/2^a   aparece no somatório acima mas 1/2^(a+1)  não aparece.
> Observe que o mínimo múltiplo comum dos denominadores dos
> termos do somatório tem a potência 2^a como fator. Agora, no
> numerador de cada  fração, já com denominador igual ao mínimo
> múltiplo comum, temos sempre um número par, com exceção do
> termo correspondente a 1/2^a. Logo, a soma dos numeradores
> é ímpar o que nos leva a concluir que o somatório não é  um
> número inteiro.
> Abraços,
> Luiz Alberto
>
>  - Original Message -
> *From:* MauZ <[EMAIL PROTECTED]>
> *To:* obm-l@mat.puc-rio.br
> *Sent:* Monday, March 10, 2008 6:13 PM
>  *Subject:* [obm-l] soma de série
>
> mostrar que 1+1/2+1/3+...+1/n não é inteiro pra qquer N>1.
>
> Obrigado!
>
> --
>
> No virus found in this incoming message.
> Checked by AVG.
> Version: 7.5.518 / Virus Database: 269.21.7/1324 - Release Date: 10/3/2008
> 19:27
>
>


Re: [obm-l] Fw: Resultados da Cone Sul

2007-06-18 Por tôpico JoaoCarlos_Junior
Professor Shine, outros:
O que atualmente se realiza no intuito de se minimizar dificuldades relacionadas à origem dos estudantes brasileiros?
Fraternalmente, João.
Oi gente, recebi boas notícias lá do Uruguai! Parabéns a todos! []'sShine- Forwarded Message From: Yuri Lima <[EMAIL PROTECTED]>To: OCM-L <[EMAIL PROTECTED]>; [EMAIL PROTECTED]; [EMAIL PROTECTED]; Carlos Shine <[EMAIL PROTECTED]>; Olimpíada de Matemática <[EMAIL PROTECTED]>Sent: Saturday, June 16, 2007 8:03:46 PMSubject: [conesul2006] Resultados da Cone SulOlá Chicos, A XVIII Olimpíada de Matemática do Cone Sul está terminada, e tivemos um saldo de: - 1 ouro (Renan); - 3 pratas. As provas foram realizadas na quinta e na sexta e as coordenaccoes e distribuiccao de medalhas foram feitas hoje de manha. Houve uma situaccao inusitada, com 5 medalhas de ouro, 6 de prata e 6 de bronze. Os 4 peruanos sacaram ouro, o que acabou fazendo com que ficássemos em segundo lugar geral na classificaccao dos países, seguidos pelos argentinos em terceiro. Estamos enviando arquivos anexos com os 6 problemas da prova e as pontuaccoes de todos os medalhistas. De qualquer forma, as pontuaccoes do Brasil foram: Renan - 50Marcelo - 44Thiago - 43Grazyelly - 36. Os cortes das medalhas foram:Ouro - 50Prata - 33Bronze - 25. Tais cortes se devem a grande saltos entre as pontuaccoes do último de uma faixa e o primeiro da faixa seguinte. Vocês podem ver no arquivo anexo. Levando em conta a inexperiência e as origens de três dos participantes (Barreiras, Varginha e Joinville), achamos que foi um ótimo resultado. Um abracco a todos, Yuri e Samuel.



Pinpoint customers who are looking for what you sell. __._,_.___[IMAGE]Suas configurações de e-mail: E-mail individual |Tradicional Alterar configurações via web (Requer Yahoo! ID) Alterar configurações via e-mail: Alterar recebimento para lista diária de mensagens | Alterar para completo Visite seu grupo | Termos de Uso do Yahoo! Grupos | Descadastrado __,_._,___



Ready for the edge of your seat? Check out tonight's top picks on Yahoo! TV. Instruções para entrar na lista, sair da lista e usar a lista em
http://www.mat.puc-rio.br/~nicolau/olimp/obm-l.html
===

Re: [obm-l] Fw: Triângulo.

2007-05-13 Por tôpico Marcelo Costa

Olá, tenham um bom dia e Feliz dia das Mães para as suas e para as que são.
Gostaria de que alguém me indicasse um livro de geometria com problemas que
necessitam de construções auxiliares para a solução, de preferência um livro
que já tenha  tais soluções. Caso ninguém saiba de nada parecido agradeceria
que me enviassem problemas isolados.
Muitíssimo obrigado!
Marcelo


Re: [obm-l] Fw: Triângulo.

2007-04-21 Por tôpico Rogerio Ponce
Ola' Anna Luisa,
o triangulo APB e' isosceles, de modo que o angulo APB = angulo B
Tambem o triangulo CPQ e' isosceles, e o angulo CPQ = angulo C
Logo, o angulo QPA = angulo A , pois 180-B-C = A
Alem disso, o angulo PQA e' igual a soma dos angulos da base do triangulo CPQ, 
e portanto vale 2C.
Olhando para o triangulo isosceles PQA, podemos entao dizer que os angulos da 
base valem 2C e o vertice vale A.
Dai vem
A + 2C + 2C= 180 , ou  seja, 2B+2C+2C=180
Alem disso sabemos que A+B+C=180, ou seja, 2B+B+C=180
Assim temos o sistema
2C+B=90
C+3B=180
Resolvendo, obtemos C=18 e B=54 , de onde A=108.
Portanto a resposta e' C=18.

[]'s
Rogerio Ponce

Anna Luisa <[EMAIL PROTECTED]> escreveu:
 - Original Message -  From: Anna  Luisa 
 To: Bruno OBM 
 Sent: Saturday, April 21, 2007 1:29 PM
 Subject: Triângulo.

 

  Olá.
 Acho que esse problema deve ser fácil, mas não  estou conseguindo de jeito 
nenhum.
 Alguém pode me dar uma ajudinha?
  
 ABC é um triângulo no qual o ângulo  é o dobro do  ângulo B; P e Q são pontos 
dos lados BC e AC tais que AB = AP = PQ = QC.  Determine o menor ângulo do 
trângulo.
  
 Obrigada.
 Anna.



 __
Fale com seus amigos  de graça com o novo Yahoo! Messenger 
http://br.messenger.yahoo.com/ 

[obm-l] Re: [obm-l]Fw: Olímpica de PA

2006-10-11 Por tôpico Artur Costa Steiner
A soma dos termos da PA é 140 + 161 = 301. A soma de termos equidistantes dos extremos eh igaula aa soma dos extremos. Sendo a_1 e a_n os termos inicial e final, temos a_1 + a_n = 43. A soma dos termos e S = (a_1 +a_n)*n/2 => 301 = 43*n/2 => n= 18.  
 
Acho que isso nao e olimpico nao
Artur
- Original Message From: Gustavo Duarte <[EMAIL PROTECTED]>To: obm-l@mat.puc-rio.brSent: Saturday, October 7, 2006 1:36:16 PMSubject: [obm-l]Fw: Olímpica de PA


 
Agreadeço desde ja se alguém puder alguma ajuda  
 
Em uma  certa PA a soma dos termos de ordem IMPAR é 140, a soma dos termos de ordem PAR é 161,e a soma de dois termos equidistantes dos extremos é 43. Qual o numero de termos desa PA ?    SOL. 14


Re: [obm-l] Fw: Olímpica de PA

2006-10-07 Por tôpico cleber vieira
Olá Gustavo, a soma de todos os termos da PA equivale a soma dos termos de ordem ímpar com os termos de ordem par.  Sn = ( A1 + An )*n / 2  logo, Sn = 301 como a soma de dois termos equidistantes dos extremos é 43 então (A1 + An) = 43 substituindo encontramos n = 14.  Abraços  CleberGustavo Duarte <[EMAIL PROTECTED]> escreveu:     Agreadeço desde ja se alguém puder alguma ajuda       Em uma  certa PA a soma dos termos de ordem IMPAR é 140, a soma dos termos de ordem PAR é 161,e a soma de dois termos equidistantes dos
 extremos é 43. Qual o numero de termos desa PA ?    SOL. 14 
		 
Yahoo! Search 
Música para ver e ouvir: You're Beautiful, do James Blunt

RE: [obm-l] Re: [obm-l] Re: [obm-l] Re: [obm-l] Fw: congru�ncia

2006-03-05 Por tôpico Qwert Smith
Talvez uma maneira menos tecnica mas que pelo menos pra mim e mais facil de 
ver e a seguinte:


10^2 = 8 mod 23

Queremos todo k tal que 10^k = 8 mod 23

n^(a+b) = n^a * n^b

se k = 2 + t entao 10^k = 10^2 * 10^t e 10^k mod 23 = 8 * 10^t mod 23

o problema entao se reduz a encontrar todos valores de t para os quais 10^t 
= 1 mod 23


Como eu tb nao entendo quase nada de congruencias eu sempre procuro chegar 
em a ~= 1 mod b que em geral da pra se resolver com conhecimentos minimos




From: "Marcelo Salhab Brogliato" <[EMAIL PROTECTED]>
Reply-To: obm-l@mat.puc-rio.br
To: 
Subject: [obm-l] Re: [obm-l] Re: [obm-l] Re: [obm-l] Fw: congruência
Date: Sun, 5 Mar 2006 00:20:37 -0300

Opa,
bem, nao consegui entender algumas coisas... se puder, por favor, da uma 
explicada um pouco melhor..


isso eu entendi:
10^11 = 10*(10^2)^5 = 10*8^5 = 10*16

mas nao entendi pq o fato de nao ser congruo a 1 (mod 23) faz com que 22 
seja o menor numero com essa propriedade!


Tbem nao entendi pq: 10^a = 10^b (mod 23) <=> a = b (mod 22)

abraços,
Salhab
  - Original Message -
  From: Marcio Cohen
  To: obm-l@mat.puc-rio.br
  Sent: Saturday, March 04, 2006 11:22 PM
  Subject: [obm-l] Re: [obm-l] Re: [obm-l] Fw: congruência


Como 23 eh primo, 10^22 = 1 (mod 23), e como 10^2 = 8 e 10^11 = 
10*(10^2)^5 = 10*8^5 = 10*16 != 1 (mod 23), 22 eh o menor numero com essa 
propriedade.

Logo, 10^a = 10^b (mod 23) se e somente se a = b (mod 22).
Como 10^2 = 8 (mod 23), a resposta é que os valores de k para os quais 
temos 10^k = 8 (mod 23) são exatamente os inteiros positivos que deixam 
resto 2 na divisão por 22 (2, 24, 46, ...)


  Abraços,
  Marcio


- Original Message -
From: Marcelo Salhab Brogliato
To: obm-l@mat.puc-rio.br
Sent: Saturday, March 04, 2006 7:06 PM
    Subject: [obm-l] Re: [obm-l] Fw: congruência


Olá,

vc quer saber para quais valores de k temos:
10^k = 8 (mod 23), certo?

bom, temos que:
100 = 8 (mod 23)
10^(2n) = 8^n (mod 23)
isso é, para k par temos que a unica solucao é k=2 (n=1).

ainda nao consegui extender essa solucao para k impar.. estou 
tentando!


PS: sei mto pouco sobre congruencia, talvez minha solucao esteja 
errada


abraços,
Salhab


  - Original Message -
  From: Leo
  To: obm-l@mat.puc-rio.br
  Sent: Saturday, March 04, 2006 12:16 AM
  Subject: [obm-l] Fw: congruência



  - Original Message -
  From: Leo
  To: obm-l@mat.puc-rio.br
  Sent: Friday, March 03, 2006 8:11 PM
  Subject: congruência


  Como resolver a seguinte congruência
  10^k cong 8 (mod 23) ... pra k=2 eh verdadeira mas como achar o caso 
geral???



=
Instruções para entrar na lista, sair da lista e usar a lista em
http://www.mat.puc-rio.br/~nicolau/olimp/obm-l.html
=


[obm-l] Re: [obm-l] Fw: [obm-l] Re: [obm-l] Fw: congru ência

2006-03-04 Por tôpico Marcelo Salhab Brogliato



cara, minha solucao esta errada! :) rs
da uma olhada na solucao do Cohen..
eu nao consegui entender.. to tentando ainda rs.. 
mas testei os valores que ele encontrou e deu certinho!!
 
abraços,
Salhab

  - Original Message - 
  From: 
  Leo 

  To: obm-l@mat.puc-rio.br 
  Sent: Saturday, March 04, 2006 9:32 
  PM
  Subject: [obm-l] Fw: [obm-l] Re: [obm-l] 
  Fw: congruência
  
  Eu estava fazendo algumas kestões do Cone-sul aih 
  vai a kestaum
   
  (Cone-Sul 1992 - Chile)ENCONTRE um número inteiro 
  positivo n de maneira tal que se na sua representaçaum decimal lhe eh colocado 
  um 2 à esquerda e um 1 à direita, o número resultante seja igual a 
  33n
   
  seja n... se colocarmos 2 a eskerda e 1 a direita 
  teremos
  suponhamos q n tenha k dígitos
  10n+1+2*10^(k+1)=33n
  q chega em 2*10^(k+1) + 1 = 23n
  ou seja devemos ter 2*10^(k+1) + 1 cong a 0 (mod 
  23) fazendo algumas operações chega-se em 10^k cong 8 (mod 23).. portanto se acharmos soluções para k acharemos m... Como 
  a kestaum pede para ENCONTRAR tems q k=2 q stisfaz a congruência acima nos dah 
  n=87 De fato 2871=33*87
  Pensei se poderíamos achar todas as soluções... 
  ou no contrário mostrar q tal soluçaum eh únik
   
  Grato
   
  Leonardo Borges Avelino
   
   
  - Original Message - 
  From: Marcelo Salhab 
  Brogliato 
  To: obm-l@mat.puc-rio.br 
  Sent: Saturday, March 04, 2006 7:06 PM
  Subject: [obm-l] Re: [obm-l] Fw: congruência
  
  Olá,
   
  vc quer saber para quais valores de k 
  temos:
  10^k = 8 (mod 23), certo?
   
  bom, temos que:
  100 = 8 (mod 23)
  10^(2n) = 8^n (mod 23)
  isso é, para k par temos que a unica solucao é 
  k=2 (n=1).
   
  ainda nao consegui extender essa solucao para k 
  impar.. estou tentando!
   
  PS: sei mto pouco sobre congruencia, talvez minha 
  solucao esteja errada
   
  abraços,
  Salhab
   
   
  
- Original Message - 
From: 
Leo 

To: obm-l@mat.puc-rio.br 
Sent: Saturday, March 04, 2006 12:16 
AM
Subject: [obm-l] Fw: congruência

 
- Original Message - 
From: Leo 
To: obm-l@mat.puc-rio.br 
Sent: Friday, March 03, 2006 8:11 PM
Subject: congruência

Como resolver a seguinte 
congruência
10^k cong 8 (mod 23) ... pra k=2 eh verdadeira 
mas como achar o caso 
geral???


[obm-l] Re: [obm-l] Re: [obm-l] Re: [obm-l] Fw: congru ência

2006-03-04 Por tôpico Marcelo Salhab Brogliato



Opa,
bem, nao consegui entender algumas coisas... se 
puder, por favor, da uma explicada um pouco melhor..
 
isso eu entendi:
10^11 = 10*(10^2)^5 = 10*8^5 = 
10*16
 
mas nao entendi pq o fato de nao ser congruo a 1 
(mod 23) faz com que 22 seja o menor numero com essa propriedade!
 
Tbem nao entendi pq: 10^a = 10^b (mod 23) <=> 
a = b (mod 22)
 
abraços,
Salhab

  - Original Message - 
  From: 
  Marcio Cohen 
  To: obm-l@mat.puc-rio.br 
  Sent: Saturday, March 04, 2006 11:22 
  PM
  Subject: [obm-l] Re: [obm-l] Re: [obm-l] 
  Fw: congruência
  
    Como 23 eh primo, 10^22 = 1 (mod 23), e 
  como 10^2 = 8 e 10^11 = 10*(10^2)^5 = 10*8^5 = 10*16 != 1 (mod 
  23), 22 eh o menor numero com essa 
  propriedade. 
    Logo, 10^a = 10^b (mod 23) se e somente se 
  a = b (mod 22).
    Como 10^2 = 8 (mod 23), a resposta é que 
  os valores de k para os quais temos 10^k = 8 (mod 23) são exatamente os 
  inteiros positivos que deixam resto 2 na divisão por 22 (2, 24, 46, 
  ...)
   
      Abraços,
      Marcio
   
   
  
- Original Message - 
From: 
Marcelo Salhab 
Brogliato 
To: obm-l@mat.puc-rio.br 
Sent: Saturday, March 04, 2006 7:06 
PM
Subject: [obm-l] Re: [obm-l] Fw: 
congruência

Olá,
 
vc quer saber para quais valores de k 
temos:
10^k = 8 (mod 23), certo?
 
bom, temos que:
100 = 8 (mod 23)
10^(2n) = 8^n (mod 23)
isso é, para k par temos que a unica solucao é 
k=2 (n=1).
 
ainda nao consegui extender essa solucao para k 
impar.. estou tentando!
 
PS: sei mto pouco sobre congruencia, talvez 
minha solucao esteja errada
 
abraços,
Salhab
 
 

  - Original Message - 
  From: 
  Leo 
  
  To: obm-l@mat.puc-rio.br 
  Sent: Saturday, March 04, 2006 12:16 
  AM
  Subject: [obm-l] Fw: 
congruência
  
   
  - Original Message - 
  From: Leo 
  
  To: obm-l@mat.puc-rio.br 
  Sent: Friday, March 03, 2006 8:11 PM
  Subject: congruência
  
  Como resolver a seguinte 
  congruência
  10^k cong 8 (mod 23) ... pra k=2 eh 
  verdadeira mas como achar o caso 
  geral???


[obm-l] Fw: [obm-l] Re: [obm-l] Fw: congruência

2006-03-04 Por tôpico Leo



Eu estava fazendo algumas kestões do Cone-sul aih 
vai a kestaum
 
(Cone-Sul 1992 - Chile)ENCONTRE um número inteiro 
positivo n de maneira tal que se na sua representaçaum decimal lhe eh colocado 
um 2 à esquerda e um 1 à direita, o número resultante seja igual a 
33n
 
seja n... se colocarmos 2 a eskerda e 1 a direita 
teremos
suponhamos q n tenha k dígitos
10n+1+2*10^(k+1)=33n
q chega em 2*10^(k+1) + 1 = 23n
ou seja devemos ter 2*10^(k+1) + 1 cong a 0 (mod 
23) fazendo algumas operações chega-se em 10^k cong 8 (mod 23).. portanto se acharmos soluções para k acharemos m... Como a 
kestaum pede para ENCONTRAR tems q k=2 q stisfaz a congruência acima nos dah 
n=87 De fato 2871=33*87
Pensei se poderíamos achar todas as soluções... ou 
no contrário mostrar q tal soluçaum eh únik
 
Grato
 
Leonardo Borges Avelino
 
 
- Original Message - 
From: Marcelo Salhab Brogliato 

To: obm-l@mat.puc-rio.br 
Sent: Saturday, March 04, 2006 7:06 PM
Subject: [obm-l] Re: [obm-l] Fw: congruência

Olá,
 
vc quer saber para quais valores de k 
temos:
10^k = 8 (mod 23), certo?
 
bom, temos que:
100 = 8 (mod 23)
10^(2n) = 8^n (mod 23)
isso é, para k par temos que a unica solucao é k=2 
(n=1).
 
ainda nao consegui extender essa solucao para k 
impar.. estou tentando!
 
PS: sei mto pouco sobre congruencia, talvez minha 
solucao esteja errada
 
abraços,
Salhab
 
 

  - Original Message - 
  From: 
  Leo 

  To: obm-l@mat.puc-rio.br 
  Sent: Saturday, March 04, 2006 12:16 
  AM
  Subject: [obm-l] Fw: congruência
  
   
  - Original Message - 
  From: Leo 
  To: obm-l@mat.puc-rio.br 
  Sent: Friday, March 03, 2006 8:11 PM
  Subject: congruência
  
  Como resolver a seguinte congruência
  10^k cong 8 (mod 23) ... pra k=2 eh verdadeira 
  mas como achar o caso geral???


[obm-l] Re: [obm-l] Re: [obm-l] Fw: congruência

2006-03-04 Por tôpico Marcio Cohen



  Como 23 eh primo, 10^22 = 1 (mod 23), e 
como 10^2 = 8 e 10^11 = 10*(10^2)^5 = 10*8^5 = 10*16 != 1 (mod 
23), 22 eh o menor numero com essa 
propriedade. 
  Logo, 10^a = 10^b (mod 23) se e somente se a 
= b (mod 22).
  Como 10^2 = 8 (mod 23), a resposta é que os 
valores de k para os quais temos 10^k = 8 (mod 23) são exatamente os inteiros 
positivos que deixam resto 2 na divisão por 22 (2, 24, 46, ...)
 
    Abraços,
    Marcio
 
 

  - Original Message - 
  From: 
  Marcelo Salhab 
  Brogliato 
  To: obm-l@mat.puc-rio.br 
  Sent: Saturday, March 04, 2006 7:06 
  PM
  Subject: [obm-l] Re: [obm-l] Fw: 
  congruência
  
  Olá,
   
  vc quer saber para quais valores de k 
  temos:
  10^k = 8 (mod 23), certo?
   
  bom, temos que:
  100 = 8 (mod 23)
  10^(2n) = 8^n (mod 23)
  isso é, para k par temos que a unica solucao é 
  k=2 (n=1).
   
  ainda nao consegui extender essa solucao para k 
  impar.. estou tentando!
   
  PS: sei mto pouco sobre congruencia, talvez minha 
  solucao esteja errada
   
  abraços,
  Salhab
   
   
  
- Original Message - 
From: 
Leo 

To: obm-l@mat.puc-rio.br 
Sent: Saturday, March 04, 2006 12:16 
AM
Subject: [obm-l] Fw: congruência

 
- Original Message - 
From: Leo 
To: obm-l@mat.puc-rio.br 
Sent: Friday, March 03, 2006 8:11 PM
Subject: congruência

Como resolver a seguinte 
congruência
10^k cong 8 (mod 23) ... pra k=2 eh verdadeira 
mas como achar o caso 
geral???


[obm-l] Re: [obm-l] Fw: congruência

2006-03-04 Por tôpico Marcelo Salhab Brogliato



Olá,
 
vc quer saber para quais valores de k 
temos:
10^k = 8 (mod 23), certo?
 
bom, temos que:
100 = 8 (mod 23)
10^(2n) = 8^n (mod 23)
isso é, para k par temos que a unica solucao é k=2 
(n=1).
 
ainda nao consegui extender essa solucao para k 
impar.. estou tentando!
 
PS: sei mto pouco sobre congruencia, talvez minha 
solucao esteja errada
 
abraços,
Salhab
 
 

  - Original Message - 
  From: 
  Leo 

  To: obm-l@mat.puc-rio.br 
  Sent: Saturday, March 04, 2006 12:16 
  AM
  Subject: [obm-l] Fw: congruência
  
   
  - Original Message - 
  From: Leo 
  To: obm-l@mat.puc-rio.br 
  Sent: Friday, March 03, 2006 8:11 PM
  Subject: congruência
  
  Como resolver a seguinte congruência
  10^k cong 8 (mod 23) ... pra k=2 eh verdadeira 
  mas como achar o caso geral???


Re: [obm-l] Fw: congruência

2006-03-04 Por tôpico Jefferson Franca
Vc já tentou fazer o seguinte: Chame 10^k de y e resolva a seguinte equação diofantina y - 23x = 8, acho que sai por aí.Leo <[EMAIL PROTECTED]> escreveu:     - Original Message -   From: Leo   To: obm-l@mat.puc-rio.br   Sent: Saturday, March 04, 2006 12:16 AM  Subject: Fw: congruência   - Original Message -   From: Leo   To: obm-l@mat.puc-rio.br   Sent: Friday, March 03, 2006 8:11 PM  Subject: congruênciaComo resolver a seguinte congruência  10^k cong 8 (mod 23) ... pra k=2 eh verdadeira mas como achar o caso geral???
		 
Yahoo! Acesso Grátis 
Internet rápida e grátis. Instale o discador agora!

Re: [obm-l] Fw: Probabilidade

2005-10-06 Por tôpico Felipe Takiyama
Olá!

Bem, vou tentar alguma coisa:
probabilidade de ganhar na 1ª tentativa: 3/6
na 2ª: 1/6 * 3/6
na 3ª: 1/6 * 1/6 * 3/6
.
.
.
na n-esima: 1/6 * 1/6 *...* 1/6 *3/6=[(1/6)^n-1]*3/6
Agora, basta somar todas as probabilidades, observando que é uma soma dos
infinitos termos de uma PG. Para um problema de olimpíada, isso é facil demais,
por isso deve haver algum engano na minha solução

Felipe


Citando fgb1 <[EMAIL PROTECTED]>:

>
> - Original Message -
> From: fgb1
> To: obm-l@mat.puc-rio.br
> Sent: Saturday, February 19, 2005 11:59 PM
> Subject: Probabilidade
>
>
> Uma roleta circular foi dividia em 6 setores de mesma área. Em 3 desses
> setores estava escrito: ganha o carro. Em 2 desses setores estava escrito:
> Perde o carro e no outro : jogue novamente. Qual é a probabilidade de um
> jogador ganhar o carro?
>
> O aluno me disse que era de uma olimpíada recente. Alguém, por acaso,
> reconhece a Olímpíada?
>



___
Quer 50% de desconto nas ligações DDD à noite e nos finais de semana ?? 
Plano SIM 21 da Embratel. Inscreva-se grátis. 
Mais informações acesse www.embratel.com.br.

=
Instruções para entrar na lista, sair da lista e usar a lista em
http://www.mat.puc-rio.br/~nicolau/olimp/obm-l.html
=


Re: [obm-l] Fw: Probabilidade

2005-10-06 Por tôpico Marcio Cohen



Do Rio de janeiro ;)

  - Original Message - 
  From: 
  Marcio Cohen 
  To: obm-l@mat.puc-rio.br 
  Sent: Thursday, October 06, 2005 12:44 
  AM
  Subject: Re: [obm-l] Fw: 
  Probabilidade
  
  Sim. A questão é da olimpíada estadual de 
  matemática de 2005, mas o enunciado não é exatamente assim (embora o sentido 
  seja esse).
  
- Original Message - 
From: 
fgb1 
To: obm-l@mat.puc-rio.br 
Sent: Wednesday, October 05, 2005 9:45 
PM
Subject: [obm-l] Fw: 
Probabilidade

 
- Original Message - 
From: fgb1 
To: obm-l@mat.puc-rio.br 
Sent: Saturday, February 19, 2005 11:59 PM
Subject: Probabilidade

Uma roleta circular foi dividia em 6 setores de 
mesma área. Em 3 desses setores estava escrito: ganha o carro. Em 2 desses 
setores estava escrito: Perde o carro e no outro : jogue novamente. Qual é a 
probabilidade de um jogador ganhar o carro?
 
O aluno me disse que era de uma olimpíada 
recente. Alguém, por acaso, reconhece a Olímpíada?
  


Re: [obm-l] Fw: Probabilidade

2005-10-05 Por tôpico Marcio Cohen



Sim. A questão é da olimpíada estadual de 
matemática de 2005, mas o enunciado não é exatamente assim (embora o sentido 
seja esse).

  - Original Message - 
  From: 
  fgb1 
  To: obm-l@mat.puc-rio.br 
  Sent: Wednesday, October 05, 2005 9:45 
  PM
  Subject: [obm-l] Fw: Probabilidade
  
   
  - Original Message - 
  From: fgb1 
  To: obm-l@mat.puc-rio.br 
  Sent: Saturday, February 19, 2005 11:59 PM
  Subject: Probabilidade
  
  Uma roleta circular foi dividia em 6 setores de 
  mesma área. Em 3 desses setores estava escrito: ganha o carro. Em 2 desses 
  setores estava escrito: Perde o carro e no outro : jogue novamente. Qual é a 
  probabilidade de um jogador ganhar o carro?
   
  O aluno me disse que era de uma olimpíada 
  recente. Alguém, por acaso, reconhece a Olímpíada?
    


Re: [obm-l] FW: Re: INTUIÇÕES PROBABILÍSTI CAS!

2005-09-05 Por tôpico Chicao Valadares
esse é conhecido como paradoxo do aniversario...a
quantidade de pessoas necessarias numa sala p/ que a
chance de duas delas façam aniversario no mesmo dia
com no minimo 50% de chance é sqrt(365) ~= 20
pessoas...   

isso é muito usado em fundamentos de criptografia p/ o
calculo da probabilidade de colisao de uma funçao(ou
conjunto de funçoes) dita aleatoria...
vc pode encontrar o calculo disso(bem interessante) na
pagina do curso de criptografia do professor mihir
bellare , do MIT...

Abraços.


> > >Suponhamos que existam trinta pessoas numa
> reunião. Qual é a 
> >probabilidade
> > >de duas delas terem nascido no mesmo dia (isto é,
> dia e mês e não
> > >necessariamente no mesmo ano)? Uma em doze talvez
> ou, digamos, uma em 
> >dez...
> >A probabilidade de todas nascerem em dias
> diferentes e'
>
>(365/365).(364/365).(363/365)(336/365)=0,2936837572807313400437603...,
> >e portanto a nossa probabilidade, que e' um menos
> isso, é
> >0,7063162427192686599562396..., ou seja, mais de
> 70%.
> >P.S..: Não estou considerando o efeito de anos
> bissextos.
> >
> > >   Abraços!
> > >
> >
>
>_
> > >Chegou o que faltava: MSN Acesso Grátis. Instale
> Já!
> > >http://www.msn.com.br/discador
> > >
> 
>
_
> Chegou o que faltava: MSN Acesso Grátis. Instale Já!
> 
> http://www.msn.com.br/discador
> 
>
=
> Instruções para entrar na lista, sair da lista e
> usar a lista em
> http://www.mat.puc-rio.br/~nicolau/olimp/obm-l.html
>
=
> 


"O Binômio de Newton é tão belo como a Vênus de Milo.
O que há é pouca gente para dar por isso... "
Fernando Pessoa - Poesias de Alvaro Campos

_
As informações existentes nessa mensagem e no(s) arquivo(s) anexado(s) 
são
para uso restrito, sendo seu sigilo protegido por lei. Caso não seja
destinatário, saiba que leitura, divulgação ou cópia são proibidas. 
Favor
apagar as informações e notificar o remetente. O uso impróprio será 
tratado
conforme as normas da empresa e a legislação em vigor. Agradecemos sua
colaboração.


The information mentioned in this message and in the archives attached 
are
of restricted use, and its privacy is protected by law. If you are not 
the
addressee, be aware that reading, disclosure or copy are forbidden. 
Please
delete this information and notify the sender. Inappropriate use will 
be
tracted according to company's rules and valid laws. Thank you for your
cooperation.

__
Converse com seus amigos em tempo real com o Yahoo! Messenger 
http://br.download.yahoo.com/messenger/ 
=
Instruções para entrar na lista, sair da lista e usar a lista em
http://www.mat.puc-rio.br/~nicolau/olimp/obm-l.html
=


Re: [obm-l] Fw: [obm-x] Construcao geometrica [era da obm-l]

2005-04-08 Por tôpico Daniel S. Braz
> Andaram pedindo referências sobre CG. A mais citada e que talvez a
> lista toda conheça é o livro do Wagner do IMPA. Há pouco descobri
> que tinha um livro que havia ficado dentro de um envelope e que
> gostei muito. Chama-se Desenho Geométrico e é publicado pela
> Biblioteca do Exército, Coleção Marechal Trompowski. Tenho o
> Volume 2 Tomo 1 e acho que a coleção é formada por 3 volumes.
> Alguém saberia dizer algo a respeito dessas publicações?
> Estão à venda? Onde?

esse livro do wagner está esgotado...infelizmente...(aliás, alguem
sabe se vai sair outra edição?qdo?)

o outro livro esta a venda em:
http://www.livronet.com.br/listagens/MATEM.htm

[]s
daniel

--

On Apr 8, 2005 4:31 PM, Luís Lopes <[EMAIL PROTECTED]> wrote:
> Sauda,c~oes,
> 
> Oi Claudio,
> 
> Recebi a mensagem abaixo num endereço que está pra caducar
> (pra onde você Claudio mandou). Favor escrever somente pro hotmail.
> 
> E também não sei o que ocorreu pois mandei o problema de cg
> abaixo pra obm-l e o Fábio respondeu via obm-x que não conheço.
> Será que é uma tentativa de separar alguns problemas da obm-l?
> 
> O fato é que a solução do Fábio é a que estava procurando.
> Percebi que tinha que construir um triângulo equilátero numa
> paralela à transversal mas não imaginei uma construção.
> 
> Quando mandei o problema queria mencionar outras coisas mas
> acabei esquecendo. Vão agora.
> 
> Andaram pedindo referências sobre CG. A mais citada e que talvez a
> lista toda conheça é o livro do Wagner do IMPA. Há pouco descobri
> que tinha um livro que havia ficado dentro de um envelope e que
> gostei muito. Chama-se Desenho Geométrico e é publicado pela
> Biblioteca do Exército, Coleção Marechal Trompowski. Tenho o
> Volume 2 Tomo 1 e acho que a coleção é formada por 3 volumes.
> Alguém saberia dizer algo a respeito dessas publicações?
> Estão à venda? Onde?
> 
> []'s
> Luís
> 
> >--
> >From: Fábio Dias Moreira <[EMAIL PROTECTED]>
> >Reply-To: [EMAIL PROTECTED]
> >Date: Thu, 7 Apr 2005 21:51:50 -0300
> >To: Claudio Buffara <[EMAIL PROTECTED]>
> >Subject: Re: [obm-x] Construcao geometrica
> >
> >[7/4/2005, [EMAIL PROTECTED]:
> > > Um da outra lista:
> >
> > > São dados um ângulo (imagine de 50 graus) e uma transversal
> > > cortando os dois lados do ângulo formando um triângulo de
> > > tamanho conveniente.
> >
> > > Trace um círculo tangente aos lados do ângulo e determinando
> > > na transversal uma corda de comprimento igual ao raio do círculo.
> >
> >Note que se deslizarmos a transversal, mantendo-a paralela à original,
> >obteremos soluções homotéticas. Logo basta achar uma solução para uma
> >dessas paralelas.
> >
> >Escolha um ponto O qualquer da bissetriz do ângulo. Trace duas retas r
> >e s que façam ângulos de 60 e -60 graus com a transversal. Trace a
> >circunferência de centro O que tangencia o ângulo. Trace as paralelas
> >a r e s que passam por O. Elas determinam pontos R, R', S e S' sobre a
> >circunferência. Evidentemente, ORS e OR'S' são equiláteros e RS é
> >paralelo à transversal original. Finalmente, fazemos uma homotetia
> >para levar a transversal RS (ou R'S') à transversal original,
> >encontrando duas soluções.
> >
> >[]s,
> >
> >--
> >Fábio Dias Moreira
> >
> >
> 
> =
> Instruções para entrar na lista, sair da lista e usar a lista em
> http://www.mat.puc-rio.br/~nicolau/olimp/obm-l.html
> =
> 


-- 
"A essência da Matemática reside na sua liberdade." (G. Cantor)

=
Instruções para entrar na lista, sair da lista e usar a lista em
http://www.mat.puc-rio.br/~nicolau/olimp/obm-l.html
=


Re: [obm-l] FW: Fwd: Caro penis... otemooo

2004-08-24 Por tôpico João Luís Gomes Guimarães
Será que teremos que ficar aguaentando spam nessa lista? Fala sério...


- Original Message - 
From: "Laurito Alves" <[EMAIL PROTECTED]>
To: <[EMAIL PROTECTED]>; <[EMAIL PROTECTED]>;
<[EMAIL PROTECTED]>; <[EMAIL PROTECTED]>; <[EMAIL PROTECTED]>;
<[EMAIL PROTECTED]>
Sent: Tuesday, August 24, 2004 8:22 AM
Subject: [obm-l] FW: Fwd: Caro penis... otemooo





>From: Anildo Antônio Alves Júnior <[EMAIL PROTECTED]>
>To: [EMAIL PROTECTED], [EMAIL PROTECTED], [EMAIL PROTECTED]
>Subject: Fwd: Caro penis... otemooo
>Date: Sat, 21 Aug 2004 13:26:12 -0300 (ART)
>
>
>
>
>
>
>
>
>___
>Yahoo! Mail agora com 100MB, anti-spam e antivírus grátis!
>http://br.info.mail.yahoo.com/

_
MSN Messenger: converse com os seus amigos online.
http://messenger.msn.com.br

=
Instruções para entrar na lista, sair da lista e usar a lista em
http://www.mat.puc-rio.br/~nicolau/olimp/obm-l.html
=


Re: [obm-l] Re: [obm-l] Fw:_sub-seq üência_de_{1,...,204}(*um problema parecido na Ibero*)

2004-05-15 Por tôpico Claudio Buffara
on 14.05.04 18:28, Domingos Jr. at [EMAIL PROTECTED] wrote:

> ok, matei o problema!!!
> o valor crítico é 65!!!
> 
> aqui eu demonstro que o limitante superior é 65, supondo que vcs
> demonstraram corretamente que o limite inferior é 65, acabou!
> 
> Seja S = {s_1, ..., s_n} contido em {1,2,3, ..., 2004}
> com s_1 < s_2 < ... < s_n
> defina d_i = s_{i+1} - s_i, i = 1..n-1
> 
> se para algum i, d_i = d_{i+j} com j >= 2
> s_{i+1} - s_i = s_{i+j+1} - s_{i+j}
> <=> s_{i+j} + s_{i+1} = s_{i+j+1} + s_i
> i < i + 1 < i+j < i+j+1 e portanto temos 4 elementos em S do jeito que
> queremos.
> 
> supondo que não tenhamos isso...
> seja 1 <= i < i + j < k < k + m <= n
> veja que
> s_i + s_{k+m} = s_{i+j} + s_k <=>
> s_{k+m} - s_k = s_{i+j} - s_i <=>
> (s_{k+m} - s_{k+m-1}) + (s_{k+m-1} - s_{k+m-2}) ... + (s_{k+1} - s_k) =
> (s_{i+j} - s_{i+j-1}) + ... + (s_{i+1} - s_i) <=>
> d_{k+m-1} + d_{k+m-2} + ... + d_k = d_{i+j-1} + ... + d_i
> 
> portanto provamos que duas somas consecutivas de elementos de {d_i},
> isoladas, não podem ser iguais.
> fixe um inteiro i > 1 e suponha a < i, b >= i com
> (1) d_a + ... + d_{i-1} = d_i + ... + d_b de forma que b - a é máximo
> 
> suponha que tenhamos também a' < i, b' >= i com
> d_{a'} + ... + d_{i-1} = d_i + ... + d_{b'}
> não podemos ter b' > b, pois o lado direito da soma seria maior que (1) e o
> lado esquerdo só poderia aumentar de tamanho contrariando a hipótese de b -
> a ser máximo.
> se i <= b' < b, a < a' < i e aí devemos ter
> d_a + ... + d_{a' - 1} = d_{b' + 1} + ... + d_b, o que é absurdo, pois essas
> são duas somas consecutivas isoladas.
> 
> então mostramos que para i = 2,..., n há no máximo 2 sub-seqüências
> consecutivas de mesma soma...
> 
> temos um total de Binomial(n-1, 2) subseqüências possíveis (basta escolher
> um índice para o começo e outro para o final) e no máximo n-1 são contadas
> duas vezes, ou seja, temos pelo menos Binomial(n-1, 2) + 1 - n subseqüências
> consecutivas com somas DISTINTAS, note que tais somas representam distâncias
> entre elementos de S e, como no nosso caso, S contido em {1,..., 2004}, há
> 2003 possíveis diferenças logo,
> 
> Binomial(n-1, 2) + 1 - n <= 2003
> (n-1)(n-2)/2 + (1-n) <= 2003
> (n-1)(n-4) <= 4006
> n <= 65
> 
> [ ]'s
> 
Oi, Domingos:

Li e reli o seu argumento e nao achei nenhum furo.
Se estiver mesmo OK (colegas da lista, por favor deem sua opiniao tambem),
ele terah sido um grande passo pra solucao.

Infelizmente, como o Auggy apontou, o limitante inferior de 65 estava
errado.
Logo, estamos com 38 <= Ncritico <= 66 (os dois limitantes sao seus).

Mas a melhor noticia eh que esse estah sendo um problema dificil no qual
varios participantes da lista estao pensando. Acho que isso eh um exemplo de
como essa lista pode ser interessante e valiosa.

[]s,
Claudio.


=
Instruções para entrar na lista, sair da lista e usar a lista em
http://www.mat.puc-rio.br/~nicolau/olimp/obm-l.html
=


[obm-l] Re: [obm-l] Fw:_sub-seq üência_de_{1,...,204}(*um problema parecido na Ibero*)

2004-05-14 Por tôpico Domingos Jr.
ok, matei o problema!!!
o valor crítico é 65!!!

aqui eu demonstro que o limitante superior é 65, supondo que vcs
demonstraram corretamente que o limite inferior é 65, acabou!

Seja S = {s_1, ..., s_n} contido em {1,2,3, ..., 2004}
com s_1 < s_2 < ... < s_n
defina d_i = s_{i+1} - s_i, i = 1..n-1

se para algum i, d_i = d_{i+j} com j >= 2
s_{i+1} - s_i = s_{i+j+1} - s_{i+j}
<=> s_{i+j} + s_{i+1} = s_{i+j+1} + s_i
i < i + 1 < i+j < i+j+1 e portanto temos 4 elementos em S do jeito que
queremos.

supondo que não tenhamos isso...
seja 1 <= i < i + j < k < k + m <= n
veja que
s_i + s_{k+m} = s_{i+j} + s_k <=>
s_{k+m} - s_k = s_{i+j} - s_i <=>
(s_{k+m} - s_{k+m-1}) + (s_{k+m-1} - s_{k+m-2}) ... + (s_{k+1} - s_k) =
(s_{i+j} - s_{i+j-1}) + ... + (s_{i+1} - s_i) <=>
d_{k+m-1} + d_{k+m-2} + ... + d_k = d_{i+j-1} + ... + d_i

portanto provamos que duas somas consecutivas de elementos de {d_i},
isoladas, não podem ser iguais.
fixe um inteiro i > 1 e suponha a < i, b >= i com
(1) d_a + ... + d_{i-1} = d_i + ... + d_b de forma que b - a é máximo

suponha que tenhamos também a' < i, b' >= i com
d_{a'} + ... + d_{i-1} = d_i + ... + d_{b'}
não podemos ter b' > b, pois o lado direito da soma seria maior que (1) e o
lado esquerdo só poderia aumentar de tamanho contrariando a hipótese de b -
a ser máximo.
se i <= b' < b, a < a' < i e aí devemos ter
d_a + ... + d_{a' - 1} = d_{b' + 1} + ... + d_b, o que é absurdo, pois essas
são duas somas consecutivas isoladas.

então mostramos que para i = 2,..., n há no máximo 2 sub-seqüências
consecutivas de mesma soma...

temos um total de Binomial(n-1, 2) subseqüências possíveis (basta escolher
um índice para o começo e outro para o final) e no máximo n-1 são contadas
duas vezes, ou seja, temos pelo menos Binomial(n-1, 2) + 1 - n subseqüências
consecutivas com somas DISTINTAS, note que tais somas representam distâncias
entre elementos de S e, como no nosso caso, S contido em {1,..., 2004}, há
2003 possíveis diferenças logo,

Binomial(n-1, 2) + 1 - n <= 2003
(n-1)(n-2)/2 + (1-n) <= 2003
(n-1)(n-4) <= 4006
n <= 65

[ ]'s

=
Instruções para entrar na lista, sair da lista e usar a lista em
http://www.mat.puc-rio.br/~nicolau/olimp/obm-l.html
=


Re: [obm-l] Fw:_sub-seq üência_de_{1,...,204}(*um problema parecido na Ibero*)

2004-05-14 Por tôpico Claudio Buffara
on 14.05.04 13:30, Qwert Smith at [EMAIL PROTECTED] wrote:

> 
>> From: Claudio Buffara <[EMAIL PROTECTED]>
> 
>> Entrementes, considere os conjuntos:
>> A = {1,2,3,5,9,15,20,25}
>> e
>> B = {49*x + y | x, y pertencem a A} = {50, 51, 52, ..., 1245, 1250}.
>> 
>> B tem 64 elementos e eh tal que quaisquer dois pares disjuntos de elementos
>> de B tem soma distinta. Isso leva nossa cota inferior para 65.
>> Ou seja, chegamos a 65 <= Ncritico <= 90.
> 
> Epa!  ki tal { 99, 50 } e { 98, 51 }?

Como voce consegue 98?
49*1 + 25 = 74   e   49*2 + 1 = 99


> Na verdade voce ta garantindo que B
> vai
> ter varios pares cuja diferenca e a mesma que a diferenca entre os elementos
> de A
> A = { a, b, c, d, e }
> B = { ...,N*a + c, N*a + d, ... , N*b + c, N*b + d, ... }
> 
> (N*b + d) + (N*a + c) = (N*a + d) + (N*b + c) para qualquer a,b,c,d ou N
> 
Por outro lado, essa objecao eh irrefutavel.

Primeiro a dos numeros triangulares e agora essa.
Eh! Hoje a coisa tah complicada...

[]s,
Claudio.

=
Instruções para entrar na lista, sair da lista e usar a lista em
http://www.mat.puc-rio.br/~nicolau/olimp/obm-l.html
=


Re: [obm-l] Fw:_sub-seq üência_de_{1,...,204}(*um problema parecido na Ibero*)

2004-05-14 Por tôpico Qwert Smith

From: Claudio Buffara <[EMAIL PROTECTED]>

Entrementes, considere os conjuntos:
A = {1,2,3,5,9,15,20,25}
e
B = {49*x + y | x, y pertencem a A} = {50, 51, 52, ..., 1245, 1250}.
B tem 64 elementos e eh tal que quaisquer dois pares disjuntos de elementos
de B tem soma distinta. Isso leva nossa cota inferior para 65.
Ou seja, chegamos a 65 <= Ncritico <= 90.
Epa!  ki tal { 99, 50 } e { 98, 51 }? Na verdade voce ta garantindo que B 
vai
ter varios pares cuja diferenca e a mesma que a diferenca entre os elementos 
de A
A = { a, b, c, d, e }
B = { ...,N*a + c, N*a + d, ... , N*b + c, N*b + d, ... }

(N*b + d) + (N*a + c) = (N*a + d) + (N*b + c) para qualquer a,b,c,d ou N
_
Check out the coupons and bargains on MSN Offers! http://youroffers.msn.com
=
Instruções para entrar na lista, sair da lista e usar a lista em
http://www.mat.puc-rio.br/~nicolau/olimp/obm-l.html
=


Re: [obm-l] Fw:_sub-seq üência_de_{1,...,204}(*um problema parecido na Ibero*)

2004-05-14 Por tôpico Claudio Buffara
Title: Re: [obm-l] Fw:_sub-seq üência_de_{1,...,204}(*um problema parecido na Ibero*)



Obrigado! 
Nao olhei em detalhe, mas parece ser relevante para o nosso problema.

Entrementes, considere os conjuntos: 
A = {1,2,3,5,9,15,20,25}
e
B = {49*x + y | x, y pertencem a A} = {50, 51, 52, ..., 1245, 1250}.

B tem 64 elementos e eh tal que quaisquer dois pares disjuntos de elementos de B tem soma distinta. Isso leva nossa cota inferior para 65.
Ou seja, chegamos a 65 <= Ncritico <= 90.

[]s,
Claudio.

on 14.05.04 10:19, Johann Peter Gustav Lejeune Dirichlet at [EMAIL PROTECTED] wrote:

Problema 3, 13a Iberoamericana.
Veja http://www.kalva.demon.co.uk/ibero/isoln/isol983.html

Claudio Buffara <[EMAIL PROTECTED]> wrote:
Qual Ibero? Qual Eureka?

on 13.05.04 17:12, Johann Peter Gustav Lejeune Dirichlet at [EMAIL PROTECTED] wrote:

Esta e apenas uma ideia vaga que pode ajudar: este problema e parecido com um da Iberoamericana. Ele esta na Eureka! e talvez na pagina do Scholes.Talvez a ideia da demonstraçao seja util.
Porque nao generalizar?
Ass.:Johann

"Domingos Jr." <[EMAIL PROTECTED]> wrote:
aqui vão, 37 elementos entre 1 e 2004 de forma que não existem a < b < c < d
com a+d = b+c.

1 2 3 5 8 13 21 30 39 53 74 95 128 152 182 212 258 316 374 413 476 531 546
608 717 798 862 965 1060 1161 1307 1386 1435 1556 1722 1834 1934

fiz uns testes com uns algoritmos aleatórios e nenhum deles deu mais que
37... será que a resposta do problema é 38?

olha o que eu estava pensando...

seja um S subcj. de [2004] em que não possamos tomar a < b < c < d com a + d
= b + c.
suponha que existam elementos a, a + k e b, b + k, com b != a, a + k
então, a + (b + k) = (a + k) + b e isso não pode ocorrer.
logo se uma diferença entre elementos de S se repete, só pode acontecer com
elementos
a, a + k, a + 2k
note que não podemos inserir a + 3k no conjunto, c.c., a + (a + 3k) = (a +
k) + (a + 2k)
ou seja, a diferenç! a k pode aparecer no máximo 2 vezes.

eu imagino que vcs já tinha percebido isso, né?


[ ]'s



TRANSIRE SVVM PECTVS MVNDOQVE POTIRI 

CONGREGATI EX TOTO ORBE MATHEMATICI OB SCRIPTA INSIGNIA TRIBVERE 

Fields Medal(John Charles Fields)
 
N.F.C. (Ne Fronti Crede)


Yahoo! Messenger - Fale com seus amigos online. Instale agora! 






Re: [obm-l] Fw:_sub-seq üência_de_{1,...,204}(*um problema parecido na Ibero*)

2004-05-14 Por tôpico Johann Peter Gustav Lejeune Dirichlet
Problema 3, 13a Iberoamericana.
Veja http://www.kalva.demon.co.uk/ibero/isoln/isol983.htmlClaudio Buffara <[EMAIL PROTECTED]> wrote:
Qual Ibero? Qual Eureka?on 13.05.04 17:12, Johann Peter Gustav Lejeune Dirichlet at [EMAIL PROTECTED] wrote:
Esta e apenas uma ideia vaga que pode ajudar: este problema e parecido com um da Iberoamericana. Ele esta na Eureka! e talvez na pagina do Scholes.Talvez a ideia da demonstraçao seja util.Porque nao generalizar?Ass.:Johann"Domingos Jr." <[EMAIL PROTECTED]> wrote:
aqui vão, 37 elementos entre 1 e 2004 de forma que não existem a < b < c < dcom a+d = b+c.1 2 3 5 8 13 21 30 39 53 74 95 128 152 182 212 258 316 374 413 476 531 546608 717 798 862 965 1060 1161 1307 1386 1435 1556 1722 1834 1934fiz uns testes com uns algoritmos aleatórios e nenhum deles deu mais que37... será que a resposta do problema é 38?olha o que eu estava pensando...seja um S subcj. de [2004] em que não possamos tomar a < b < c < d com a + d= b + c.suponha que existam elementos a, a + k e b, b + k, com b != a, a + kentão, a + (b + k) = (a + k) + b e isso não pode ocorrer.logo se uma diferença entre elementos de S se repete, só pode acontecer comelementosa, a + k, a + 2knote que não podemos inserir a + 3k no conjunto, c.c., a + (a + 3k) = (a +k) + (a + 2k)ou seja, a diferenç! a k pode aparecer no máximo 2 vezes.eu imagino que vcs já tinha percebido isso,
 né?[ ]'s
TRANSIRE SVVM PECTVS MVNDOQVE POTIRI CONGREGATI EX TOTO ORBE MATHEMATICI OB SCRIPTA INSIGNIA TRIBVERE Fields Medal(John Charles Fields)
 
N.F.C. (Ne Fronti Crede)Yahoo! Messenger - Fale com seus amigos online. Instale agora!

  1   2   >